Never doubt a small group of committed citizens can change the world. Indeed, it's the only thing that ever has.
Welcome to the AISLES Australian Law Network
A
ustralian Law Network.
I
nformation with Daily Court Digests & New Law Reviews
S
upport for you with any legal matters you are facing.
L
egal advice and services provided by qualified lawyers.
E
xpert Legal Document Templates and Drafting
S
ervices : ADR Processes, Mediation and more...

AISLES Australian Law Network provides a unique experience for members to have increased access to justice.

AISLES produces a daily digest of cases coming out of Australian Courts in all jurisdictions giving our members the ability to maintain or increase their knowledge of Australian Law in a social media style environment.

Lawyers, Mediators and other related professionals join to keep their fingers on the pulse of the Court and to get access to cases that have been digested for a quick read for lawyers and get to show case their skills and provide legal services to members.

Members with an interest in the law get to read the plain English summary to get a feel for whether they want to know more about the cases and have access to Lawyers, Mediators and other professionals providing services that they can interact with in a social media setting discretely using anonymous profiles or contact them directly with their real profiles.

AISLES specialises in increasing access to justice by providing specialised Alternative Dispute Resolution processes that include mediation and pre-trial mentoring programs that ensure you are fully prepared.

Want to find out more?  Book a free call to discuss your matter and discover the options available to you. 

Join AISLES as a subscribed member here:


CLICK HERE TO JOIN

New Discussions
  •  · 
Beginners Guide to the Australian Law Network 1.  Getting Started After you have created an acco…
The AISLES-AI Legal Research Assistant App is ready. It has been custom trained on Australian Law.…
  •  · 
How come the Australian Immigration authorities were not aware that this female slave was possibly a…
  •  · 
  •  · 
  •  · There are many people who arrive in Australia on various tourist, work or student visas that just go…
  •  · 
Just after views on this situation.Young person, now 19 accused of numerous crimes from 4 years prev…
  •  · 
  •  · 
  •  · Grame Thompson Nothing unusual in any of this. Legal aid will refer if any conflicts.As it is also a…
  •  · 
If a person obtains evidence improperly (ie stole it, logged in without authority to a person's acco…
  •  · 
  •  · 
  •  · Grame Thompson Cameron McKenzie Note different States have different rules of evidence for State bas…
Hi all, currently this is a popular topic of conversation; No Jab No Pay (family tax entitlements)…
Latest Posts (Gallery View)
Added a post 

This case, Shepherd v The State of South Australia [2024] SAET 2, involves an employee, Mr. Daniel Shepherd, who sought a review of a decision to reject his compensation claim for pericarditis which he claimed was caused by having a third dose of the COVID-19 vaccine. The respondent, the State of South Australia, admitted that the vaccine caused pericarditis but argued that the injection did not arise from employment but from a lawful State Government vaccination directive. The respondent also contended that any liability for any injury is excluded by legislation. The case was heard in the South Australian Employment Tribunal.

FACTS

The case in question is Shepherd v The State of South Australia (in right of the Department for Child Protection) [2024] SAET 2. The applicant, Daniel Shepherd, was a child and youth support worker employed by the Department for Child Protection (DCP). In line with a directive under the Emergency Management Act 2004 (SA), Shepherd was required to have a third dose of the COVID-19 vaccine to continue working.

After receiving this third dose on 24 February 2022, Shepherd experienced severe chest pain which was later diagnosed as post-vaccine pericarditis, an inflammation of the membrane surrounding the heart. As a result, he made a claim for weekly payments of income support and medical expenses which was rejected by the State of South Australia.

The state initially did not accept that the vaccine had caused the injury but later admitted that it had resulted in Shepherd's incapacity for work. However, they continued to defend against his claim on two grounds: 1) that the injury did not arise from employment within the meaning of s 7 of the Return to Work Act 2014 (SA), but rather from a direction given under the Emergency Management Act 2004 (SA), and 2) if s 7 of the RTW Act was satisfied, s 32A of the EM Act excludes any liability arising from a direction given under the EM Act or any act or omission by the state in relation to its management of COVID-19.

ISSUES

  1. Whether the applicant's pericarditis, which occurred following a third dose of the COVID-19 vaccine, arose from his employment or from a lawful State Government vaccination directive.
  2. Whether the respondent can avoid liability for any injury caused by the vaccination under Section 32A of the Emergency Management Act 2004 (SA) (EM Act).
  3. Whether the applicant's employment was a significant contributing cause of his work injury as per Section 7 of the Return to Work Act 2014.

ANALYSIS

Issue: The primary issue in this case was whether the pericarditis suffered by Mr. Shepherd following his third dose of COVID-19 vaccine was an injury arising out of employment, making him eligible for compensation under the Return to Work Act 2014 (RTW Act). Additionally, it was to be determined whether Section 32A of the Emergency Management Act 2004 (EM Act) could exclude any liability for the injury.

Rule: Under Section 7 of the RTW Act, employment must be a significant contributing cause of a work injury, but not necessarily its only or most significant cause. In contrast, Section 32A of the EM Act potentially excludes any liability arising from a direction given under the act or any act or omission of the state in managing the COVID-19 pandemic.

Application: Judge Calligeros ruled that Mr. Shepherd's injury resulted from both the vaccination mandate and his employment with DCP. While it was agreed that he had received the third dose due to a lawful State Government vaccination directive, his employment was still considered a significant contributing cause. This satisfied Section 7 of the RTW Act. As for Section 32A of the EM Act, it did not prohibit Mr. Shepherd's claim as it did not clearly and unambiguously lead to that conclusion. Rejecting his claim would not achieve the objectives of the EM Act.

Conclusion: The court held that Mr. Shepherd's injury arose from both a vaccination mandate and his employment, satisfying Section 7 of the RTW Act and thus making him eligible for compensation. Furthermore, it concluded that Section 32A of the EM Act did not exclude any liability for his injury.

Take Home Lesson: This case highlights that when determining compensation eligibility for injuries resulting from mandatory vaccinations under workplace law in Australia, both the circumstances leading to vaccination and legislative requirements need careful consideration. Employment does not need to be the sole or most significant cause of injury for compensation to be awarded, but a significant contributing factor. Furthermore, legislation that may potentially exclude liability needs to be unambiguous and clear in its intent.

 

Added a post 

AZC20 v Minister for Immigration, Citizenship, Migrant Services and Multicultural Affairs [2023] HCA 26 (6 September 2023)

Intro:-

This is an appeal from the Full Court of the Federal Court of Australia.

Facts:-

The underlying facts "reveal an extraordinarily long deprivation of the [appellant's] liberty by way of executive detention". The appellant, a citizen of Iran, arrived in Australia by boat in July 2013 and has been in immigration detention ever since, his protection visa application having been finally refused in February 2021

Following the final determination of his visa application in February 2021, the appellant commenced proceedings in the Federal Court of Australia against the Minister for Home Affairs and the Commonwealth of Australia seeking an order of habeas corpus or a writ in the nature of habeas corpus, a declaration that the appellant was falsely imprisoned, and other relief ("the habeas proceeding").He contended that he fell within s 198AD(2) of the Act, which requires that an officer take an "unauthorised maritime arrival" to whom the provision applies, as soon as reasonably practicable, from Australia to a regional processing country

The primary judge made a declaration that s 198AD(2) of the Act applied to the appellant (order 1), and ordered the Secretary to perform, or cause to be performed, the duty under s 198AD(2) of the Act to take the appellant from Australia to a regional processing country as soon as reasonably practicable (order 2) ("the s 198AD mandamus order").

Subsequently, the Minister for Home Affairs exercised the discretionary power in s 198AE of the Act to determine that the duty to remove to a regional processing country under s 198AD did not apply to the appellant ("the s 198AE Determination").About twelve days later, on 10 November 2021, the Commonwealth parties filed notices of appeal to the Full Court of the Federal Court from orders 1 to 5 and 8 of the 13 October 2021 order.

Full Court of the Federal Court

Before the Full Court, the Commonwealth parties made submissions on what they described as the "preliminary issue" of the utility of the appeals. The Commonwealth parties accepted that, irrespective of the outcome of the appeals and from the time of the s 198AE Determination, the duty in s 198(6) of the Act was applicable, and the s 198AD mandamus order and the home detention order ceased to have effect. However, the Commonwealth parties submitted that the appeals were not futile for two reasons. First, the issues in the appeals had a "substantive overlap" with the s 198 mandamus proceeding.

And second, even if the substantive issues were rendered moot, the Commonwealth parties emphasised that the Full Court retained a discretion to hear the appeals on the basis that there was a significant public interest because, as at 14 October 2021, there were approximately 130 persons potentially affected by the primary judge's conclusion that the applicable duty for persons in the appellant's position was the s 198AD duty and there were a number of proceedings on foot in which the issue of the Federal Court's power to make home detention orders was being agitated.

In its reasons for judgment, the Full Court considered the "preliminary issue" as a question of discretion, not jurisdiction. Relevantly, the Full Court held that the s 198AE Determination had "effectively" quelled the controversy between the parties about the application of s 198AD to the appellant and that this meant that "the order in the nature of mandamus was rendered inapplicable, and there was no basis for the [home detention order] to be carried into effect".

However, while the Full Court accepted that the s 198AD mandamus order had been rendered moot, the Full Court did not accept the appellant's submission that the primary judge's orders were entirely "arid" in respect of any effect on the appellant's own position. The Court observed that the appellant still had the s 198 mandamus proceeding before the primary judge, in which he was seeking mandamus to compel his removal to a country other than Iran, that "a decision about his status under the ... Act, and which removal provisions apply to him, may be relevant to the issues between the parties in relation to any outstanding relief"[16], and that the Federal Court's ability to make a home detention order "will also clarify some likely aspects of the proceeding still before the primary judge" (emphasis added).

The second reason given by the Full Court as to why it should deal with the Commonwealth parties' substantive arguments and proceed to determine the appeals was that the primary judge's orders and reasoning had "been employed in litigation relating to other individuals in similar circumstances, and other justices of [the Federal Court] have been invited to follow it".

The Full Court said:-

"Once that occurs, given the [Commonwealth parties'] position on the issues, it would place another single judge in a position of deciding if they are convinced the primary judge's orders and reasoning are wrong ... These appeals are a suitable vehicle to avoid single judges being faced with those issues of comity, which are not always straightforward." (emphasis added)

Issue:-

Where at time of appeals primary judge's orders did not have any operative legal effect, was there was a "matter" within meaning of Ch III of Constitution at time Full Court made orders determining appeals?

Consideration:-

What is a matter?

It is trite that federal jurisdiction arising from the subject matters in ss 75 and 76 of the Constitution is limited to deciding "matters". The original and appellate jurisdiction of the Federal Court is created by legislation passed under s 77(i) of the Constitution. Section 77(i) empowers Parliament to make laws, with respect to any of the matters mentioned in ss 75 and 76, defining the jurisdiction of any federal court other than the High Court. The need for there to have been a "matter" before the Full Federal Court for it to have had jurisdiction in the appeals was not in dispute[19].

It is well established that a "matter" does not mean a legal proceeding between parties or a bare description of a subject matter that falls within a head of federal judicial power in ss 75 and 76 of the Constitution. Rather, "matter" has two elements: "the subject matter itself as defined by reference to the heads of jurisdiction [in ss 75 and 76], and the concrete or adequate adversarial nature of the dispute sufficient to give rise to a justiciable controversy". There was no dispute in this case that the first element was satisfied.

As to the second element, as was most recently affirmed in Unions NSW v New South Wales ("Unions [No 3]"), "[e]xceptional categories aside, there can be no 'matter' within the meaning of Ch III of the Constitution unless 'there is some immediate right, duty or liability to be established by the determination of the Court' in the administration of a law and unless the determination can result in the Court granting relief which both quells a controversy between parties and is available at the suit of the party seeking that relief". The requirement to identify some "immediate right, duty or liability" to be established by the determination of the court "reinforces that the controversy that the court is being asked to determine is genuine, and not an advisory opinion divorced from a controversy". That requirement applies in both original and appellate jurisdiction.

Unions [No 3] was a matter in original jurisdiction. The concept of "matter" has been most often analysed in the context of original jurisdiction. That is unsurprising because, usually, the existence of a matter on appeal is uncontroversial. That requires explanation.

An appeal is against orders, not reasons for judgment. The respective rights, duties or liabilities of the parties have been determined by the orders that have been made by the court below, including, usually, an order as to costs. There has been an exercise of judicial power; the whole or part of the controversy between the parties has been quelled. Where a final judgment has been rendered, the rights and obligations in controversy, as between those persons, cease to have an independent existence: they "merge" in the final judgment and no action can be brought upon the extinguished rights and obligations. However, orders may be set aside on appeal where the primary judge is shown to have erred. An appellate court is then obliged, unless the matter is remitted for rehearing, to "give the judgment which in its opinion ought to have been given in the first instance".

On appeal, therefore, the question is not whether the party can establish the claimed legal right, duty or liability, as that question has been determined. The question is not whether the party continues to have the interest necessary to obtain relief, because that question has been overtaken by the grant of relief or by the refusal of relief. The question on appeal and for determination on appeal is whether the orders of the primary judge should be affirmed, varied or reversed – that is, whether the appeal should be allowed and, if so, what orders should be made in the place of the primary judge's orders. But the appellate court's supervisory function over the exercise of original jurisdiction by the primary judge is not an end in itself. The second element required to form a "matter" still applies – there must be a controversy over some immediate right, duty or liability. Usually, there is a live controversy because the orders of the primary judge continue to have effect in determining the parties' rights, duties or liabilities, unless set aside on appeal. In seeking to appeal the orders made at first instance, one or more of the parties are seeking to challenge the continuing effect of the orders on the determination of their respective rights, duties or liabilities. As will be explained, that critical feature – any controversy over the continuing effect of the orders on the parties' rights, duties or liabilities – was absent in the appeals before the Full Federal Court.

There was no matter in the Full Federal Court

The appellant submitted before this Court that there was no "matter" before the Full Federal Court because the orders that the Commonwealth parties sought to appeal had no operative legal effect by the time the Full Court determined the appeals. At the time the appeals were filed, Nauru had informed Australia it would not accept the appellant and the Minister had voluntarily engaged s 198AE such that s 198AD did not apply to the appellant. Since the home detention order was dependent on the s 198AD mandamus order, the events rendering the s 198AD mandamus order inoperative similarly made the home detention order inoperative. Even if there was a "matter" when the appeals were filed, there ceased to be a "matter" from the moment during the hearing when the Commonwealth parties undertook not to seek the costs of the trial or the appeals. Those submissions should be accepted.

Conclusion:-

The Full Court did not have jurisdiction to determine the appeals. The appeals to this Court should be allowed with costs.

 

 

Added a post 

Vunilagi v The Queen [2023] HCA 24 (8 August 2023)

Facts:

The appellant, with three co-accused, was charged with offences against ss 54 and 60 of the Crimes Act 1900 (ACT) ("the Crimes Act (ACT)"). His trial in the Supreme Court of the Australian Capital Territory was listed to commence on 7 September 2020. On 13 August 2020, Murrell CJ made an order under s 68BA(3) of the Supreme Court Act 1933 (ACT) that the proceeding be tried by judge alone.

The background to the enactment of s 68BA and the order made by Murrell CJ was the onset of the COVID‑19 pandemic and the requirements of public health emergency declarations, which had an impact on jury trials. In late March 2020, the Supreme Court directed that jury trials would proceed in limited numbers and subject to social distancing requirements[2], but subsequently directed that jury trials would not proceed until further notice[3].

Relevant legislation

Relevant to this appeal, the Emergency Response Act added s 68BA, which was in part in these terms:

"(1) This section applies to a criminal proceeding against an accused person for an offence against a territory law if the trial is to be conducted, in whole or in part, during the COVID-19 emergency period.

(2) To remove any doubt, this section applies—

(a) to a criminal proceeding—

(i) that begins before, on or after the commencement day; and

(ii) for an excluded offence within the meaning of section 68B(4); and

(b) whether or not an election has been made by the accused person under section 68B, including before the commencement day.

(3) The court may order that the proceeding will be tried by judge alone if satisfied the order—

(a) will ensure the orderly and expeditious discharge of the business of the court; and

(b) is otherwise in the interests of justice.

(4) Before making an order under subsection (3), the court must—

(a) give the parties to the proceeding written notice of the proposed order; and

(b) in the notice, invite the parties to make submissions about the proposed order within 7 days after receiving the notice."

On 18 June 2020, notice was given to the appellant and his co-accused (under s 68BA(4)), who were then invited to make submissions because Murrell CJ proposed to make an order under s 68BA(3). The appellant made submissions and opposed the order. By the time of the hearing, his co-accused supported the making of the order. Her Honour found that it was in the interests of justice that the trial proceed before a judge alone and, as noted at the outset of these reasons, made an order accordingly. In relation to the matter stated in s 68BA(3)(a), her Honour found that the trial could not proceed as a jury trial in accordance with social distancing requirements given, in particular, the number of legal representatives. The length of the trial rendered greater the likelihood of delays being caused as a result of COVID‑19 testing requirements and constraints. Her Honour considered that it was in the interests of the complainant, the witnesses and the accused that the matter be resolved expeditiously, noting that three accused were detained in custody.

Following his conviction, the appellant appealed to the Court of Appeal. He argued that his trial miscarried on the basis that s 68BA was invalid. The Court of Appeal dismissed the appeal.

Issues:-

a) Did s 68BA, in its continuing operation, contravene the limitation derived from this Court's decision in Kable v Director of Public Prosecutions (NSW); and

b) Is the section inconsistent with the requirement in s 80 of the Constitution that the appellant's mode of trial be by jury.

Consideration:-

The Kable principle

It is not in issue that laws enacted by the Legislative Assembly for the Australian Capital Territory which affect the functions and processes of the courts are subject to the Kable principle. The principle for which Kable stands, being the same for the courts of a Territory as it is with respect to courts of the States, is that:-

"because the Constitution establishes an integrated court system, and contemplates the exercise of federal jurisdiction by State Supreme Courts, State legislation which purports to confer upon such a court a power or function which substantially impairs the court's institutional integrity, and which is therefore incompatible with that court's role as a repository of federal jurisdiction, is constitutionally invalid".

The appellant's case is that the function given to the Supreme Court by s 68BA(4) impaired its institutional integrity by departing to a significant degree from the processes which characterise the exercise of judicial power. As noted above, the appellant focuses only upon what was involved in the process prescribed by sub-s (4). The appellant does not challenge the process undertaken under s 68BA(3).

The constitutional flaw which the appellant identifies in s 68BA, as relevant to the Kable principle, was in the "gatekeeping function" given to a judge under s 68BA(4) to determine the persons, from a relevantly identical class, to be subject to the exercise of the judicial function under sub-s (3). By contrast to the function under sub-s (3), the application of the gatekeeping function in sub-s (4) was inscrutable, the appellant submits. It exposed some, but not all, persons to the risk of losing a jury trial. This latter submission appears to be based upon an incorrect premise that s 80 confers something in the nature of a personal right to a trial by jury but this may be put to one side.

Section 68BA was general in its application. By s 68BA(1) and (2) it applied to all criminal proceedings where the trial was to be conducted during the COVID‑19 emergency period. Section 68BA(3) was central to the section and its purpose. It provided the power for the Supreme Court to determine if a criminal proceeding was to be tried by a judge alone. It was expressly subject to satisfaction of the two conditions that such an order: (a) would ensure the orderly and expeditious discharge of the business of the Court; and (b) was otherwise in the interests of justice. Section 68BA(4) added a further condition to the exercise of that power. It required that before an order for a judge alone trial was made: (a) a notice be given to the parties of the proposed order; and (b) the parties be invited to make submissions about the proposed order.

Properly construed, sub-ss (3) and (4) operated together. Far from operating as a "gatekeeping" function, sub-s (4) is to be understood as facilitative of and ancillary to the power which was to be exercised under sub-s (3). The function of the sub-sections was more in the nature of case management. The appellant does not deny this. By these means the Court was able to manage its criminal caseload during a public health emergency whilst at the same time ensuring that the interests of justice were served.

The function involved in 68BA(4) was not one to consider which criminal proceedings might be a candidate for an order under sub-s (3). It did not involve any assessment or evaluation of that kind. Its sole criterion was the circumstance that an order under sub-s (3) was proposed. As soon as such a proposal was made the Court came under a duty to provide the notice and the invitation referred to in sub-s (4).

The evident purpose of s 68BA(4), as the Explanatory Statement confirms, was to provide procedural fairness to any person who might be affected if the order proposed to be made under sub-s (3) was made. It ensured that no accused person would have their mode of trial altered without first being given notice of that proposal and the opportunity to be heard with respect to it. The appellant accepts that procedural fairness is required if a court's procedure can be said to conform to the Kable principle.

Ground 2: Section 80 of the Constitution

Section 80 of the Constitution, which appears in Ch III, provides in relevant part that "[t]he trial on indictment of any offence against any law of the Commonwealth shall be by jury ...". The offences in question here were against ss 54 and 60 of the Crimes Act (ACT). The first question which arises with respect to s 80 of the Constitution, the appellant accepts, is whether they are laws of the Commonwealth. The appellant also raises a second and alternative question. It is whether the reference in s 80 to "any law of the Commonwealth" includes a law made by the legislature of a territory. The answer to both questions is "no".

Conclusion:-

Appeal is dismissed.

Added a post 

CCIG Investments Pty Ltd v Schokman [2023] HCA 21 (2 August 2023)

Intro:-

This is an appeal from the Supreme Court of Queensland

Facts:-

In late 2016 the respondent, Mr Schokman, commenced employment with the appellant at Daydream Island Resort and Spa as a food and beverage supervisor. The island is part of the Whitsunday Islands, which are situated off the coast of Queensland. His employment contract contained a clause which stated "[a]s your position requires you to live on the island, furnished shared accommodation located at Daydream Island Resort and Spa will be made available to you while you are engaged in this position at a cost of $70 per week".

The contract also referred to a tenancy agreement and a number of other documents, such as an employee handbook, policies, practices and procedures, and Staff Village Regulations. Other than the contract itself, none of these documents were put into evidence, although the trial judge, Crow J, accepted that they had existed. It would seem that many documents were lost as a result of a cyclone which later affected the island.

Initially Mr Schokman was provided with a room to himself, but shortly thereafter a new worker, Mr Hewett, moved in and shared the accommodation with Mr Schokman. Mr Hewett's contract of employment was not in evidence. The case was conducted on the basis that Mr Hewett's contract of employment, so far as it related to accommodation, was in the same terms as Mr Schokman's contract. Both men worked at a restaurant within the resort. Mr Schokman held the superior position as supervisor; Mr Hewett was a team leader.

In the late evening of 6 November 2016, Mr Schokman spent some time at the staff bar. Mr Hewett came to the bar after finishing work at the restaurant. Mr Schokman observed Mr Hewett have a few drinks, but Mr Hewett did not seem overly intoxicated. Mr Schokman left the bar at approximately 1:00 am and returned to his room. Mr Hewett followed shortly afterwards. Mr Hewett was visibly upset and began complaining about his work environment and told Mr Schokman that he had issues with the management team. Mr Schokman said that he did not wish to discuss work issues at home and that they could talk about them at work the following day. Mr Hewett said that he would let Mr Schokman get some sleep and he left the unit, taking some drinks with him.

Mr Hewett returned at about 3:00 am. Mr Schokman heard him vomiting in the bathroom and then walking around whilst hiccupping. Mr Schokman went back to sleep. He was woken about 30 minutes later in a distressed condition and unable to breathe. Mr Hewett was standing over Mr Schokman's bed with his shorts pulled down and his penis exposed. He was urinating on Mr Schokman, who was inhaling the urine and choking. Mr Schokman yelled at Mr Hewett, who continued urinating on him for a short period of time and then stepped away. Mr Hewett went into the bathroom, and then came out and apologised to Mr Schokman. When Mr Schokman attempted to leave the room, Mr Hewett stood in front of him and apologised again.

Mr Schokman brought proceedings against the appellant. He claimed damages on two alternative bases. In the first place, he claimed damages based on a breach of the appellant's duty of care owed to him as an employee. The alternative claim was that the appellant was vicariously liable as employer for the negligent act of its employee, Mr Hewett. Both claims failed.

The claim for vicarious liability was the subject of an appeal to the Court of Appeal, and is the subject of this appeal. In some respects, however, the argument for Mr Schokman reflects a case of a duty of care owed by his employer to him. This may be seen especially in its focus on the position in which Mr Schokman was placed by the employment, rather than attention being directed to the position of Mr Hewett, and the connection between Mr Hewett's employment and his tortious act as relevant to vicarious liability.

The judgments below

The trial judge did not accept that the actions of Mr Hewett were committed in the course of his employment with the appellant[1]. His Honour considered that the relevant enquiry was as to whether there was a connection or nexus between the employment enterprise and the wrong that justified the imposition of vicarious liability on the employer for the wrong. Whilst his Honour accepted that the occasion for the tort committed by Mr Hewett arose out of the requirement of shared accommodation, his Honour did not consider that it was a fair allocation of the consequences of the risk arising to impose vicarious liability on the employer for the drunken misadventure of Mr Hewett with respect to his toileting. There was no history of Mr Hewett becoming intoxicated and nothing which would have put the employer on notice that Mr Hewett may have engaged in what was bizarre conduct[2].

The Court of Appeal allowed Mr Schokman's appeal[3]. McMurdo JA (Fraser and Mullins JJA agreeing) considered that the circumstances of this case were analogous to those in Bugge v Brown[4], where the employer had been held vicariously liable for the acts of the employee by reference to the terms of his employment. It was a term of Mr Hewett's employment that he reside in the staff accommodation and more particularly in the room assigned to him. He was occupying that room as an employee pursuant to, and under the obligations of, his employment contract, not as a stranger as referred to in Bugge v Brown[5]. It followed that there was the requisite connection between the employment and the employee's actions[6].

Issue:-

Was Mr. Hewett's wrongful act in course or scope of employment?

Consideration:-

In the course or scope of employment

For an employer to be held liable for the tort of an employee the common law requires that the tortious act of the employee be committed in the course or scope of the employment. In Prince Alfred College Inc v ADC this was described as an essential requirement of the common law. In Bugge v Brown, Isaacs J referred to it as a rule of the law. The necessity for it, as providing the parameters or outer limits of vicarious liability, has never been doubted. The principle upon which the rule is based is that it is just to make the employer, whose business the employee is carrying out, responsible for injury caused to another by the employee in the course of so acting, rather than to require that the other, innocent, party bear their loss or have only the remedy of suing the individual employee.

The common law of Australia adheres to the rule that the employee's wrongful act be done in the course or scope of employment in order for liability to attach to the employer. The rule has the advantage of being objective and rational, which probably explains why it has endured.

The question whether a tortious or other wrongful act was committed in the course or scope of employment depends on the circumstances of the particular case. Although this may be stated in simple terms, the reality is that in many cases the resolution of that question can prove difficult. As the principal joint judgment said in Prince Alfred College, the course or scope of employment "is to some extent conclusionary and offers little guidance as to how to approach novel cases". This is not a novel case: "[i]t is the nature of that which the employee is employed to do on behalf of the employer that determines whether the wrongdoing is within the scope of the employment". That is, "[i]t is the identification of what the employee was actually employed to do and held out as being employed to do that is central to any inquiry about course of employment". This enquiry requires consideration of "the conduct of the parties subsequent to the contract that establishes their relationship, especially the conduct of the person whose actions have caused the injury". Aspects of the course or scope of employment may be functional, geographical or temporal.

Whether an act was committed in the course or scope of employment is not determined by reference to whether the tortious employee's act can be said to have been authorised by the employer. An unauthorised, intentional or even criminal act may be committed in the course or scope of employment, and therefore render the employer liable. In that sense, the rule may have a broad operation. On the other hand, the law also recognises that it would be unjust to make the employer responsible for every act which the employee chooses to do, as Isaacs J said in Bugge v Brown. Most relevantly, an act done when the employee was on a "frolic of [their] own" will not attract liability. Consistently with the policy of the law, an employer should not be held liable for acts totally unconnected with the employment.

Where no more can be pointed to than that the employment provides an opportunity for the employee's wrongful act to take place, the connection with the employment is tenuous. Such a circumstance is to be distinguished from that where an employee is placed in a special position by reason of the employment so that the act in question may be seen as one to which the ostensible performance of the employer's work by the employee "gives occasion", to adopt the words of Dixon J in Deatons Pty Ltd v Flew. In such a circumstance the requisite connection would be present.

In Prince Alfred College, it was explained that in determining whether vicarious liability arises for an act of sexual abuse of a child that took place in a school or other institution, regard may be had to any special role the employer has assigned to the employee. Features of the employment such as authority, power, trust, control and the ability to achieve intimacy should be considered. Clearly a role embodying features of this kind may point to a strong connection between the employment and the wrongful act. The employment may be seen to provide more than a mere opportunity for the act to take place; it may provide the very occasion for it.

An analogy with Prince Alfred College?

Mr Schokman sought to draw an analogy between the circumstances in Prince Alfred College and those arising from the shared accommodation in his case. He contended that his compulsory housing with Mr Hewett made him vulnerable because he was required to sleep in a setting which was intimate.

The argument put for Mr Schokman misapprehends what was said in Prince Alfred College. In the passage from that case on which the argument relies, it was said that the appropriate enquiry concerning the sexual abuser was whether his role as a housemaster placed him in such a position of power and intimacy that the performance of his role could be said to give the occasion for his wrongful acts such that they could be said to have been committed in the course or scope of the employment. No such enquiry is presented by the circumstances of this case. Mr Hewett was not assigned any special role concerning Mr Schokman and no part of what Mr Hewett was employed to do was required to be done in the accommodation.

The most that could be said to arise from the circumstance of shared accommodation was that it created physical proximity between the two men. It therefore provided the opportunity for Mr Hewett's drunken actions to affect Mr Schokman. But, as has been seen, the cases hold that mere opportunity provides an insufficiently strong connection with the employment to establish vicarious liability.

An analogy with Bugge v Brown?

Mr Schokman also contended that an analogy might be drawn between this case and the case of Bugge v Brown. The two circumstances which he identifies as common to both cases are that the tortious act of the employee occurred whilst he was on a break from his employment and that each employee was fulfilling the requirements of his employment when carrying out the tortious act.

In the present case, Mr Hewett was at leisure and not at his place of work when he committed the tortious act. He was on a "break" only in the sense that it occurred outside the carrying out of his duties or in the period between carrying them out. The functional, geographical and temporal aspects of Mr Hewett's course or scope of employment were absent. In Bugge v Brown, the employee's act, lighting a fire, was in preparation for the employee's midday meal whilst working remotely. It occurred whilst he was carrying out his work. These comparisons may be put to one side.

Central to the case in Bugge v Brown was that the act of lighting the fire was itself a requirement of, and authorised by, the employment. By contrast in this case, Mr Hewett could only be said to be acting in accordance with his employment contract by sharing the accommodation provided for and being present in it. As has been explained, that does not provide a proper connection to the employment.

Isaacs J held it to be beyond question that the cooking of the meal was "intimately connected" with the performance of the day's task. The cooking of the meal by the employee was done "in the line of [his] employment". He was not on a "frolic of his own" in cooking the meal and that act was not so remote from the employment as directed that the employee could be regarded as a stranger in doing so. The most that could be said was that he lit the open fire in the paddock in disregard of an instruction.

Conclusion:-

The appeal should be allowed with costs. 

Added a post 

Disorganized Developments Pty Ltd v South Australia [2023] HCA 22 (2 August 2023)

Intro:-

This is an appeal from the Supreme Court of South Australia.

Facts:-

Section 83GD(1) of the Criminal Law Consolidation Act 1935 (SA) ("the 1935 Act") makes it an offence for a participant in a criminal organisation to enter or attempt to enter a "prescribed place". For the purposes of that offence, the Hells Angels motorcycle club is a criminal organisation. The second and third appellants, Mr Stacy and Mr Taylor, are members of that motorcycle club and, accordingly, they are participants in a criminal organisation for the purposes of s 83GD(1).

This appeal from a decision of the Court of Appeal of the Supreme Court of South Australia concerns the validity of two Regulations[3] ("the Cowirra Regulations") which each purport to declare a portion of land in Cowirra, east of Adelaide in South Australia ("the Cowirra land"), as a "prescribed place" for the purposes of the 1935 Act. The first appellant ("Disorganized Developments") is the registered proprietor of the two parcels of land that comprise the Cowirra land. Mr Stacy and Mr Taylor are the directors and only shareholders of Disorganized Developments and are occupiers of the Cowirra land.

Issue:-

Is the declaration power conditioned by a duty to afford procedural fairness to the appellants as owners and occupiers of the Cowirra land? Was the presumption displaced by statute?

Consideration:-

Appellants' rights and interests in the Cowirra land

As owners and occupiers, the appellants have property rights in the Cowirra land. Mr Stacy and Mr Taylor seek access to the Cowirra land in order to exercise their rights as occupiers including, from time to time, to reside on the land. Disorganized Developments also has interests in accessing the Cowirra land through its directors in order to maintain it and otherwise to discharge its obligations as owner of the land. The obligations include statutory obligations under various South Australian statutes and regulations and common law duties to protect invitees and trespassers from harm or injury arising from conditions on the land.

Decisions made in the exercise of statutory powers that affect the rights of individuals with respect to property are a category of cases that has a long history of attracting a duty of procedural fairness as a matter of "fundamental justice", "long-established doctrine" and a "deep-rooted principle of the law", subject to displacement by Parliament through express words or necessary implication in the relevant statute. Even so, the Court of Appeal found that the declaration power was not conditioned by a duty of procedural fairness.

Court of Appeal's reasons

In the Court of Appeal, the appellants contended that they were owed procedural fairness as owners and occupiers and as members of the Hells Angels motorcycle club as the criminal organisation associated with the Cowirra land. In this Court, the appellants relied only on their rights and interests as owners and occupiers.

The Court of Appeal acknowledged that the effect of declaring land to be a prescribed place may be to interfere severely with rights and obligations of ownership and occupation, and accepted that the appellants' rights were directly affected in a manner sufficient to give them standing to challenge the Cowirra Regulations, but did not consider that this was determinative of the question whether procedural fairness was required.

The Court of Appeal concluded that the legislature did not intend that there should be an obligation to afford procedural fairness to an owner or occupier of land, whether or not the owner or occupier is a participant in a criminal organisation, having regard to: (1) the "general and preventative policy [of Div 2] of disruption of an apprehended sphere of criminal activity", which the Court considered may be undermined by a duty of procedural fairness; (2) the incidental nature of the interests of owners and occupiers to the prohibition in s 83GD(1), and the lack of any necessary correspondence between these interests and interests of persons to whom the prohibition is directed; (3) the indeterminacy of the class of persons who are the subject of s 83GD(1), that is, participants in a criminal organisation; and (4) Parliament's initial determination by the 2015 Regulations to declare 16 places as prescribed places, in an exercise of "unequivocally legislative power that did not require procedural fairness to any person, such that it could not be inferred to have intended a differential obligation to owners and occupiers of places in the exercise of delegated legislative power to make further declarations.

Procedural fairness applies

The existence of a duty to afford procedural fairness is a question of statutory interpretation. In Twist v Randwick Municipal Council, Barwick CJ described the common law rule that a statutory authority having power to affect the rights of a person is bound to hear her or him before exercising the power as "both fundamental and universal", although subject to legislative displacement. Barwick CJ explained that, if it appears that the legislature "has not addressed itself" to the question of natural justice, the court will approach the task of statutory interpretation "with a presumption that the legislature does not intend to deny natural justice to the citizen", and "may presume that the legislature has left it to the courts to prescribe and enforce the appropriate procedure to ensure natural justice".

Since Twist, the law has evolved to include an established and "strong" common law presumption, generally applicable to any statutory power the exercise of which is capable of having an adverse effect on legally recognised rights or interests, that the exercise of the power is impliedly conditioned on the observance of procedural fairness. Consistent with the historical scope of the duty of procedural fairness, the core operation of the presumption requires the provision of procedural fairness where the relevant power directly affects the rights or interests of a particular individual. In such a case, the presumption operates "unless clearly displaced by the particular statutory scheme".

Notwithstanding the breadth of the stated presumption, there remain statutory powers that are not conditioned upon a duty to give procedural fairness. In particular, powers that affect individuals in an undifferentiated way from the general public may not attract an obligation of procedural fairness. Thus, in Kioa v West, members of this Court emphasised the need for people to be affected as individuals for procedural fairness to apply where a decision affects many people. Mason J adopted a distinction between a decision which directly affects the person individually and one which simply affects her or him as a member of the public or of a class of the public. Brennan J considered that (subject to displacement by the text of the statute, the nature of the power and the administrative framework created by the statute) the presumption "applies to any statutory power the exercise of which is apt to affect the interests of an individual alone or apt to affect his interests in a manner which is substantially different from the manner in which its exercise is apt to affect the interests of the public". Deane J referred to a direct effect on, relevantly, rights and interests of a person in her or his individual capacity, as distinct from a member of the general public or of a class of the general public.

This is not a case that requires consideration of the scope of procedural fairness in relation to a power liable adversely to affect a large group of persons. In this Court, the appellants' case was based squarely upon their individual property rights and interests that would be directly affected by a valid declaration of the blocks comprising the Cowirra land as prescribed places. Declarations of land as prescribed places affect owners and occupiers of the land to a significant degree, and in a manner markedly different from other persons who might be adversely affected by such a declaration, in the sense envisaged in Kioa. The possible interests of a broader class of participants in criminal organisations do not detract from the application of the presumption in this case.

South Australia argued that several features of the 1935 Act supported a conclusion that the declaration power is not conditioned by a duty to afford procedural fairness despite its capacity to affect the rights and interests of owners and occupiers of land, being: (1) the Governor in Council as the repository of the power; (2) the unfettered nature of the declaration power; (3) the Parliament's power to make regulations by statute, as evidenced by the 2015 Regulations; and (4) the provisions for parliamentary oversight of the declaration power. There was no suggestion that the rights and interests of the appellants were not a permissible consideration. Significantly, and contrary to the Court of Appeal's view, South Australia did not press that the legislative purpose of disrupting activities of criminal organisations might be frustrated or compromised by the affording of that procedural fairness.

The features of the 1935 Act identified by South Australia are insufficient, individually and cumulatively, to establish an intention to displace the common law presumption.

Since FAI Insurances Ltd v Winneke, it has been established that the mere vesting of decision-making authority in the Governor in Council is not a sufficient manifestation of a legislative intention to exclude the duty of procedural fairness.

In South Australia v O'Shea, Mason CJ did not discern a legislative intention to exclude a duty to act fairly by the vesting of authority in the Governor in Council, at least where the relevant decision was not to be made principally by reference to issues of general policy. In concluding that the critical question was whether Mr O'Shea had an adequate opportunity to make submissions, Mason CJ rejected an argument that the political or policy aspects of the decision placed it outside the ambit of procedural fairness. Rather, there were issues on which Mr O'Shea was not entitled to be heard. Wilson and Toohey JJ, and Brennan J, focused on the nature of the decision and the political responsibility attaching to the determination of the public interest, rather than the repository of the power. Writing in dissent, Deane J rejected the identity of the decision maker as a relevant factor.

In O'Shea, Mason CJ also addressed the significance of the participation by Cabinet in the process by which the South Australian Governor in Council exercises statutory power. He rejected a submission that the primarily political, social and economic concerns of Cabinet would deny the existence of a duty to act fairly in a matter which turned on considerations "peculiar to the individual". He also rejected the further objection that the private and confidential nature of Cabinet processes precludes the imposition of a duty to act fairly, observing that, in an appropriate case, a requirement that there be placed before Cabinet submissions of an individual affected by the relevant decision would not intrude upon Cabinet's control of its own proceedings.

Contrary to South Australia's submission, in the absence of any clear words that would displace the presumptive obligation to afford procedural fairness, the broad scope of the regulation‑making power conferred by s 370 has no significance. While there are earlier decisions supportive of the view that the unfettered nature of a power is inconsistent with a duty of procedural fairness, requirements of procedural fairness conditioning the exercise of unfettered statutory powers are now commonplace.

There is no reason to conclude that the scope of the regulation-making power is unconstrained by a duty of procedural fairness simply because the exercise of legislative power is not so constrained. Similarly, the general and limited oversight of the regulation‑making power by a Parliamentary Committee and the availability of disallowance are not a source of an implication to exclude procedural fairness: South Australia did not suggest that oversight of this kind was likely to afford procedural fairness to owners or occupiers, or that it would involve consideration of matters that might be raised by an owner or occupier if procedural fairness was afforded and that might avoid the arbitrary exercise of the regulation‑making power.

Content of the obligation to afford procedural fairness

A distinction is sometimes drawn between exercises of statutory power that concern considerations personal to an individual and those that concern issues of general policy. Undoubtedly, the focus of the scheme which includes the declaration power is the disruption of criminal activity. In that context, considerations personal to the owners and occupiers of land ordinarily can be expected to be secondary to broader policy considerations. However, the proper exercise of the declaration power requires the identification of facts to connect the proposed prescribed place with the purpose of disruption. In this way, the exercise of the declaration power is an application of the policy to disrupt criminal activity, rather than the formulation of policy.

There is no reason to doubt that an owner or occupier may have something to say of relevance about the characteristics of the land or its uses, or about possible adverse impacts of declaring a place as a prescribed place, which might affect an assessment of whether to make such a declaration.

Procedural fairness under this scheme requires reasonable notice to an owner or occupier of a proposal to declare a place a prescribed place, to give them an opportunity to supply information or make submissions as to matters within their knowledge as an owner or occupier that may be relevant to a decision to exercise the declaration power. It does not require an opportunity to make more general submissions as to the likely efficacy of the exercise of the declaration power in disrupting serious criminal activity. Accordingly, this is not a case where procedural fairness would require owners or occupiers to be informed of the nature or content of information that might form the basis of a recommendation to the Governor to declare a place to be a prescribed place.

Orders:-

The appeal should be allowed. The orders of the Court of Appeal should be set aside and, in lieu thereof, it should be declared that the Cowirra Regulations are invalid. South Australia should pay the appellants' costs in this Court and below.

Added a post 

Minister for Immigration, Citizenship, Migrant Services and Multicultural Affairs v Thornton [2023] HCA 17 (14 June 2023)

Intro:-

The Full Court of the Federal Court of Australia concluded that the appellant's decision under s 501CA(4) of the Migration Act 1958 (Cth) refusing to revoke a decision to cancel the respondent's visa gave rise to jurisdictional error because the appellant took into account a consideration made irrelevant by s 85ZR(2) of the Crimes Act 1914 (Cth) – namely the respondent's offending as a child for which no conviction was recorded – when, under s 184(2) of the Youth Justice Act 1992 (Qld), the respondent was taken never to have been convicted of any of those offences committed as a child.

Facts:-

The respondent, Mr Thornton, is a citizen of the United Kingdom who arrived in Australia with his family in 1999 on an Australian parent visa when he was three years old. He has lived in Australia since that time on a succession of temporary visas, the last of which was a Class BB Subclass 155 Five Year Resident Return visa, granted in 2011.

At the age of 16, Mr Thornton appeared three times in the Queensland Childrens Court and was found guilty of five offences: "failure to appear in accordance with undertaking", for which he was reprimanded; three offences – "going armed so as to cause fear", "serious assault police biting/spitting/applied bodily fluid/faeces", and "assault or obstruct police officer" – for which he was placed on a probation period of six months; and "assault or obstruct police officer", for which he was placed on a good behaviour bond for a period of six months. When he was 17, Mr Thornton appeared in a Queensland Magistrates Court and was found guilty of an offence of "commit public nuisance", for which he was ordered to pay a fine.

No conviction was recorded for any of those offences.

From when Mr Thornton turned 18 in September 2014 up until February 2018, at the age of 21, he was found guilty of a range of offences, including possessing dangerous drugs and property suspected of being connected with drugs offences, public nuisance, assaulting or obstructing police officers, failure to appear, drunk and disorderly behaviour, domestic violence offences, and several contraventions of domestic violence orders. He received various sentences for those offences, ranging from fines with community service or good behaviour periods without any conviction being recorded, to periods of imprisonment ranging from two to 18 months.

On 21 February 2018, Mr Thornton's visa was cancelled under s 501(3A) of the Migration Act as a delegate of the Minister was satisfied that he did not pass the character test because he had a substantial criminal record, and he was serving a sentence of imprisonment on a full-time basis ("the cancellation decision"). In the Minister's notice of visa cancellation, Mr Thornton was invited to make representations about revocation of the cancellation decision under s 501CA(4) of the Migration Act.

Minister's decision

Before stating that conclusion, the Minister made several observations concerning Mr Thornton's offending, including his childhood offending. The Minister first noted that he had assessed all of the information in the attachments to his reasons, which included Mr Thornton's juvenile criminal history, and in particular Mr Thornton's representations, which included acknowledgment of his childhood offending. The Minister noted that although he held the view that the Australian community might afford a higher tolerance of Mr Thornton's criminal conduct given that he had lived in Australia most of his life since he was three years old, he thought that "would be offset to at least some degree by the fact that he began offending as a minor and had a number of offences recorded before reaching adulthood". The Minister noted that Mr Thornton "has a history of mainly drug-related and violent offences since he was 16 years old" and referred to Mr Thornton having been fined and placed on probation, including as a result of his appearances in "juvenile courts", for offences "without any convictions being recorded". The Minister also stated that he considered that the fact that Mr Thornton had "repeatedly committed offences of or related to domestic violence, and other assault offences add[ed] more gravity to his offending" (emphasis added). Finally, in his concluding remarks, the Minister stated that he "gave significant weight to the serious nature of the crimes committed by Mr THORNTON, that are of a violent nature". As Mr Thornton's criminal history makes clear, the majority of his juvenile offences involved some form of violence.

Unsurprisingly, there was no dispute that the Minister had taken into account findings of guilt in relation to offences committed when Mr Thornton was a child for which no convictions were recorded. That concession was properly made.

Irrelevant consideration

The next question was whether that consideration – findings of guilt in relation to offences committed when Mr Thornton was a child for which no convictions were recorded – was an irrelevant consideration.

The requirement to read, identify, understand and evaluate the representations does not detract from established principle in respect of the types of errors that may be jurisdictional, including where a decision-maker took into account an irrelevant consideration. Accordingly, although the Minister was required to consider Mr Thornton's representations, which included references to his juvenile offending, if Mr Thornton's childhood offences for which no convictions were recorded were an irrelevant consideration for the purpose of the Minister coming to his decision as to whether to revoke the cancellation decision, he would be in error to consider those offences. Determining whether such an error occurred turns on the proper construction of s 85ZR(2) of the Crimes Act read with s 184(2) of the Youth Justice Act.

The task of construction must start with the text of each provision, having regard to its context and purpose. Further, the context is to be considered "at the first stage of the process of construction", where context is to be understood in its widest sense as including "surrounding statutory provisions, what may be drawn from other aspects of the statute and the statute as a whole".

Issue:-

Is s 184(2) of the Youth Justice Act a law of a State under which a person is, in particular circumstances or for a particular purpose, to be taken never to have been convicted of an offence under a law of that State?

Consideration:-

Relevant legislation

Section 85ZR(2) of the Crimes Act provides that:-

"Despite any other Commonwealth law or any Territory law, where, under a State law or a foreign law a person is, in particular circumstances or for a particular purpose, to be taken never to have been convicted of an offence under a law of that State or foreign country:

(a) the person shall be taken, in any Territory, in corresponding circumstances or for a corresponding purpose, never to have been convicted of that offence; and

(b) the person shall be taken, in any State or foreign country, in corresponding circumstances or for a corresponding purpose, by any Commonwealth authority in that State or country, never to have been convicted of that offence."

Section 184(2) of the Youth Justice Act provides that:-

"Except as otherwise provided by this or another Act, a finding of guilt without the recording of a conviction is not taken to be a conviction for any purpose."

Intersection of s 85ZR of the Crimes Act and Youth Justice Act

Section 184 of the Youth Justice Act engages s 85ZR of the Crimes Act. The "particular circumstance[]" – the condition – referred to in s 85ZR(2) is found in s 184(2): a finding of guilt against a child has been made, and the court has decided, or been mandated, under s 183 not to record a conviction. Section 184(2) deems a person never to have been convicted of an offence and takes away the adverse consequences which attend a conviction. In sum, consistent with s 85ZR of the Crimes Act, ss 183 and 184 of the Youth Justice Act prescribe a particular circumstance in which a person – a child – is taken never to have been convicted of an offence under the law of Queensland.

Accordingly, s 85ZS(1)(d)(ii) of the Crimes Act engages s 184(2) of the Youth Justice Act so that the Minister could not take into account under s 501CA(4) of the Migration Act any of the findings of guilt made against Mr Thornton when he was a child for which no convictions were recorded, and, having regard to the express wording of s 85ZS(1)(d)(ii) and the expanded definition of when "a person shall be taken to have been convicted of an offence", the Minister could not take into account that Mr Thornton had been charged with offences committed when he was a child for which no convictions were recorded. The Minister therefore took into account an irrelevant consideration.

Jurisdictional error

The next question is whether the Minister's error in taking into account the irrelevant consideration of the findings of guilt made against Mr Thornton when he was a child, for which no convictions were recorded, was "material" in the sense that it deprived Mr Thornton of a realistic possibility that the decision made by the Minister could have been different if that irrelevant consideration had not been taken into account, so as to give rise to jurisdictional error.

The starting point, and what is critical, is the nature of the error in this case. Consideration of the nature of the error involves identifying the relevant "historical facts" as to what occurred in the making of the Minister's decision.

There is no bright line to be drawn to determine whether the particular error in a given case falls into one of the categories of error identified by the principal joint judgment in MZAPC v Minister for Immigration and Border Protection which necessarily result in "a decision exceeding the limits of decision‑making authority without any additional threshold [of materiality] needing to be met" by an applicant. The nature of the error has to be worked out in each case in the context of a particular decision under a particular statute; that is, a determination of whether the decision could have been different had the error not occurred "cannot be answered without determining the basal factual question of how the decision that was in fact made was in fact made".

The error in this case was "relevant to the actual course of the decision‑making". It was not suggested that it was an error that was, without any additional threshold, necessarily material. In this case, the course of the Minister's decision-making reveals that his taking into account of the findings of guilt made against Mr Thornton as a child, without convictions being recorded, infected the whole of his reasoning in coming to the conclusion that there was not "another reason" why the cancellation decision should be revoked.

Conclusion:-

The appeal be dismissed with cost.

Added a post 

Did visa criterion in cl 790.227 of Sch 2 of the Migration Regulations 1994 (Cth) permit Minister to refuse to grant visa solely on ground that it is not in national interest to grant SHEV visa to person convicted of people smuggling?

In a landmark decision, an Iranian plaintiff's appeal for a SHEV visa was upheld, overturning the prior refusal by the Minister for Home Affairs. The court ruled that the Minister's refusal based on national interest was inappropriate. A victory for justice, this case sets a precedent for future immigration rulings. JusticePrevails VisaVictory ImmigrationReform


ENT19 v Minister for Home Affairs [2023] HCA 18 (14 June 2023)

Intro:

The plaintiff sought judicial review of the Decision in the original jurisdiction of this Court, seeking various remedies on different grounds, including writs of habeas corpus, mandamus and certiorari, and declarations relating to the validity and construction of cl 790.227. During the oral hearing, the plaintiff also sought a declaration that he satisfies the criteria for the grant of a SHEV.

Facts:

The plaintiff, a citizen of Iran, arrived in Australia by boat in December 2013 and was immediately detained under s 189 of the Migration Act 1958 (Cth). On 3 February 2017, the plaintiff made a valid application for a temporary protection visa – a Safe Haven Enterprise (Class XE) Subclass 790 visa (a "SHEV")[34].

In October 2017, the plaintiff was convicted after pleading guilty to the aggravated offence of people smuggling

on 27 June 2022. The first defendant, the Minister for Home Affairs, purportedly made a decision under s 65 of the Act to refuse the plaintiff's application because she was not satisfied of the visa criterion in cl 790.227 of Sch 2 of the Migration Regulations 1994 (Cth) that the grant of the SHEV was in the national interest ("the Decision"). The Minister's reasons reveal that the criterion in cl 790.227 was not met because in her view it was not in the national interest to grant a protection visa to a person convicted of a people smuggling offence.

Non-satisfaction of cl 790.227 was the sole basis for refusing the plaintiff's application. Satisfaction of the visa criteria other than that in cl 790.227 was not in issue. The defendants accepted that at the time of the Decision all of the criteria for the grant of the visa, apart from cl 790.227, were satisfied.

Minister's refusal to grant the SHEV

The second brief attached a draft Statement of Reasons for personally refusing the SHEV under s 65 relying on cl 790.227, which the Minister signed on 27 June 2022 without amendment. In that Statement of Reasons, the Minister set out her reasons for her decision that she was not satisfied that granting the SHEV to the plaintiff was in the national interest.

Relevantly, under the heading "Other factors taken into account in determining whether the grant of the visa would be in the national interest", the Minister stated that she did "not accept [the plaintiff's] submission that refusing to grant him a SHEV on the basis of clause 790.227 would be a refusal to grant the visa 'on character grounds', a matter dealt with by s 36(1C) of the Act". The reasons did not refer to PIC 4001 or s 501. The Minister stated that "a refusal to grant [the plaintiff] a SHEV on the basis of clause 790.227 is because of my assessment of other adverse impacts that granting a protection visa to a person who has been convicted of people smuggling would have on Australia's border protection regime, and the policy that underpins it".

Next, the Minister stated:-

"I regard protecting and safeguarding Australia's territorial and border integrity, which includes measures to combat people smuggling, to be matters that clearly go to the national interest. In my view, granting a protection visa to a person who has been convicted of people smuggling would send the wrong signal to people who may be contemplating engaging in similar conduct in the future, thereby potentially weakening Australia's border protection regime. It is not in the national interest for a person convicted of people smuggling to be seen to get the benefit of a protection visa." (emphasis added)

This statement is, in effect, a statement of general policy that it is not in the national interest for convicted people smugglers to get protection visas.

The Minister then set out what she described as an "additional reason" why she considered that granting the SHEV to the plaintiff would not be in the national interest:-

"I also consider it is in the national interest to maintain the confidence of the Australian community in the protection visa program. People smuggling can be seen to conflict with the values underlying the protection visa program since it involves taking advantage of, and exploiting, those seeking protection by smuggling them across borders. The grant of a protection visa to a non‑citizen who has been convicted of people smuggling may erode the community's confidence in the program." (emphasis added).

Issue:-

Was the Decision authorised by cl 790.227, properly construed?

Consideration:-

Regulations are to be construed according to the ordinary principles of statutory construction. The starting point for the ascertainment of the meaning of a provision is its text, while at the same time regard is to be had to its context and purpose. Of course, the statutory context of regulations includes the Act under which the regulations were made and are sustained. Context should be regarded at the first stage and not at some later stage and it should be regarded in its widest sense, including by reference to legislative history and extrinsic material. As Kiefel CJ, Nettle and Gordon JJ explained in SZTAL v Minister for Immigration and Border Protection[98]:-

"This is not to deny the importance of the natural and ordinary meaning of a word ... Considerations of context and purpose simply recognise that, understood in its statutory, historical or other context, some other meaning of a word may be suggested, and so too, if its ordinary meaning is not consistent with the statutory purpose, that meaning must be rejected."

The context of the words, consideration of the consequences of adopting a provision's literal meaning, the purpose of the statute and principles of construction may lead a court to adopt a construction that departs from the literal meaning of the words of a provision. One such principle is that legislation must be construed on the prima facie basis that its provisions are intended to give effect to harmonious goals. As expressed by Gageler J in SAS Trustee Corporation v Miles, "statutory text must be considered from the outset in context and attribution of meaning to the text in context must be guided so far as possible by statutory purpose on the understanding that a legislature ordinarily intends to pursue its purposes by coherent means". Where conflict appears to arise in construing an Act, "the conflict must be alleviated, so far as possible, by adjusting the meaning of the competing provisions to achieve that result which will best give effect to the purpose and language of those provisions while maintaining the unity of all the statutory provisions", and this "will often require the court 'to determine which is the leading provision and which the subordinate provision, and which must give way to the other'". Ultimately, the task in applying the accepted principles of statutory construction is to discern what Parliament is to be taken to have intended.

Clause 790.227 was inserted by Parliament into the Regulations

Unusually, cl 790.227 was not made by the Governor‑General in the exercise of the regulation-making power under s 504 of the Act. Instead, it was inserted by Parliament into the existing Regulations by the Migration and Maritime Powers Legislation Amendment (Resolving the Asylum Legacy Caseload) Act 2014 (Cth), which also amended the Act to introduce the SHEV as a new class of protection visa[104]. Clause 790.227 was inserted into the Regulations at the same time as all of the other criteria in the Regulations for the new SHEV class, including cl 790.226 which prescribed PIC 4001 as a criterion for the visa.

That is, Parliament did not create a statutory visa criterion by inserting cl 790.227 into the Act – for example, by amending s 36. Rather, Parliament made cl 790.227 in the form of an amendment to the Regulations, which were made by the Governor-General under s 504 of the Act. In doing so, Parliament made it clear that the Governor‑General was able to amend or repeal the Regulations as amended[106]. When the Regulations as amended were registered under the then Legislative Instruments Act 2003 (Cth), s 5(3) of that Act provided that, by virtue of the registration, the instrument was taken to be a "legislative instrument", defined as an instrument of a legislative character "that is or was made in the exercise of a power delegated by the Parliament"[107]. In short, cl 790.227 cannot be construed as if it were a provision of the Act, because it is not and never has been part of the Act, and the amending Act which inserted it into the Regulations did not express such an intention. It is a clause of the Regulations.

The subordinate status of cl 790.227 to the Act does not mean the clause is less binding in law than a statutory provision. However, it may, and here does, indicate that in reconciling provisions that otherwise present issues of inconsistency or incoherency, Parliament intended the clause to give way more readily or be adjusted if necessary to ensure a harmonious interpretation.

Proper construction of cl 790.227

The construction put forward by the plaintiff that should be accepted is that cl 790.227 does not operate to permit the Minister or delegate to reconsider or revisit, under the criterion of "the national interest", those matters that have already been considered as part of the decision-making process under s 65 (some of which are committed to the Minister personally) and to treat those matters as sufficient to form the opinion that the Minister or delegate is not satisfied that the grant of the visa is in the national interest.

That is not to say that cl 790.227 has no work to do. The concept of "the national interest" as used in the Act is undoubtedly broad and the possible considerations it may encompass cannot be catalogued. In this context, as cl 790.227 is a general visa criterion to be satisfied for all visa decisions for that class, "the national interest" must be informed in part by consideration of the nature of visas of that class, specifically, protection visas. The satisfaction of the criteria that are set out in the Act for protection visas, in particular the satisfaction of one of the criteria in s 36(2), is the primary basis on which Parliament expects that the Minister or delegate will be satisfied that the grant of the visa is in the national interest. That said, of course there might be other considerations that weigh against the general expectation in cl 790.227 that the grant of protection visas to persons to whom Australia owes protection obligations is in the national interest. For example, such a scenario might be where Australia is at war with the country from which the applicant seeks refuge. It is for this reason that, although the expression of cl 790.227 is in positive terms – as a criterion for grant of a visa, not for refusal of a visa – the parties were right to describe it in its negative sense because it is not a criterion that sits independently of all the others; the premise that it is in the national interest to grant a visa when a person is owed protection obligations and meets the other criteria can only be displaced by other national interest matters.

Again, put in different terms, it can be accepted, as the defendants submitted, that cl 790.227 is a cumulative requirement for the grant of a SHEV operating in addition to the other visa criteria and powers – cl 790.227 provides an additional basis to refuse the visa if the Minister considers, for some other reason, that the grant of the visa is not in the national interest. But that reason must be "another" reason. Determining whether that is the case will depend on an evaluation of the Minister or delegate's reasons.

As has been said, cl 790.227 was not intended to be a trump card for the Minister to use to refuse the visa under s 65 without needing to consider, or be constrained by, any of the other criteria and powers relevant to the decision. Unlike s 501 of the Act, cl 790.227 is not a special visa refusal power conferred by the Act. It is a positive visa criterion in the Regulations to be satisfied for all grants of a protection visa by the Minister and delegates alike. It cannot be treated as if it were a personal dispensing power. The Decision should be quashed.

Orders:-

In the circumstances, a writ of certiorari should issue quashing the decision of the first defendant made on 27 June 2022 to refuse to grant the plaintiff a SHEV and a writ of mandamus should issue commanding the first defendant to determine the plaintiff's visa application according to law within 14 days of the date of this order. The Minister has conceded that, at the date of the Decision, the plaintiff met all of the criteria for the SHEV other than cl 790.227. These reasons explain the proper construction and application of cl 790.227. The Court's power to grant declaratory relief is limited by the scope of the issues in the proceeding and a declaration as to the position conceded by the Minister is unnecessary.

The defendants should pay the plaintiff's costs.

Added a post 

Vanderstock v Victoria [2023] HCA 30 (18 October 2023)

Introduction: In a landmark decision, the High Court of Australia recently rendered judgment on Vanderstock v Victoria [2023] HCA 30, a case challenging the validity of Victoria's Zero and Low Emission Vehicle Distance-based Charge Act 2021. This case delved into the intricate interplay between sections 51(ii) and 90 of the Australian Constitution, shedding light on the taxation powers of both the Commonwealth and State governments. The Court's ruling, which declared a section of the Victorian Act invalid, has significant implications for fiscal federalism and the balance of power between states and the federal government in Australia. We explore the key issues, the ruling's implications, and the lessons learned from this precedent-setting case.


Facts:

The case in question is Vanderstock v Victoria [2023] HCA 30, heard by the High Court of Australia. The plaintiffs, Christopher Vanderstock and another party, challenged the validity of section 7(1) of the Zero and Low Emission Vehicle Distance-based Charge Act 2021 (Vic) ("ZLEV Charge Act"). This provision purports to obligate the registered operator of a zero or low emissions vehicle ("ZLEV") to pay a charge for using the ZLEV on "specified roads", which include all roads in Victoria and elsewhere in Australia over which the public has a right to pass. The charge is determined annually at a prescribed rate for each kilometre travelled by the ZLEV on specified roads in a financial year. The plaintiffs argued that this provision is invalid because it imposes a duty of excise within the meaning of section 90 of the Constitution.

The defendant in this case was the State of Victoria, represented by R J Orr KC, Solicitor-General for the State of Victoria, with S Zeleznikow and M R Salinger. Various Attorneys-General from different states intervened in support of Victoria. The Attorney-General of the Commonwealth intervened in support of the plaintiffs.

The question for consideration was whether section 7(1) of the ZLEV Charge Act invalidly imposes a duty of excise within the meaning of s 90 of the Constitution. If so, this would mean that only the Commonwealth Parliament could impose such a charge.

In its judgment, delivered on 18 October 2023, the Court held that section 7(1) does impose a duty of excise and therefore is invalid. The Court ordered that the defendant should pay the costs of proceedings.

Issues:

The issues in this case revolve around the interpretation and application of sections 51(ii) and 90 of the Australian Constitution. These sections concern the taxation powers of the Commonwealth and State governments, respectively.

The first issue is whether a concurrent power to tax (s 51(ii)) equates to a limitation (s 90). The justices argued that these two provisions should not be conflated or interpreted to expand the power in s 51(ii) or the limitation in s 90.

The second issue relates to whether State taxation powers are concurrent with, and independent of, that of the Commonwealth. This question arises from different interpretations of Professor Zines' analysis on the scope of the taxation power in s 51(ii).

The third issue involves assessing whether what is proposed as a "duty of excise" - any tax on goods - alters and affects the structural, political and constitutional balance between State and federal governments.

The fourth issue is whether a tax with any assumed effect on demand for goods is beyond the legislative power of the States.

Finally, there is an issue concerning constitutional facts and their relevance in determining constitutional validity. The argument here revolves around whether it's appropriate for constitutional validity to be decided based on potential economic consequences, particularly without evidence.

All these issues are relevant because they involve significant interpretations of key provisions in the Australian Constitution that shape fiscal relations between different levels of government in Australia. Decisions made on these issues could have significant implications for state autonomy, fiscal federalism, and Australia's broader constitutional framework.

Main Issue: The main issue in the case of Vanderstock v Victoria [2023] HCA 30 was whether section 7(1) of the Zero and Low Emission Vehicle Distance-based Charge Act 2021 (Vic) ("ZLEV Charge Act") is invalid as it imposes a duty of excise within the meaning of section 90 of the Constitution.

Rule: Section 90 of the Australian Constitution gives exclusive power to the Commonwealth Parliament to impose duties of customs and excise. It restricts states from levying taxes on goods that could distort interstate trade, commerce, and market competition.

Application: The High Court examined whether the charge imposed by the Victorian legislation fell within the definition of an excise. The ZLEV Charge Act obliges registered operators of zero or low emissions vehicles ("ZLEV") to pay a charge for their use on specified roads, which includes all roads in Victoria and elsewhere in Australia where public access is granted. This charge is determined annually based on each kilometre travelled by the ZLEV on specified roads in a financial year, making it a debt payable by the registered operator to Victoria.

The court applied past judgments, particularly Capital Duplicators Pty Ltd v Australian Capital Territory [No 2] (1993) 178 CLR 561 and Ha v New South Wales (1997) 189 CLR 465, which held that duties of excise within s 90 are inland taxes on goods. The court had to consider whether a tax imposed at the stage of consumption could be considered an excise, something not decided in those previous cases. The court found that such a tax can indeed be an excise, contradicting Dickenson's Arcade Pty Ltd v Tasmania (1974) 130 CLR 177 and overruling its decision.

Conclusion: The court concluded that section 7(1) of the ZLEV Charge Act is invalid because it imposes a duty of excise within the meaning of section 90 of the Constitution. This decision reinforces the principle that the power to impose duties of customs and excise is exclusive to the Commonwealth Parliament.

Reasoning for Judgement: The court's reasoning was based on the definitions and characterizations of "excise" in previous cases and its application to this particular charge. They noted that a tax on goods imposed at the stage of consumption can indeed be an excise, which led them to overrule a previous decision (Dickenson's Arcade).

Take Home Lesson: This case reaffirms that states cannot enact laws that effectively impose taxes on goods, as this power is reserved exclusively for the Commonwealth Parliament under section 90 of the Constitution. It also broadens the definition of 'excise' to include charges imposed at the point of consumption, adding another layer of complexity to constitutional tax law. This means lawmakers must be careful when drafting legislation involving charges or taxes on goods, even when these are not traditional production or sales taxes.

Added a post 

QYFM v Minister for Immigration, Citizenship, Migrant Services and Multicultural Affairs [2023] HCA 15 (17 May 2023)

Intro:

This is an appeal from a decision of the Full Court of the Federal Court of Australia.

Facts:-

The matter before the Full Court was an appeal by the present appellant, who is a citizen of Burkina Faso. The respondents to the appeal were the present respondents, the Minister for Immigration, Citizenship, Migrant Services and Multicultural Affairs ("the Minister") and the Administrative Appeals Tribunal ("the AAT"). The appeal was from a decision of a single judge of the Federal Court dismissing an application by the appellant for judicial review of a decision of the AAT. The decision of the AAT had affirmed a decision of a delegate of the Minister not to revoke the cancellation of the appellant's visa.

Under the Migration Act 1958 (Cth) ("the Act"), the Minister is obliged to cancel a person's visa if the Minister is satisfied that the person does not pass the "character test" and if the person is serving a full-time sentence of imprisonment. The Act provides that a person does not pass the "character test" if the person has "a substantial criminal record", which exists if the person has been sentenced to a term of imprisonment of 12 months or more. Upon notice to the person and upon representations being made by the person, the Minister has a discretion to revoke the cancellation decision if, relevantly, satisfied that there is "another reason" (apart from the person passing the character test) why that decision should be revoked. A decision of a delegate of the Minister made in the exercise of that discretion is reviewable on its merits by the AAT.

Following a trial on indictment in the County Court of Victoria in 2013, the appellant was convicted of a drug importation offence under the Criminal Code (Cth). He was sentenced to a term of imprisonment of ten years with a non-parole period of seven years. He appealed against his conviction to the Victorian Court of Appeal, which in 2014 dismissed the appeal.

In 2017, while the appellant was serving the sentence of imprisonment, a delegate of the Minister made the decision to cancel his visa on the basis that he did not pass the "character test" by reason of the sentence of imprisonment. In 2019, another delegate of the Minister decided not to revoke that cancellation decision. The decision not to revoke the cancellation of the appellant's visa was affirmed by the AAT in 2020. The appellant applied for judicial review of that decision of the AAT and was unsuccessful before the primary judge, leading to the appeal to the Full Court.

The appellant was unrepresented before the primary judge and at the time of filing the original notice of appeal from the decision of the primary judge to the Full Court. The appellant subsequently obtained legal representation, resulting in a written application to the Full Court for leave to amend the notice of appeal to raise grounds which had not been advanced before the primary judge. The proposed grounds were to the effect that the decision of the AAT was legally unreasonable and that certain findings of the AAT were not supported by probative evidence. The application for leave to amend was listed for hearing at the time scheduled for hearing of the appeal.

The appeal was scheduled to be heard on the morning of 17 August 2021 before a Full Court constituted by McKerracher, Griffiths and Bromwich JJ. It is a matter of public record that, from 2012 until his appointment to the Federal Court in 2016, Bromwich J held the office of Director of Public Prosecutions under the Director of Public Prosecutions Act 1983 (Cth).

Minutes before the commencement of the hearing on that day, the associate to Bromwich J sent an email to the legal representatives of the parties. The email stated that his Honour had asked the associate to advise the parties that he had appeared for "the Crown" in the appellant's unsuccessful conviction appeal in the Victorian Court of Appeal in 2014. The fact that his Honour had appeared as senior counsel for the respondent in that appeal was apparent from the record of the decision of the Victorian Court of Appeal, which was included in the appeal book for the Full Court proceedings. The email said that his Honour did not consider that circumstance to give rise to an apprehension of bias as the appeal "related to a pure legal question" but nonetheless wished to draw it to the attention of the parties "in order that any application for his Honour to recuse himself" could be made.

At the commencement of the hearing of the appeal before the Full Court, counsel for the appellant announced that he had instructions to apply for Bromwich J "to recuse himself". Counsel for the appellant proceeded to make that application orally, relying solely on the circumstances disclosed in the email. At the conclusion of the oral submissions of counsel for the appellant there was a short adjournment, following which the Full Court reconvened.

Upon the Full Court reconvening, McKerracher J invited Bromwich J to "deal with the application". Bromwich J explained that he declined to recuse himself from sitting on the appeal for reasons he proceeded to elaborate. McKerracher J then invited counsel for the appellant to continue and the hearing resumed. At the conclusion of the hearing, the Full Court reserved its decision.

The decision of the Full Court was delivered on 15 September 2021. By that decision, the Full Court granted the appellant leave to rely on the ground that the decision of the AAT was legally unreasonable, refused the appellant leave to rely on the proposed ground that findings of the AAT were not supported by probative evidence, and dismissed the appeal.

Joint reasons for judgment then published by McKerracher and Griffiths JJ comprehensively addressed the merits of the application for leave to amend the notice of appeal and the appeal. Those reasons said nothing about the application which had been made orally at the hearing.

In separate reasons for judgment, Bromwich J agreed with McKerracher and Griffiths JJ as to the merits and went on to recapitulate the reasons he had given during the hearing for considering that the circumstances outlined in the email from his associate did not give rise to an apprehension of bias. Those reasons, in essence, were that: as Director of Public Prosecutions his practice was to appear only in appeals which raised issues of principle; the appellant's appeal against conviction to the Victorian Court of Appeal was an appeal of that character, turning wholly on a legal question as to the admissibility of evidence; by virtue of his appearance in that appeal, he had acquired no knowledge of the criminal history of the appellant beyond that which was apparent to all members of the Full Court from the record of the decision of the Victorian Court of Appeal contained in the appeal book for the Full Court proceedings; the fact of the conviction was not in issue in the appeal before the Full Court, it being common ground that the appellant failed the "character test"; and the contents of the decision of the Victorian Court of Appeal did not feature in the appeal before the Full Court in any way, the record of that decision having been included in the appeal book only because it had been part of the material which had been before the AAT.

His Honour also made clear what was in any event to be inferred from the timing of his associate's email: that he had only noticed that he had appeared in the conviction appeal during the course of his final preparation on the morning of the hearing before the Full Court on 17 August 2021. His Honour acted prudently in accordance with standard judicial practice by promptly notifying the parties of circumstances properly recognised by him to have the potential to be seen to give rise to an apprehension of bias.

Issue:-

Whether challenged judge's appearance as senior counsel for the Crown in the appellant's appeal against his convicton for drug importation offense was sufficient to have given rise to reasonable apprehension of bias on the part of challenged judge?

Consideration:-

The Full Court should have decided the objection

The application made to the Full Court by counsel for the appellant at the commencement of the hearing, although framed as an application that Bromwich J "recuse himself", was in substance an objection to the Full Court as then constituted hearing and determining the appeal. Was the application appropriately left by McKerracher and Griffiths JJ to be considered and determined by Bromwich J alone? Or should it have been considered and determined by the Full Court constituted by all three of them?
...

The question arising in the circumstances of the present case falls to be resolved at the level of principle within the framework established in Ebner v Official Trustee in Bankruptcy. Foundational to that framework are two propositions. One is that impartiality is an indispensable aspect of the exercise of judicial power. The other is that "[b]ias, whether actual or apprehended, connotes the absence of impartiality". Leaving to one side exceptional circumstances of waiver or necessity, an actuality or apprehension of bias is accordingly inherently jurisdictional in that it negates judicial power.

Once the jurisdictional significance of bias is appreciated, it becomes apparent that responsibility for ensuring an absence of bias – whether actual or apprehended – lies with a court as an institution and not merely with a member of that court whose impartiality might be called into question. The duty, indeed the "first duty", of any court is to be satisfied of its own jurisdiction. The upshot of that duty, as elaborated by Gibbs J in The Queen v Federal Court of Australia; Ex parte WA National Football League, is that:-

"When the question is raised before a court of limited jurisdiction whether a condition of its jurisdiction has been satisfied, that court is not obliged immediately to refrain from proceeding further. It can and should decide whether the condition is satisfied and whether it has jurisdiction to proceed, but its decision is not conclusive."

Thus, an objection to a multi-member court as constituted hearing and determining a matter based on an allegation of bias on the part of one or more of its members (including an objection brought by way of an application for recusal or disqualification) raises a question of jurisdictional fact which that court can and must determine for itself in order to be satisfied of its own jurisdiction. The determination of that question of jurisdictional fact is not antecedent to the performance of the curial function, but part of that function. The determination ought to be reflected in a curial order which embodies the court's formal resolution of the objection, subject to applicable procedures for appeal or review for jurisdictional error.

In the case of the Full Court of the Federal Court, precisely who constitutes "the court" for the purposes of hearing and determining an appeal is spelt out in the Federal Court of Australia Act 1976 (Cth). Subject to presently immaterial exceptions, the appellate jurisdiction of the Federal Court is required to be exercised by a Full Court consisting of three or more judges sitting together. The making of arrangements about which judges are to constitute the Full Court in a particular matter or class of matters lies within the responsibility of the Chief Justice. The process by which the Chief Justice makes arrangements can be expected to be tailored to minimise the risk of any issue of bias arising on the hearing of a matter, including by allowing a legitimate concern about bias on the part of a judge to be raised and addressed administratively as an aspect of that process. But once a Full Court consisting of three or more judges is constituted and is seized of the hearing of an appeal, responsibility for the discharge of the judicial power involved in hearing and determining the appeal devolves to those three or more judges acting institutionally as the Full Court. The institutional responsibility of the Full Court as so constituted includes the consideration and determination of an objection to its jurisdiction. The Full Court as so constituted has a duty to hear and determine the appeal unless the court as so constituted determines that the objection to jurisdiction is well-founded.

The view that an allegation of bias on the part of a member of an appellate or collegiate court can and should be considered and determined by the court as distinct from the impugned member alone was cogently developed by Sir Anthony Mason, writing soon after Kartinyeri. Sir Anthony concluded that the court has an institutional responsibility to ensure that it is constituted "in accordance with the provisions of the law governing the judicial process, the exercise of judicial power and natural justice". He pointed out that that institutional responsibility of the court as constituted was entirely congruent with what then was, and remains, the common practice in Australian courts according to which judges at first instance hear and determine allegations of bias raised against themselves. The rationale for that practice he explained to be that "[a]t first instance, the judge who is the target of the objection determines the objection because [that judge] constitutes the court".

To require an allegation of bias on the part of a member of a multi-member court to be considered and determined by the court as constituted should not be thought to place an undue strain on judicial collegiality. In the context of explaining the appropriateness of appellate review of non-recusal decisions, it was emphasised in Livesey that the determination of questions of apprehended bias can involve evaluations of degree on which reasonable minds might differ and that a conclusion of apprehended bias on the part of an individual judge implies no criticism of that judge. As for sensitivities between judges at different levels of the judicial hierarchy, so for sensitivities between judges within a multi-member court.

The application that Bromwich J "recuse himself" ought therefore to have been considered and determined by the Full Court.

Apprehended bias should have been found

The basis on which apprehended bias should have been found remains to be explained.

The criterion for the determination of an apprehension of bias on the part of a judge was definitively stated in Ebner by reference to previous authority and has often been repeated. The criterion is whether "a fair-minded lay observer might reasonably apprehend that the judge might not bring an impartial mind to the resolution of the question the judge is required to decide". The "double might" serves to emphasise that the criterion is concerned with "possibility (real and not remote), not probability".

Application of the criterion was identified in Ebner, and has been reiterated, logically to entail: (1) identification of the factor which it is said might lead a judge to resolve the question other than on its legal and factual merits; (2) articulation of the logical connection between that factor and the apprehended deviation from deciding that question on its merits; and (3) assessment of the reasonableness of that apprehension from the perspective of a fair-minded lay observer.

...

In the present case, close adherence to the logic of the Ebner analysis has greater utility.

Although the appellant placed some emphasis on the fact of Bromwich J having held the statutory office of Director of Public Prosecutions at the time of the appellant's conviction appeal, and by inference at the time of his indictment, trial and conviction, the appellant failed to articulate how that factor alone might logically be said to lead to an apprehension that his Honour might not resolve the questions in the appeal to the Full Court on their legal and factual merits. None is apparent.

The appellant is on firmer ground in relying on the fact that Bromwich J had appeared as counsel against him in his conviction appeal. It will be recalled that becoming aware of that fact had prompted Bromwich J correctly to cause the parties to be notified of the potential for an apprehension of bias to have arisen.

In articulating the logical connection between that factor and an apprehension that Bromwich J might not bring an impartial mind to the resolution of the legal questions before the Full Court on their merits, the appellant relied on the observation of Gageler J in Isbester that "a person who has been the adversary of another person in the same or related proceedings can ordinarily be expected to have developed in that role a frame of mind which is incompatible with the exercise of that degree of neutrality required dispassionately to weigh legal, factual and policy considerations relevant to the making of a decision which has the potential adversely to affect interests of that other person".

His Honour's appearance as counsel against the appellant in his earlier conviction appeal was sufficient to give rise to a reasonable apprehension on the part of a fair-minded lay observer of the possibility that his Honour had formed and retained an attitude to the appellant incompatible with the degree of neutrality required dispassionately to resolve issues in a subsequent proceeding to which the appellant was a party. The circumstance that the conviction led to the cancellation of the appellant's visa so as to be causally related to the subject-matter of the appeal concerning the non-revocation of the cancellation decision reinforced the reasonableness of that apprehension in the circumstances of the case. The facts that his Honour had been concerned as counsel only to argue a question of law and had acquired no knowledge of the criminal history of the appellant not apparent from the record of the decision of the Victorian Court of Appeal were not to the point. Neither that nor any other of the considerations referred to by his Honour was sufficient to allay the apprehension of bias which reasonably arose.

The effect of apprehended bias

The Solicitor-General of the Commonwealth, who appeared for the Minister, conceded that it followed from a conclusion of apprehended bias on the part of Bromwich J that the Full Court was deprived of jurisdiction to hear and determine the appeal even though the decision was in fact unanimous. Despite being prepared to concede that bias on the part of one out of three members of a multi-member court was sufficient to deprive the court of jurisdiction, the Solicitor-General sought to reserve to a future case whether bias on the part of one out of, say, five or seven members of a multi-member court would have that effect. The attempted reservation indicates that something needs to be said about the basis on which the concession is correct.

Once it is accepted that absence of bias is inherent in the exercise of judicial power and that the jurisdiction of a multi-member court is to be exercised by all of the judges who constitute the court for the hearing and determination of a matter, it becomes apparent that bias on the part of any one of those judges deprives the court as so constituted of jurisdiction to proceed with the hearing and determination of that matter. Where bias on the part of an individual judge is established, that is the end of the jurisdictional inquiry. No numerical exercise is involved. It is not a question of counting apples in a barrel[84]. Nor is it to the point to inquire into whether the outcome of the exercise of jurisdiction by the court as so constituted would or could have been different if the judge was not biased or if the biased judge did not participate.

That jurisdictional consequence of bias on the part of any of its members has an important practical dimension for a multi-member court. Each member of the court has an individual duty to give effect to his or her own true view of the facts and applicable law. In the discharge of that duty, however, members of the court can properly be expected to confer together in private in order to obtain the benefit of each other's views and to agree where they can. For the public to be able to have confidence in the outcome of such a closed deliberative process, the public must be confident that each participant in the process is free from bias. The process and the outcome would be tainted were a biased judge "in the room".

Conclusion:-

The appeal should be allowed. The decision of the Full Court should be set aside. The matter should be remitted to the Federal Court to be heard and determined by a differently constituted Full Court. The Minister should pay the appellant's costs of the appeal and of the hearing to date of the appeal to the Full Court.

Added a post 

McEwan v Clark & Ors [2023] QCA 120 (2 June 2023)

Intro:-

This is an appeal from the Supreme Court of Queensland at Brisbane.

Facts:-

In 2019, the appellant was charged with 30 offences, alleged to have been committed under the Criminal Code (Cth) and the Criminal Code (Qld), which include allegations that she attempted to dishonestly obtain a financial advantage from the Australian Taxation Office (ATO), dishonestly influenced a Commonwealth official, and engaged in forgery and uttering.

The appellant is currently being prosecuted in the Magistrates Court with respect to those offences.

On 12 December 2022, Magistrate Saggers made two relevant orders:

(a) the first was that there be a disclosure hearing in the Magistrates Court on 9 March 2023; and
(b) the second was that the committal hearing on the charges be set down for five days commencing on 8 May 2023.

On 13 December 2022, the appellant filed an Originating Application in the Supreme Court seeking relief which can be characterised in the following way:

(a) Order 1: that the committal hearing in the Magistrates Court be delisted;

(b) Order 2: the ATO proceedings and the committal proceedings in the Magistrates Court be adjourned until a final determination is made on the appellant’s application for a permanent stay of the prosecutions in proceeding No. 9181/2022;

(c) Order 3: all outstanding disclosure listed in the Disclosure Request served on the CDPP, ATO, and the Queensland Director of Public Prosecutions (DPP) under the directions of Judge Moynihan KC dated 4 November 2022, and again served on 21 November 2022 and 9 December 2022, be provided within seven days by 20 December 2022;[3]

(d) Order 4: all outstanding disclosure listed in the Disclosure Request served on the QPS dated 9 December, and again 13 December 2022, be provided directly to the appellant within seven days by 20 December 2022;

(e) various other orders relating to disclosure in the Magistrates Court; and

(f) Order 6: the ATO proceedings be placed on the Supreme Court Supervised Case List.

On 21 December 2022, the Originating Application came before the learned primary judge.

During oral submissions before the learned primary judge, the appellant sought to join Magistrate Saggers and Magistrate Merrin. That was based on assertions of “abuse and fraud and being shut down in the Magistrates Court”.[4]

The learned primary judge:-

(a) dismissed the application for the committal proceeding to be delisted;

(b) dismissed the applications for disclosure;

(c) ordered that the appellant not file any further application in the Supreme Court, in relation to the committal proceeding presently being heard in the Magistrates Court Brisbane, without the leave of that Court; and

(d) ordered the appellant pay the Eighth and Ninth Respondents’ costs of the application.

The primary judge's approach

His Honour then expressed his findings in this way:-

“Having considered the material, I am satisfied it is not appropriate to make any of the orders sought by the applicant.

First, this Court does not use any supervisory jurisdiction to interfere with committal proceedings being conducted in another Court. That is to fragment proceedings, and it is contrary to the interests of justice.

Second, the matter of disclosure is specifically to be the subject of a determination by the Magistrates Court, prior to the hearing of that committal proceeding. It would equally be to fragment the process for this Court to interfere in that process.

In those circumstances, the application for the committal proceeding to be delisted and/or adjourned and the applications for disclosure are dismissed.”

Issue:-

Did primary judge deny Appellant natural justice and procedural fairness by not allowing the appellant a hearing for the Court to determine if there are exceptional circumstances warranting the committal proceedings be fragmented?

Consideration:-

For the reasons which follow, the appellant’s contentions must be dismissed.

First, it is true to say that the essential elements of procedural fairness traditionally encompass two requirements, fairness and detachment, which relevantly involve “the absence of the actuality or the appearance of disqualifying bias and the according of an appropriate opportunity of being heard”.

Having read the transcript of the hearing below, it cannot be concluded that there was an absence of fairness or detachment on the part of the learned primary judge. The appellant’s written submissions and affidavits were considered, she was able to make such oral submissions as she wished, and the primary judge’s engagement with her did not suggest any lack of fairness or lack of detachment. The appellant was afforded a fair hearing.

Secondly, what the appellant sought below was to have the Supreme Court interfere in the proceedings of the Magistrates Court. More particularly, the substantive proceedings below are of essentially an administrative kind, namely the conduct of a committal hearing into charges on an indictment. The appellant wrongly characterises that as an application to have the committal proceedings “uplifted to the Supreme Court for supervision”. No such application was made. The application was to have the committal proceedings delisted and adjourned pending the determination of a permanent stay application.

An application for a permanent stay had been made in the Magistrates Court, but refused by Magistrate Merrin on 1 August 2022. Whilst the stay application in the Supreme Court has previously been set for hearing, it has been adjourned pending the determination of an appeal concerning the issuing of subpoenas in that application.

Thirdly, in so far as the application below sought orders concerning disclosure relevant to the committal, both the Magistrates Court and the District Court had made relevant orders, and most importantly, a hearing had already been set in the Magistrates Court to deal with disclosure issues. The appellant was thus seeking that the Supreme Court (and now, this Court) interfere in another court’s conduct concerning matters of practice and procedure.

It is apparent that orders have been made as to disclosure in the Magistrates Court, and a hearing in that respect was set for 9 March 2023, though then adjourned. While the Magistrates Court is seized of those issues, it would be wrong of this Court to intervene absent some clear evidence warranting such a rare course of action. There is none beyond mere assertion by the appellant.

Fourthly, in so far as the orders sought below, and in this Court, concern the delisting of the committal proceeding, a permanent stay has been sought in the Supreme Court. As noted above, that application had previously been set for hearing, but adjourned pending the determination of an appeal concerning the issuing of subpoenas in that application. The orders sought from this Court would unjustifiably interfere with the due hearing of that relief.

Fifthly, the orders sought below, and in this Court, seek to overturn interlocutory orders made by the Magistrates Court. Section 222 of the Justices Act 1886 (Qld), which is the statutory source of the right to appeal orders of the Magistrates Court, does not apply to interlocutory orders.

Sixthly, beyond mere assertion by the appellant, there was no evidence of anything approaching the sort of exceptional circumstances that might justify the Supreme Court interfering in such a way that it would fragment criminal proceedings in another court. It is well established that courts should not interfere with or fracture the course of a criminal proceeding except “where the need for such interference is absolutely plain and manifestly required”. In Obeid, Gageler J adopted what was said by Kirby J in Frugtniet:

“This Court has more than once ... emphasised how rare it is to make orders which would have the effect of interfering in the conduct of a criminal trial. No case has been brought to my notice where the Court has made a stay order equivalent to the one sought on this summons. Although I do not doubt that, in a proper case, the Court would have the jurisdiction to make such an order to protect the utility of its process, it would be truly exceptional for it to do so. The Court expressed its attitude of restraint most recently in its decision in R v Elliott. There are many earlier such cases. They evidence the strong disposition of appellate courts in Australia – and especially of this Court – not to interfere in the conduct of criminal trials except in the clearest of cases where the need for such interference is absolutely plain and manifestly required. Analogous principles apply ... to the provision of a stay to prevent the commencement of a trial so as to permit a constitutional point to be argued. That point will not be lost to the plaintiff. If need be, at a later stage, it can be raised again.”

Seventhly, the appellant’s approach wrongly assumes that the Supreme Court’s supervisory jurisdiction over inferior courts is at large or unconstrained. It is not. It extends to cases where the decision of the inferior court is affected by jurisdictional error in the sense discussed in Kirk v Industrial Court (NSW)[16] and Craig v South Australia.No such jurisdictional error has been identified. A potentially erroneous decision of an inferior court acting within jurisdiction will not ordinarily be amenable to the Court’s supervisory jurisdiction.

Eighthly, the decision of the learned primary judge was made in the exercise of his Honour’s discretion. In order to succeed, the appellant would have to show error of the kind referred to in House v The King; that is to say that the learned primary judge acted on wrong principle, allowed extraneous or irrelevant matters to affect him, mistook the facts, failed to take a material consideration into account, or that the decision was plainly unreasonable or unjust, thereby bespeaking error. No such error has been demonstrated.

Conclusion:-

The appeal is dimissed.

Added a post 

El Khouri v Gemaveld Pty Ltd [2023] NSWCA 78 (26 April 2023)

Intro:

The applicants invoke this Court’s supervisory jurisdiction seeking to quash development consent granted by the Land and Environment Court over the first respondent’s neighbouring land because they have established on evidence which was not before that Court that the development exceeds a height restriction applicable to the land. Following a remitter at which facts were found, the principal and dispositive issue is one of statutory construction, namely, whether compliance with the height restriction is a jurisdictional fact which can be reviewed by this Court on the basis of evidence not before the Land and Environment Court. For the reasons which follow, I have concluded that it is not a jurisdictional fact.

Facts:-

Gemaveld applied for approval of development comprising demolition works and the construction of a multi-level dwelling house, swimming pool, front fence, landscaping and site works. The plans approved for Gemaveld’s land are for a residence with four levels, numbered 0 (which is street level), -1, -2 and -3.

The land is zoned residential. The Kogarah Local Environmental Plan 2012 (Kogarah LEP) imposes a height restriction of 9m over all three blocks.

The Council refused development consent, and Gemaveld commenced proceedings in Class 1 of the jurisdiction of the Land and Environment Court. One of the issues identified in the litigation was the height of the building proposed by Gemaveld, and it seems that Gemaveld obtained new plans showing the building envelope sitting wholly within a line drawn 9m from ground level, on both the northern and southern elevations of the proposed development.

Pursuant to s 34 of the Land and Environment Court Act 1979 (NSW), a conciliation conference was conducted and agreement was reached between Gemaveld and the Council. Section 34(3) provides as follows:
(3) If, either at or after a conciliation conference, agreement is reached between the parties or their representatives as to the terms of a decision in the proceedings that would be acceptable to the parties (being a decision that the Court could have made in the proper exercise of its functions), the Commissioner—

(a) must dispose of the proceedings in accordance with the decision, and

(b) must set out in writing the terms of the decision.

In April 2022, a Commissioner of the Land and Environment Court satisfied himself that the agreed decision was one that the Court could have made in the proper exercise of its functions and accordingly made orders granting consent: Gemaveld Pty Ltd v Georges River Council [2022] NSWLEC 1182. The Commissioner was explicitly satisfied that “[t]he proposed development complies with the height of building standard of 9m at cl 4.3 of the [Kogarah] LEP”: at [7(3)].

By summons filed in this Court on 6 July 2022, shortly before three months after the Commissioner’s decision, the applicants sought judicial review of the Commissioner’s decision. The fact that they did so within three months means that it is unnecessary to consider the effect of the privative clause in s 3.27 of the Environmental Planning and Assessment Act 1979 (NSW) or the time limitation in r 59.10(1) of the Uniform Civil Procedure Rules 2005 (NSW). The second and third grounds of the summons were based on a failure to notify and a failure to afford procedural fairness. Those grounds were abandoned after the Council served its evidence. The first and only ground which was pressed was that:-

"The Court had no power to make the decision to grant consent to the DA because it was not a decision that the Court could have made in the proper exercise of its functions, within the meaning of s 34(3) ... in that the height of the proposed development exceeded the relevant height control in the Kogarah [LEP] and there was no request to vary that standard as required by cl 4.6 of that LEP."

It will be clear from the state of the evidence summarised above that the proceedings raised the question of jurisdictional fact acutely. It was common ground in this Court that the 9m height control was exceeded by the proposed developments. But the position before the Commissioner was precisely the opposite. It was common ground before the Commissioner, and the entirety of the evidence pointed in one direction, that there was no exceedance of that height control.

Thus, it is clear that the Commissioner formed the opinion that the proposed building did not exceed the 9m height limit, and it is clear that he did so properly in a manner which is not susceptible to challenge based on the evidence before him. On the other hand, if the power to issue consent turns not on the opinion that the building did not exceed the height control, but on the fact that the building did not exceed the height control, then plainly that fact is not established.

Issue:-

Was the height of the proposed building a jurisdictional fact necessary to enliven the LEC's power undef Section 34(3) of the Land and Environment Court Act?

Consideration:-

The starting point is to construe the power whose exercise is impugned in this proceeding.

Section 34(3) of the Land and Environment Court Act is reproduced above. There is no issue that agreement was reached between Gemaveld and the Council so as to engage the subsection. The Commissioner was thereupon under an obligation to dispose of the proceedings in accordance with the agreed decision, subject only to the words in brackets: “being a decision that the Court could have made in the proper exercise of its functions”.

As Basten JA explained in Al Maha Pty Ltd v Huajun Investments Pty Ltd (2018) 233 LGERA 170; [2018] NSWCA 245 at [16]:

"If a decision of the Land and Environment Court could be set aside or declared invalid by this Court, it is not “a decision that the Court could have made in the proper exercise of its functions”. Nor would it make sense to read s 34(3) as implicitly authorising such a decision. If the development application lacked essential elements, the deficiencies could render a consent one which could not have been granted in the proper exercise of the Court’s functions."

Jurisdictional fact

When reference is made to a “jurisdictional fact”, the issue is whether a precondition to the exercise of statutory power has been satisfied. The High Court has described jurisdictional facts as “an essential condition of the jurisdiction of the magistrate” (Ridley v Whipp [1916] HCA 76; (1916) 22 CLR 381 at 385; [1916] HCA 76), “an essential preliminary or a condition precedent” to the jurisdiction (Parisienne Basket Shoes Pty Ltd v Whyte [1938] HCA 7; (1938) 59 CLR 369 at 385; [1938] HCA 7), “any event or fact or circumstance” which is “made a condition upon the occurrence or existence of which the jurisdiction of a court shall depend” (Parisienne Basket Shoes at 391), a “condition of jurisdiction” (R v Trade Practices Tribunal; Ex parte Tasmanian Breweries Pty Ltd [1970] HCA 8; (1970) 123 CLR 361 at 399; [1970] HCA 8), and “a preliminary question on the answer to which ... jurisdiction depends” (R v Judges of the Federal Court of Australia; Ex parte Pilkington ACI (Operations) Pty Ltd [1978] HCA 60; (1978) 142 CLR 113 at 125; [1978] HCA 60). Many of the more modern decisions have arisen in the context of challenges to decisions under planning and environmental laws, including Australian Heritage Commission v Mount Isa Mines Ltd (1997) 187 CLR 297; [1997] HCA 10, Timbarra Protection Coalition Inc v Ross Mining NL (1999) 46 NSWLR 55; [1999] NSWCA 8, Woolworths Ltd v Pallas Newco Pty Ltd (2004) 61 NSWLR 707; [2002] NSWCA 422 and Anvil Hill Project Watch Association Inc v Minister for the Environment and Water Resources (2008) 166 FCR 54; [2008] FCAFC 3. However, there is nothing peculiar to the principles governing jurisdictional facts in decisions made under planning and environmental laws. In Gedeon v Commissioner of the New South Wales Crime Commission (2008) 236 CLR 120; [2008] HCA 43 at [43]- [44], the High Court noted that the term could be used imprecisely, and gave the following description:-

"The expression “jurisdictional fact” was used somewhat loosely in the course of submissions. Generally the expression is used to identify a criterion the satisfaction of which enlivens the exercise of the statutory power or discretion in question. If the criterion be not satisfied then the decision purportedly made in exercise of the power or discretion will have been made without the necessary statutory authority required of the decision maker.

The concept appears from the following passage in the reasons of Latham CJ in R v Connell; Ex parte The Hetton Bellbird Collieries Ltd:-

“The subject matter with which the Industrial Authority deals is, inter alia, rates of remuneration. There is power to deal with this subject matter in respect of rates of remuneration which existed on the specified date only if the authority is satisfied that the rates in question are anomalous. Unless this condition is fulfilled, the authority cannot act – it is a condition of jurisdiction.”

One reason for difficulty is terminological. The precondition may be a question of fact. Alternatively it may be merely that the donee of the power be satisfied of or hold the opinion that the precondition is satisfied. There is a difficulty of nomenclature here, because the attainment of a state of satisfaction or the holding of an opinion is itself a question of fact as to the state of mind of the donee of the power.

Application to a consent granted by the Land and Environment Court following a conciliation conference

Significantly, the development consent challenged in the present appeal was granted by the Land and Environment Court constituted by a Commissioner exercising the power conferred by s 34(3) of the Land and Environment Court Act , rather than by the local council exercising the powers and functions under ss 4.15 and 4.16 of the Environmental Planning and Assessment Act, as had been the case in Ross v Lane. It was primarily on this basis that the applicants sought to distinguish the result reached in Ross v Lane.

However, the only issue for the Commissioner was whether the decision to which Gemaveld and the Council had agreed during the conciliation conference was one which could have been made by the Land and Environment Court in the proper exercise of its functions. That involves a notional inquiry. But it involves the same considerations, and the same powers, as if there had been a hearing by that Court in the exercise of Class 1 of its jurisdiction, and that in turn involves the same powers and functions of the consent authority, including ss 4.15 and 4.16. In order for the Commissioner to determine whether the decision to which the parties had reached agreement was one which “the Court could have made in the proper exercise of its functions” within the meaning of s 34(3), it was necessary to identify the powers and functions of the Court determining an appeal from the refusal of Gemaveld’s application under s 8.7 of the Environmental Planning and Assessment Act, which included at least by reason of s 8.14 “all the functions and discretions which the consent authority whose decision is the subject of the appeal had in respect of the matter the subject of the appeal”, and that picked up the obligations to have regard to the matters in s 4.15(1) and to exercise the power in s 4.16(1).

Thus on the reasoning upheld by this Court in Ross v Lane, compliance with cl 4.3 of the Kogarah LEP being a matter to which regard was required pursuant to s 4.15(1)(a) was not a jurisdictional fact.

Conclusion:-

For those reasons, there is no material difference for the purposes of jurisdictional facts, between a development consent granted “on the merits” whether by the consent authority or by the Land and Environment Court after hearing an appeal, and a development consent granted under s 34(3) following a successful conciliation conference. In both cases, environmental planning instruments are a mandatory consideration by reason of s 4.15(1)(a). In the case of a determination on the merits, that occurs at the time the power is exercised. In the case of a successful conciliation conference, that occurs when the Court forms the opinion required by s 34(3) that the agreed decision is one which the Court could have made in the proper exercise of its functions. But in neither case is compliance with the environmental planning instrument a jurisdictional prerequisite to the power to grant consent.

It is plain that the Commissioner had regard to cl 4.3 of the Kogarah LEP. He expressly formed the only view that was open to him on the evidence which was available to him, namely, that there was compliance with the height requirement. That decision is not vitiated merely because the applicants can establish, by evidence not made available to the Commissioner, that there was not compliance with that clause.

Accordingly, the amended summons filed on 2 August 2022 must be dismissed.

 

Added a post 

Parry v Secretary, Department of Health [2023] HCA 9 (16 March 2023)

Intro:-

By way of an application for a constitutional writ filed on 20 December 2022, the plaintiffs seek a writ of certiorari quashing the respondent Secretary's decision (or decisions) under the Therapeutic Goods Act 1989 (Cth) ("the TG Act") in relation to use of the SPIKEVAX (elasomeran) COVID-19 vaccine in children six months of age and older. The plaintiffs alternatively seek a declaration that the Secretary's decision was made unlawfully. A writ of mandamus was originally sought but that part of the application is no longer pressed. The plaintiffs request expedition.

Facts:-

There is some ambiguity as to whether the plaintiffs seek to challenge the Secretary's provisional determination made on 9 November 2021 with respect to the SPIKEVAX vaccine, being a decision made under s 22D of the TG Act; the Secretary's decision made on 19 July 2022 to provisionally register the SPIKEVAX vaccine for use in children, being a decision made under s 25(3)(a) of the TG Act; or both. For present purposes, nothing turns on that ambiguity and it is sufficient to proceed on the basis that the plaintiffs are challenging whichever decisions by the Secretary concerning the SPIKEVAX vaccine are open to challenge by them.

The plaintiffs' application is made on two grounds. The first is that the Secretary's decision was unreasonable. The plaintiffs allege that is so because the Secretary could not have reasonably been satisfied of the safety and efficacy of the vaccine, or that its apparent benefits outweighed the associated risks, having regard to the evidence viewed properly and in context. The second is that the Secretary failed to comply with s 30C(2) of the TG Act, which requires the Secretary to give written notice to the Gene Technology Regulator if the therapeutic good is or contains a genetically modified product or organism, and that the decision is therefore invalid. The application is supported by a number of supporting affidavits totalling more than 2,000 pages. Applications for leave to intervene have been filed by Toni Reihana and William Anicha Bay.

Issue:-

Where significant case management and fact finding required, is the matter suitable to be heard in original jurisdiction of High Court of Australia or should the matter be remitted to the Federal Court?

Consideration:-

Section 44(1) of the Judiciary Act 1903 (Cth) empowers the Court on application or by its own motion to remit any matter pending before the Court, subject to exceptions not presently relevant, to another court that has jurisdiction with respect to the subject-matter of and the parties to the proceeding. Section 44(4) provides that such order may be made without an oral hearing. Whilst no formal application has been filed by a party seeking remittal of the matter to another court, the plaintiffs' application anticipated the possibility of remittal and made submissions opposing that course. In the Secretary's response filed 9 February 2023, the Secretary submitted the matter should be remitted to the Federal Court of Australia. In their reply filed 27 February 2023, the plaintiffs responded to the Secretary's submissions on remittal. Accordingly, both parties have had the opportunity to make, and have made, submissions on the issue of remittal, to which I now turn.

The Secretary submits the proceeding should be remitted to the Federal Court, which would have jurisdiction over the subject-matter of the proceeding under s 39B(1) or (1A)(c) of the Judiciary Act. In opposing remittal, the plaintiffs submit that this Court is the only appropriate forum for determination of the proceeding because their claim invites the Court to identify and apply a new category of standing based on the special interest said to arise "[w]here the fabric of human life might be compromised or adversely impacted". Although the plaintiffs acknowledge that "the subject matter of these proceedings is extremely complex (involving issues of modern science)" and that factual disputes may arise, they submit that the nature of any such disputes will be limited to genetically modified medicines and that in any event that factor should not be determinative.

The power to remit is discretionary, "to be exercised after due consideration of all the circumstances of the case". It has been observed that whether the Court exercises that power or permits a proceeding to continue in this Court is a matter "not just for the parties, but for the Court". In that regard, the statement of Brennan CJ in Ravenor Overseas Inc v Readhead as to the purpose of the remittal power is instructive:

"The power of remitter contained in s 44 of the Judiciary Act 1903 (Cth) is designed to ensure that this Court is not diverted from its principal functions by the need to hear and determine matters in the original jurisdiction which could properly be brought in an Australian trial court."

This matter is not suitable to be heard in the original jurisdiction of this Court. It is evident from the parties' submissions that they are unlikely to reach agreement on a set of agreed facts, raising the prospect that significant fact finding will be required. From the parties' respective positions there can be discerned a series of factually intensive issues on which they are, or are likely to become, in dispute, including the nature and extent of the plaintiffs' interest and its sufficiency for purposes of establishing standing, which is acknowledged to be a question of "fact and degree"; whether the vaccine at issue is or contains a genetically modified product or organism (a showing required to be made for the second ground to succeed); and the factual findings of the Secretary's delegates that supplied the basis for the challenged decision. Lay and expert evidence may be required to resolve those issues. The vaccine is sponsored by Moderna Australia Pty Ltd, and it is foreseeable that a question as to its joinder as a party might arise.

Having regard to these considerations, significant case management and fact finding are likely to be required to conduct a hearing of the kind contemplated by the application. Undertaking that task would unduly divert the Court from its principal functions. By contrast, a trial court will be better positioned to case manage the proceeding appropriately and determine contested questions of fact. If the first-instance decision were to be appealed or the subject of an application for judicial review, the appellate court would have the benefit of the trial judge's findings of fact.

I am satisfied that this matter is one "arising under" the TG Act for purposes of s 39B(1A)(c) of the Judiciary Act, and that the Federal Court has jurisdiction over its subject-matter and the parties on that basis. I am also satisfied, for the reasons set out, that this Court should exercise its discretion under s 44(1) of the Judiciary Act to remit the matter to the Federal Court.

Finally, the issues of intervention and expedition and the need for any extension of time for the plaintiffs to seek certain forms of relief are not for this Court to decide, but rather matters for the Federal Court on remitter.

Conclusion:-

The matter be remitted to the Sydney Registry of the Federal Court of Australia.

Added a post 

Davis v Minister for Immigration, Citizenship, Migrant Services and Multicultural Affairs; DCM20 v Secretary of Department of Home Affairs [2023] HCA 10 (12 April 2023)

Intro:-

Two appeals are brought from a decision of the Full Court of the Federal Court of Australia.

Facts:-

Each arises out of an application in the original jurisdiction of the Federal Court for judicial review of a decision of a departmental officer not to refer to a Minister a request to exercise the power conferred on that Minister by s 351(1) of the Migration Act 1958 (Cth) ("the Act") to substitute in the "public interest" a more favourable decision for a decision made or taken to be made by the Administrative Appeals Tribunal ("the Tribunal") under s 349(2)(a) of the Act affirming a refusal by a delegate of the Minister to grant the applicant a visa. The Full Court dismissed an appeal from the dismissal of each application.

Each departmental decision not to refer a request to the Minister was made in purported compliance with instructions issued in 2016 by the then Minister for Immigration and Border Protection ("the 2016 Ministerial Instructions")[2]. The 2016 Ministerial Instructions relevantly instructed departmental officers not to refer a request to exercise the power conferred by s 351 of the Act to a Minister in any case which departmental officers assessed not to "have unique or exceptional circumstances". Each departmental decision not to refer turned on the departmental officer assessing the case in relation to which the request was made not to meet that criterion for referral.

Issue:-

Did the 2016 Ministerial Instructions purport to instruct departmental officers to make decisions required to be exercised personally by Minister?

Consideration:-

The nature and content of the statutory power

Section 351 provides:

"(1) If the Minister thinks that it is in the public interest to do so, the Minister may substitute for a decision of the Tribunal under section 349 another decision, being a decision that is more favourable to the applicant, whether or not the Tribunal had the power to make that other decision.
..

(3) The power under subsection (1) may only be exercised by the Minister personally.

(4) If the Minister substitutes a decision under subsection (1), he or she is to cause to be laid before each House of the Parliament a statement that:

(a) sets out the decision of the Tribunal; and

(b) sets out the decision substituted by the Minister; and

(c) sets out the reasons for the Minister's decision, referring in particular to the Minister's reasons for thinking that his or her actions are in the public interest.

...

(7) The Minister does not have a duty to consider whether to exercise the power under subsection (1) in respect of any decision, whether he or she is requested to do so by the applicant or by any other person, or in any other circumstances."

The references throughout s 351 to "the Minister" encompass any of the Ministers who from time to time administer that section in accordance with Administrative Arrangements Orders issued by the Governor-General[6] as well as any other Minister who might be authorised to act on behalf of such a Minister.

Section 351(1) is a conferral of power on the Minister. Section 351(3) and s 351(7) explain the nature of the power so conferred. The prescription in s 351(3) that the power may only be exercised by the Minister personally means that the power is neither delegable by the Minister under s 496 of the Act nor exercisable on the Minister's behalf by any officer of the Department administered by the Minister under s 64 of the Constitution The prescription in s 351(7) that the Minister does not have a duty to consider whether to exercise the power in any circumstances means exactly what it says. Under no circumstances can the Minister be compelled to exercise the power.

But the power conferred by s 351(1) to make the procedural decision not to consider making a substantive decision in a class of case is not unbounded. The power is bounded by the exclusivity which s 351(3) attaches to the totality of the power which s 351(1) confers on the Minister and which s 351(3) attaches in particular to the assessment of the public interest. Plainly, it would not be open to the Minister to decide not to consider making a substantive decision in a class of case defined by reference to whether a departmental officer or any other person might think it to be not in the public interest to substitute a more favourable decision for a decision of the Tribunal. For the Minister to do so would be for the Minister to abdicate to that other person the core aspect of the substantive decision-making power under s 351(1) which s 351(3) makes clear can be exercised by no one but the Minister.

The statutory power limits the exercise of executive power

The unanimous reasons for judgment of this Court in Brown v West contain the following statement of constitutional principle:

"Whatever the scope of the executive power of the Commonwealth might otherwise be, it is susceptible of control by statute. A valid law of the Commonwealth may so limit or impose conditions on the exercise of the executive power that acts which would otherwise be supported by the executive power fall outside its scope."

The principle is central to the relationship between the Commonwealth Parliament and the Executive Government of the Commonwealth within the system of representative and responsible government established by Chs I and II of the Constitution. The principle was applied in Brown v West to hold that a "necessary implication" of the statutory fixing of the expenditure able to be incurred by a parliamentarian using a postal credit card was to deny the existence of executive power to increase the level of expenditure. The principle is applicable here.

Section 64 of the Constitution requires that Ministers be appointed to administer departments, although it permits several Ministers to be appointed to administer the one department. Section 67 of the Constitution makes clear that departmental officers, like Ministers, are "officers of the Executive Government of the Commonwealth"[.

The relationship between a Minister and the department administered by the Minister which can ordinarily be taken to be contemplated by the Parliament when conferring a discretionary statutory power on a Minister is that described by Brennan J in Minister for Aboriginal Affairs v Peko-Wallsend Ltd:-

"The Department does not have to draw the Minister's attention to every communication it receives and to every fact its officers know. Part of a Department's function is to undertake an analysis, evaluation and précis of material to which the Minister is bound to have regard or to which the Minister may wish to have regard in making decisions. The press of ministerial business necessitates efficient performance of that departmental function. The consequence of supplying a departmental analysis, evaluation and précis is, of course, that the Minister's appreciation of a case depends to a great extent upon the appreciation made by his Department. Reliance on the departmental appreciation is not tantamount to an impermissible delegation of ministerial function. A Minister may retain his power to make a decision while relying on his Department to draw his attention to the salient facts. But if his Department fails to do so, and the validity of the Minister's decision depends upon his having had regard to the salient facts, his ignorance of the facts does not protect the decision. The Parliament can be taken to intend that the Minister will retain control of the process of decision-making while being assisted to make the decision by departmental analysis, evaluation and précis of the material relevant to that decision."

When conferring on a Minister a discretionary statutory power unaccompanied by any duty to consider its exercise, the Parliament can ordinarily be taken to contemplate that the Minister will be able to task the department administered by that Minister with sorting the wheat from the chaff so as to bring to the personal attention of the Minister only those requests for exercises of discretionary statutory powers which departmental officers assess to warrant the Minister's personal consideration. The Parliament can be taken to contemplate that, in undertaking such assessments, departmental officers will comply with instructions issued by or under the authority of the Minister or the Secretary of the department.

All of this accords with the general observation that "when a Minister is entrusted with administrative functions he may, in general, act through a duly authorized officer of his department". The "underlying principle" throughout is that "where a power or function is conferred on a Minister, in circumstances where, given administrative necessity, Parliament cannot have intended the Minister to exercise the power or function personally, an implied power of delegation (or agency) may be inferred". But the availability of such an inference must ultimately depend on the precise statutory scheme.

In enacting s 351 of the Act, the Parliament has seen fit to entrust to the Minister alone the evaluation of the public interest in substituting a more favourable decision for a decision of the Tribunal. A necessary implication of the exclusivity imposed by s 351(3) on the power which s 351(1) confers on the Minister is to deny the existence of executive power to entrust the dispositive evaluation of the public interest in substituting a more favourable decision to an executive officer other than the Minister.

Put another way, the extension by s 61 of the Constitution of the executive power of the Commonwealth to "the execution and maintenance ... of the laws of the Commonwealth" does not authorise a Minister or any other officer of the Executive Government of the Commonwealth to undertake any non-statutory action that is expressly or impliedly excluded by a law of the Commonwealth. By confining evaluation of the public interest for the purpose of s 351(1) to the Minister personally, s 351(3) of the Act effects such an exclusion.

That is the statutory limitation on executive power which will be seen to be transgressed by the 2016 Ministerial Instructions and by the two impugned departmental decisions made in purported compliance with the 2016 Ministerial Instructions.

Conclusion:-

Appeal is allowed and the First Respondent is to pay costs of the Appellants.

Added a post 

GWRV v Minister for Immigration, Citizenship and Multicultural Affairs [2023] FCAFC 39 (15 March 2023)

Intro:-

This is an appeal against a decision of the primary judge to refuse to grant constitutional writ relief against a decision of the Administrative Appeals Tribunal (the Tribunal) to uphold a decision of a delegate of the Minister for Immigration, Citizenship, Migrant Services and Multicultural Affairs (the Minister), to refuse to grant a protection visa to the appellant. The Tribunal’s refusal was on the ground that the Serious Crime Exclusion applied and that accordingly, the appellant was not considered a refugee by operation of s 5H(2)(b) of the Migration Act.

Facts:-

To be granted a protection visa, an applicant must satisfy at least one of the criteria specified in s 36(2) of the Migration Act 1958 (Cth) (Migration Act); the refugee criterion: s 36(2)(a) and (b), and the complementary protection criterion: s 36(2)(aa) and (c). In respect to each, the criterion cannot be met if the decision-maker determining the visa application forms the view that there are serious reasons for considering that the person seeking the visa has committed a serious non-political crime before entering Australia (Serious Crime Exclusion). Although located in different sections of the Migration Act, a substantively identically worded Serious Crime Exclusion disentitling an applicant to refugee and complementary protection applies in respect to each: see s 5H(2)(b) and s 36(2C)(a)(ii).

The appellant is a citizen of Mongolia who arrived in Australia in 2017 on a tourist visa and then moved onto a series of bridging visas. In 2018, his bridging visa was cancelled and he was detained in immigration detention. He applied for the relevant protection visa in 2018. His application was refused by a delegate who found that the appellant was not a refugee within the meaning of s 5H(1) of the Migration Act. Following a successful application for review in the Tribunal (heard within the Migration and Refugee Division of the Tribunal), the appellant's visa application was remitted for consideration with a direction that he is a refugee within the meaning of s 5H(1).

Subsequently, a decision was made by a delegate of the Minister to refuse to grant the appellant a protection visa under s 65 of the Migration Act. The delegate expressly found the Serious Crime Exclusion applied in the context of both the refugee criterion (s 5H(2)(b)) and in the event that the complementary protection criterion was satisfied (s 36(2C)(a)(ii)). The delegate also found that the appellant did not meet the complementary protection criterion because the delegate formed the view that there were not substantial grounds for believing that, as a necessary and foreseeable consequence of the appellant being removed from Australia to Mongolia, there was a real risk that the appellant would suffer significant harm (s 36(2)(aa)).

The Tribunal’s Decision

The Tribunal found that the appellant did not meet the refugee criterion because the Serious Crime Exclusion in s 5H(2)(b) of the Migration Act applied.

Relevantly, the Tribunal was satisfied that there were serious reasons for considering that the appellant had committed the crimes of murder and the procurement of murder, and that each of these were serious non-political crimes. On this basis, the Tribunal found that the appellant did not meet the meaning of refugee by operation of s 5H(2)(b). Accordingly, the appellant was not entitled to a protection visa under s 36(2)(a).

It was accepted by both parties that the Tribunal, unlike the delegate, did not go on to consider separately whether the appellant satisfied the complementary protection criterion in s 36(2)(aa). Further, the Tribunal did not separately consider whether the Serious Crime Exclusion also applied in the context of complementary protection. Nor did the Tribunal discuss whether the Serious Crime Exclusion was substantively identical in the context of protection for refugees and complementary protection and therefore, whether it was of the view that the Serious Crime Exclusion would necessarily apply for complementary protection.

The proceeding before the primary judge

The appellant then applied for constitutional writ relief before the primary judge, who dismissed the application: GWRV v Minister for Immigration, Citizenship, Migrant Services and Multicultural Affairs [2022] FCA 602 (Primary Judgment). The appellant argued in substance the same sole ground of appeal before the primary judge as he argues before this Court, namely that the failure to consider the complementary protection criterion was a jurisdictional error. The primary judge ultimately found at [26] that:-

there could be no error in the Tribunal when sitting in its General Division failing to consider the aspects of the complementary protection criterion that did not concern the Serious Crime Exception. Review of that aspect was outside its jurisdiction. However, it was obliged to consider whether the Serious Crime Exception applied to complementary protection.

Relevantly, the primary judge also found at [29] that the failure to consider complementary protection was not material, because the appellant had conceded that had the Tribunal considered complementary protection, the appellant could not have succeeded on that ground given the finding that the Serious Crime Exclusion applied.

Issues:-

Where first decision maker found complementary protection criterion in s 36(2)(aa) of the Migration Act not satisfied, did bifurcated system of review require Tribunal to also consider s 36(2)(aa)?

Was failure to consider s 36(2)(aa) a material error?

Consideration:-

For the reasons that follow, we are of the view that the Tribunal correctly exercised its decision-making authority when it did not separately consider the complementary protection criterion in s 36(2)(aa) of the Migration Act. It erred, however, in failing to consider the Serious Crime Exclusion in the context of that criterion, because in this case the refusal of complementary protection was, at least in part, based on the Serious Crime Exclusion. But it was necessary for the error to be material in order to be a jurisdictional error, and it was not material.

In that context, before addressing the issues it is appropriate to recall that the Serious Crime Exclusion that disentitles an applicant to refugee and complementary protection is relevantly in substantively identical terms. Section 5H(2) operates to negate a finding that a particular person is a refugee under s 5H(1):

5H Meaning of refugee

...

(2) Subsection (1) does not apply if the Minister has serious reasons for considering that:

(a) the person has committed a crime against peace, a war crime or a crime against humanity, as defined by international instruments prescribed by the regulations; or

(b) the person committed a serious non‑political crime before entering Australia; or

(c) the person has been guilty of acts contrary to the purposes and principles of the United Nations.

(emphasis added)

Section 36(2C)(a) operates to negate a finding that a person satisfies the complementary protection criterion under s 36(2)(aa):

36 Protection visas—criteria provided for by this Act

...

(2C) A non‑citizen is taken not to satisfy the criterion mentioned in paragraph (2)(aa) if:

(a) the Minister has serious reasons for considering that:

(i) the non‑citizen has committed a crime against peace, a war crime or a crime against humanity, as defined by international instruments prescribed by the regulations; or

(ii) the non‑citizen committed a serious non‑political crime before entering Australia; or

(iii) the non‑citizen has been guilty of acts contrary to the purposes and principles of the United Nations; or

(emphasis added)

As readily apparent from the text of those provisions, the exclusion is based on the assessment of the offending conduct, and therefore, satisfaction of one provision necessarily would also result in satisfaction of the other. It would disentitle the applicant to a protection visa on either criterion.

Given that the Tribunal did not turn its mind to and consider separately whether the Serious Crime Exclusion applied to complementary protection, it must follow that it erred to that extent. However, that does not mean that the Tribunal did not consider, as a matter of fact and substance, whether the Serious Crime Exclusion applied to the circumstances of the applicant’s case on review. Thus, the matter comes down to materiality, to which we now turn.

Materiality

We are of the opinion that the materiality requirement that ordinarily applies (MZAPC at [33]) means that the error made was not jurisdictional. Further, as the primary judge concluded, if it had been an error for the Tribunal not to have considered satisfaction of the complementary protection criterion at all, that would not have been material either.

Properly understood, the High Court in MZAPC, enunciated a test for the consideration of jurisdictional error (assuming materiality has been raised as an issue) that has two aspects. First, the reviewing court must consider whether the decision-maker fell into any error. We note that in this case we have found that there was such an error on the part of the Tribunal. Second, if error is identified the reviewing court must consider whether it was material. The majority in MZAPC held that:-

"the burden of the plaintiff is to prove on the balance of probabilities the historical facts necessary to enable the court to be satisfied of the realistic possibility that a different decision could have been made had there been compliance with that condition."

(italics in original)

It is true that the High Court in MZAPC contemplated that there might be some categories of error to which the ordinary threshold of materiality does not apply. The majority said:-

"There are conditions routinely implied into conferrals of statutory decision-making authority by common law principles of interpretation which, of their nature, incorporate an element of materiality, non-compliance with which will result in a decision exceeding the limits of decision-making authority without any additional threshold needing to be met. The standard condition that a decision-maker be free from actual or apprehended bias is one example. The standard condition that the ultimate decision that is made lie within the bounds of reasonableness is another."

(citations omitted)

However, we are of the view that neither the type of error we have found occurred, nor the type of error alleged by the appellant, is of such a nature. In relation to the first, the Tribunal did perform the essence of the task conferred on it by s 500(1)(c) and, in fact and substance, considered the Serious Crime Exclusion, albeit not in the context of complementary protection. And as the primary judge similarly reasoned in relation to the second, alleged error: the Tribunal did conduct a review but, on the appellant's case, misunderstood the extent of its jurisdiction. The ordinary requirement of materiality applies to an alleged error of that kind.

As to whether the error we have found was material, there is an artificiality in the position that the Tribunal was required to separately consider the Serious Crime Exclusion in respect to complementary protection. It is difficult to discern, as a matter of practicality, what separate consideration of the Serious Crime Exclusion would entail. An additional sentence in the Tribunal's decision to the effect that the Serious Crime Exclusion would equivalently apply in the context of the complementary protection criterion, had that criterion been established, would have sufficed. This is the case, noting also that there is no challenge to the finding that the Serious Crime Exclusion applied in respect to the refugee criterion. So if the Tribunal had turned its mind to the question in the context of complementary protection, it would inevitably have found that the exclusion also negated satisfaction of that criterion.

It is also perfectly clear what would have happened if the Tribunal's reasoning had been unaffected by the other, alleged error. Having determined that the appellant was not a refugee because of the operation of the Serious Crime Exclusion in s 5H(2), the Tribunal, if it separately considered the complementary protection criterion, would have refused to grant the protection visa because of the Serious Crime Exclusion in s 36(2C)(a)(ii). This would have been so regardless of whether the Tribunal formed the view that the complementary protection criterion was satisfied: s 36(2)(aa). No such finding could survive the operation of the Serious Crime Exclusion in s 36(2C)(a)(ii) and cause the Tribunal to grant the appellant a protection visa. So much was conceded by the appellant below. As the primary judge correctly concluded, given that concession, the failure to consider complementary protection was not material and there was no jurisdictional error.

The concession was properly made. While it is true that each of s 36(2)(a) and (aa) provide independent criteria, the satisfaction of which can result in the grant of a protection visa, they are both subject to the same Serious Crime Exclusion. As explained above, the Serious Crime Exclusion is worded in substantively identical terms with respect to both criteria. There is no possibility that had the Tribunal proceeded to consider whether the appellant was entitled to complementary protection that it would have granted him a protection visa, because he would have been disentitled under the Serious Crime Exclusion in s 36(2C)(a)(ii). So much was accepted by the appellant.

Accordingly, even if the Tribunal did err in failing to separately consider the complementary protection criterion, we would find that the appellant has not discharged his burden to show that error to be material. Further, we do not consider that any error has been identified in the primary judge’s reasons on materiality.

Conclusion

For the reasons above, the appellant has failed to identify any error in the Primary Judgment and the appeal must be dismissed. There is no reason why costs should not follow the event and, accordingly, we order that the appellant pay the first respondent’s costs.

Added a post 

Minister for Immigration, Citizenship, Migrant Services and Multicultural Affairs v LPSP [2023] FCAFC 24 (3 March 2023)

Intro:-

This is an appeal from the Federal Court of Australia.

Facts:-

The Minister for Immigration, Citizenship, Migrant Services and Multicultural Affairs appeals from the judgment of a single judge exercising the Court’s original jurisdiction. By the judgment under appeal, the judge set aside a Direction made by the Administrative Appeals Tribunal in reliance on s 33 of the Administrative Appeals Tribunal Act 1975 (Cth) (AAT Act).

The Tribunal made the Direction on 11 June 2021, in conducting a review of a decision made by a delegate of the Minister under ss 36 and 65 of the Migration Act 1958 (Cth) to refuse LPSP (the first respondent to this appeal) a protection visa.

The Direction was as follows:-

Pursuant to s 33 of the Administrative Appeals Tribunal Act 1975, the Tribunal DIRECTS that:-

a) On a date to be fixed, but not before 19 June 2021, the Applicant must attend and participate in a consultation with Dr Gosia Wojnarowska, Forensic Consultant Psychiatrist, for the purpose of Dr Wojnarowska preparing an independent expert report; and

b) The Respondent must pay all fees and costs associated with Dr Wojnarowska’s assessment and preparation of her report.

The Direction was accompanied by notes in the following terms:

Notes to Direction

1. If you do not comply with a direction, the Tribunal will list the application for a directions hearing. You will be required to attend the directions hearing in person and explain why you have not complied with the direction.

2. The Tribunal can dismiss an application if an Applicant fails within a reasonable time to comply with a direction made by the Tribunal. This power is set out in section 42A(5)(b) of the Administrative Appeals Tribunal Act 1975. If you are the Applicant and you fail to comply with a direction, you may also be asked to explain at the directions hearing why your application should not be dismissed.

3. If a party fails to comply with this direction, the Tribunal will not necessarily decide to adjourn, or delay the listing of, an alternative dispute resolution process or hearing.

4. Where the Tribunal has the power to award or recommend the payment of costs, failure by a party to comply with this direction may be taken into account in making a decision relating to costs.

5. If you do not believe you will be able to comply with a timeframe or any other aspect of this direction, you must apply to the Tribunal for an extension of time to comply or a variation of the direction. You must seek the views of the other party before making such an application, and advise the Tribunal of the other party’s views. You should make the application well before the date by which you are required to comply with the direction.

6. If you have been directed to give the Tribunal a Hearing Certificate, in accordance with the General Practice Direction, the Tribunal may list your application for hearing without further consultation if you do not provide the Hearing Certificate by the specified date.

LPSP applied to this Court for judicial review of the decision to make the Direction on 18 June 2021. The three grounds of his application were as follows:-

The decision was not authorised by the enactment in pursuance of which it was purported to be made.

Particulars

a. By the decision, the Tribunal purported to make a direction compelling a person (the Applicant) to attend and participate in a medical examination – relevantly here, a psychiatric consultation – to which he did not consent conducted with a medical practitioner chosen by another party (a compulsory medical examination direction).

b. The decision was purported to be made pursuant to s 33 of the Administrative Appeals Tribunal Act 1975 (Cth) (AAT Act).

c. Section 33(2) of the AAT Act relevantly authorises the giving of directions in certain circumstances ‘as to the procedure to be followed in connection with the hearing of a proceeding before the Tribunal’, non-exhaustive examples of which are provided in s 33(3). Those examples do not include, and cannot be interpreted to include, a direction requiring a person to undergo a compulsory medical examination.

d. The power in s 33(2) is a generally worded power directed to procedure, which is insufficiently clearly worded to authorise a compulsory medical examination direction, such a direction interfering with fundamental common law rights and freedoms (including the right to privacy), and the general system of law.

e. Accordingly, s 33(2) did not authorise the direction in the present case.

f. No other sub-provision of s 33 authorised the direction in the present case.

g. The decision to make the direction is amenable to review on this ground by reason of s 5(1)(c) or (d) of the Administrative Decisions (Judicial Review) Act 1977 (Cth) or, alternatively, s 39B of the Judiciary Act 1903 (Cth).

The decision of the Primary Judge onGround One. 

The primary judge held that neither the Migration Act nor the AAT Act empowered the Tribunal to make the Direction, and rejected the Minister’s submission that it did not have a coercive effect. His Honour held that an order or direction of a court or tribunal that would interfere with a fundamental right required specific statutory authority. In reaching this conclusion, the primary judge cited S v S [1972] AC 24; Furesh v Schor [2013] WASCA 231; 45 WAR 546; Hastwell v Kott Gunning (No 5) [2020] FCA 621; and Nursing and Midwifery Board of Australia v HSK [2019] QCA 144; [2019] 1 QR 600. His Honour explained that:-

"Although the present matter concerns the powers of the Tribunal under the Act, in my view, there is no compelling reason to depart from the principle expressed in HSK or Hastwell. Whether the principle identified is an application of the principle of legality or better understood as a narrower principle of statutory construction specific to the interpretation of powers of courts and tribunals is of little moment.

For those reasons and contrary to the Minister’s contention, I regard the principle applied in Hastwell and HSK as being applicable to the construction exercise herein raised. In that regard I do not accept the Minister’s submission that the Direction involves no infringement of the applicant’s fundamental right to not disclose personal and private information."

The primary judge held that the power to issue directions under s 33 of the AAT Act was limited to procedural directions, and that a direction that compelled interference with the liberty of an individual was not a procedural direction. His Honour rejected the Minister’s submission that the Direction fell within s 33(2A)(a) of the AAT Act, and opined that the AAT Act allowed for other means for the Tribunal to ensure that “justice between the parties is achieved in the circumstances at hand”.

Issue:-

The Minister appeals from the judgment of the primary judge on the sole ground of whether the primary judge erred in concluding that the [Tribunal] was not empowered by s 33(2) of the Administrative Appeals Tribunal Act 1975 (Cth) to make the direction dated 11 June 2021, and ought to have concluded that the [Tribunal] was empowered to make the direction.

Consideration:-

Construction of section 33

Plainly enough, s 33 of the AAT Act does not expressly empower the Tribunal to direct an individual to attend and participate in a consultation with a psychiatrist, in order that the psychiatrist prepare a report relevant to an issue arising on review. The power to make such a direction might be implied from the text, context and purpose of the AAT Act, but for the principle of legality to which we shall turn shortly.

The factors supporting such an implication might include:-

- the expressly non-exhaustive character of s 33(2A);

- the discretion conferred by s 33(2A)(a) to require a party to a proceeding to provide further information in relation to the proceeding;
a party’s duty under s 33(1AB) to use the party’s best endeavours to assist the Tribunal to fulfil the objective in section 2A, including to promote a “fair, just, economical, informal and quick” mechanism of review; and

- the Tribunal’s duty under s 43 of the AAT Act to make a decision on review that is correct or preferable on the material before it: see, for example, Shi at [98], [144].

The Direction was not, however, simply a direction about a procedure to be adopted to enable relevant evidence to be obtained and to allow a party before the Tribunal a proper opportunity to respond to the case advanced by the other party, as the Minister submitted. The Direction involved an interference with LPSP’s personal liberty and autonomy. As Boddice J said in HSK at [32], with respect to the Queensland Civil and Administrative Tribunal (QCAT):

"A power to compel an interference with the liberty of an individual litigant is not generally considered necessary for the speedy and fair conduct of a proceeding."

HSK is discussed further below.

As we have seen, LPSP relied on the principle of legality to argue that the Court should not construe s 33 of the AAT Act as conferring the power on the Tribunal to make the Direction, because the Direction would interfere with his fundamental common law freedoms. The principle of legality requires that a legislative intention to diminish a fundamental common law right, freedom, or immunity, or to depart from the general system of law, must be clearly expressed, by “unmistakable and unambiguous language”: Coco v The Queen [1994] HCA 15; 179 CLR 427 at 437. There is no doubt about the principle: see further X7 v Australian Crime Commission [2013] HCA 29; 248 CLR 92 at 153; Electrolux Home Products Pty Ltd v Australian Workers’ Union [2004] HCA 40; 221 CLR 309 at [21]; and North Australian Aboriginal Justice Agency Ltd v Northern Territory [2015] HCA 41; 256 CLR 569 at [11] et seq. As Mason CJ, Brennan, Gaudron and McHugh JJ stated in Coco at 437:-

"The insistence on express authorization of an abrogation or curtailment of a fundamental right, freedom or immunity must be understood as a requirement for some manifestation or indication that the legislature has not only directed its attention to the question of the abrogation or curtailment of such basic rights, freedoms or immunities but has also determined upon abrogation or curtailment of them. The courts should not impute to the legislature an intention to interfere with fundamental rights. Such an intention must be clearly manifested by unmistakable and unambiguous language. General words will rarely be sufficient for that purpose if they do not specifically deal with the question because, in the context in which they appear, they will often be ambiguous on the aspect of interference with fundamental rights."

This principle of construction has been part of Australian law for a long time, as reference to Potter v Minahan [1908] HCA 63; (1908) 7 CLR 277 at 309 shows. The principle militates in favour of a construction that avoids encroachment on fundamental common law rights, freedoms and interests if such a construction is open. Such a construction is evidently open in this case.

There is also no need to explore whether the principle of legality is capable of being engaged in the present context. There is ample authority that a requirement that a person submit to a medical assessment or test against their will involves an interference with fundamental common law freedoms. For this reason, in conformity with the principle of legality, in the absence of clear legislative authorisation, Australian courts will not construe a generally expressed provision, such as s 33 of the AAT Act, as conferring power to make an order or direction requiring a competent person to undergo a medical examination against their will. See, for example, Furesh v Schor; Secretary, Department of Health and Community Services v JWB and SMB [1992] HCA 15, 175 CLR 218 (Marion’s Case); Kurnell Passenger & Transport Service Pty Ltd v Randwick City Council ([2009] NSWCA 59; 2009) 230 FLR 336; Washington v Washington; and HSK.

The Direction mandated a medical (psychiatric) assessment and involved an interference with LPSP’s personal liberty and autonomy. I interpolate here that many have written about the distinction between personal liberty and autonomy. For example, in Two Concepts of Liberty, Sir Isaiah Berlin drew a distinction between “negative freedom” (liberty) – being the freedom from interference from another person or body; and “positive freedom” (autonomy) – being the freedom of the individual to be “his own master”. See Isaiah Berlin, “Two Concepts of Liberty” in Four Essays on Liberty (Oxford University Press, 1969): 118-173, at 121-134. It suffices to say here that, as the following discussion shows, the authorities indicate that, via the principle of legality, the common law is concerned to protect both forms of freedom.

Thus, in Furesh v Schor, the Western Australian Court of Appeal held that it did not have inherent power to order a party to civil proceedings to submit to a mouth swab for DNA analysis against their will. Pullin JA in separate reasons recognised (at [4]) that:-

"The principles which may appear to clash are as follows:

(a) On the one hand, in the administration of justice, courts determine civil disputes by receiving evidence. 

(b) On the other hand, the common law respects and preserves the autonomy of adult persons of sound mind with respect to their body. The common law accepts that a person has rights of control and self-determination in respect of his or her body which other persons must respect. Those rights can only be altered with the consent of the person concerned."

In his Honour’s analysis, the second of these principles prevailed.

The parties’ arguments in this appeal reflected a similar tension between analogous principles. On the one hand, the Minister argued the Tribunal needed to reach the correct or preferable decision by reference to the material before it, which should include the report of a psychiatrist nominated by the Minister. On the other hand, LPSP argued that the common law was protective of individual liberty and autonomy and required clear legislative authority to authorise their curtailment.

Also in Furesh v Schor, in each of their separate reasons, Newnes and Murphy JJA referred to Marion’s Case (and S v S) in support of the proposition that, in the absence of legislation clearly conferring the power to order a party to civil proceedings to submit to a medical assessment or test, no such order could lawfully be made.

Likewise, in Washington v Washington, Parker J held (at [151]) that the South Australian Supreme Court had no inherent power to compel a party to undergo a medical examination to provide evidence in civil proceedings. In these circumstances, his Honour observed:-

"The inherent power of the Court is limited to the grant of a stay if a plaintiff refuses to undergo medical examination. The grant of a stay serves to protect the process of the Court from abuse, and therefore it involves the inherent powers in the narrow sense."

We return to the matter of a stay below.

In Kurnell Passenger & Transport Service Pty Ltd v Randwick City Council [2009] NSWCA 59; (2009) 230 FLR 336, the New South Wales Court of Appeal held that a statutory power to require a worker to submit to a medical examination if required by his employer for one purpose did not permit the employer to require a worker to submit to medical examination for a different purpose.

In Colquhoun v Capital Radiology, in the course of considering the obligation of a statutory medical panel to give reasons, the Victorian Court of Appeal observed that, but for the specific statutory conferral of the power on the panel to require a claimant for damages for non-economic loss to submit to a medical examination, it would not have been open to the panel convenor to “purport (in the exercise of power to give procedural directions) to confer such power”. The Court’s observation is consistent with the decisions of other Australian courts.

More recently, in HSK, the Queensland Court of Appeal determined that it was not open to the QCAT (the State’s administrative merits review body) to require HSK to undergo a health assessment to secure nursing registration. The appellant Board relied on the power conferred by s 62 of the Queensland Civil and Administrative Tribunal Act 2009 (Qld) (Queensland Act) to give directions for the conduct of a review, to support the challenged direction. Boddice J, with whom Morrison and McMurdo JJA agreed, characterised the power conferred by s 62 as a broad, procedural one, to be exercised for the speedy and fair conduct of a proceeding: HSK at [29]. The Court held, however, that QCAT had correctly found that s 62 of the Queensland Act did not authorise a direction that HSK undergo a further health assessment as part of the review: see HSK, at [35]. Boddice J relevantly affirmed that “[a] direction requiring an interference with the liberty of an individual litigant has generally been viewed as requiring specific statutory authority”, and that this was absent from the Queensland Act: see HSK at [34].

In Grant, on the other hand, a Full Court of this Court reached the contrary conclusion after considering the text, context and purpose of the Coal Mining Safety and Health Act 1999 (Qld). The Court held that, as a matter of construction, the Act clearly expressed an intention to curtail coal workers’ personal liberty to the extent that they could be required to attend medical examinations in certain circumstances.

In summary, before a court or tribunal in Australia can require person to undergo a medical examination against their will, the legislature must confer the power to do so in clear and unmistakable terms. This expression of the principle of legality is consistent with the decision of the High Court in Marion’s Case. In that case, respecting the need for consent before a surgical procedure might be performed, McHugh J observed (at 309) that “the common law accepts that a person has rights of control and self-determination in respect of his or her body which other persons must respect”.

The parties to this appeal did not direct the Court to any statutory provision expressly empowering the Tribunal to require a party to submit to a medical assessment against their will. As already noted, s 33 of the AAT Act does not expressly confer such a power. Consideration of the text, context and purpose of the AAT Act does not lead to the conclusion that that Act expresses a clear intention to authorise the Tribunal to curtail a party’s personal liberty and autonomy by requiring him to attend a medical examination against his will. The AAT Act does not manifest an intention to do so by “unmistakable and unambiguous language”: see Coco at 437. Having regard to the foregoing discussion, it would be an error to construe s 33 as conferring power on the Tribunal to do so.
In this context, there is no relevant difference between a physical medical assessment and a psychiatric medical assessment . A physical assessment may typically involve a greater degree of physical interference with the body than a psychiatric assessment, although this need not be so.

There was no specific evidence of what Dr Wojnarowska’s psychiatric assessment of LPSP (as foreshadowed by the Direction) was likely to involve. It would, however, involve some interference with LPSP’s personal liberty insofar as he would be required to make himself available at a particular time and date for the consultation to occur. Whether this was in-person or via electronic means is immaterial. His psychiatric assessment would also evidently involve a curtailment of his autonomy, that is, his freedom to choose if and to whom he would disclose otherwise private information about himself. As already stated, the common law protects both personal liberty and autonomy, that is, both negative and positive personal freedoms.

Australian courts have recognised that, so far as the principle of legality is concerned, there is no relevant difference between being required submit to a physical medical assessment and a psychiatric medical assessment : compare the decision of the English Court of Appeal in Lane v Willis.

In Hastwell, Jackson J proceeded on the basis that the principles concerning the power to order compulsory physical and psychiatric medical examinations were the same: compare Hastwell at [33]. A Full Court of this Court dismissed an appeal from his Honour’s judgment: see Hastwell v Kott Gunning [2021] FCAFC 70. In Washington v Washington, Parker J adopted the same approach in upholding an appeal from the decision of a Master requiring a psychologist’s assessment of a party against their will: see Washington v Washington at [137]. Most recently, the Queensland Court of Appeal in HSK also drew no distinction between a psychiatric and a physical medical assessment in construing s 62 of the Queensland Act by reference to the principle of legality. Section 62 of that Act was similar to s 33 of the AAT Act.

I note at this point that LPSP specifically submitted that this Court should follow HSK since this was a decision of an intermediate appellate court about much the same issue in connection with substantially similar legislation. Strictly speaking, even though HSK and the present appeal give rise to very similar issues, the rule as stated in Australian Securities Commission v Marlborough Gold Mines Ltd [1993] HCA 15; 177 CLR 485 and Farah Constructions Pty Ltd v Say-Dee Pty Ltd [2007] HCA 22; 230 CLR 89 does not in terms apply. This is because HSK concerned a State Act concerning a State tribunal. The State Act also differed from the Commonwealth AAT Act at various points. It may be, however, that Walker Corporation v Sydney Harbour Foreshore Authority [2008] HCA 5; 233 CLR 259 and R v Falzon [2018] HCA 29; 264 CLR 361 have widened this rule. Compare Attorney-General (Cth) v Ogawa [2020] FCAFC 180; 281 FCR 1; MNWA Pty Ltd v Deputy Commissioner of Taxation [2016] FCAFC 154; 250 FCR 381; and Parkin v Boral Limited (Class Closure) [2022] FCAFC 47; 291 FCR 116. As will be seen from the above discussion, it is unnecessary to delve into this question, because we would apply the same principles as the Queensland Court of Appeal in HSK, to reach a like conclusion.

Conclusion:-

The Tribunal might have addressed the difficulty that arose because of LPSP’s opposition to an assessment by a second psychiatrist chosen by the Minister other than by compelling LPSP to participate in a psychiatric assessment with Dr Wojnarowska. For instance, as the primary judge said, the Tribunal could have indicated that it would afford the second Zimmerman report limited weight because another psychiatrist had been unable to corroborate Dr Zimmerman’s opinion and the Minister had limited capacity to test it without another expert’s report.

As we have explained, we agree with the primary judge that the Tribunal did not have the power to make the Direction. Accordingly, we would dismiss the appeal. The Minister should pay the first respondent’s costs of the appeal.

Added a post 

Laundy Hotels (Quarry) Pty Limited v Dyco Hotels Pty Limited [2023] HCA 6 (8 March 2023)

Intro:-

This is an appeal from the Supreme Court of New South Wales and involves a contractual dispute arising from the effects of the COVID‑19 pandemic in Australia.

Facts:-

The contract

The contract was dated 31 January 2020. It provided for the sale of freehold hotel property in Pyrmont, Sydney (the Quarrymans Hotel) ("the Property"), together with an associated hotel Licence (being a specified hotel licence under the Liquor Act 2007 (NSW) and nine Gaming Machine Entitlements allocated to that Licence) and the Business[1]. The Business was defined as the hotel business trading as the Quarrymans Hotel which operates pursuant to the Licence (cl 33.1). The appellant was the Vendor. The first and second respondents together were the Purchaser. The first respondent was the purchaser of the Property and the Licence. The second respondent was the purchaser of the Goodwill, Plant and Equipment and remaining Business Assets[2]. Under cl 65.1 of the contract, the sale of the Property, Licence, and Gaming Machine Entitlements was conditional upon, and interdependent with, the sale of the Business Assets. The total purchase price was $11,250,000.

The Completion Date (specified in item 2 of Sch 1 to the contract) was 55 days after the contract date[3] in respect of the assets to be purchased by the second respondent and 56 days after the contract date in respect of the assets to be purchased by the first respondent. The parties agreed these dates to be 30 and 31 March 2020 respectively.

Clause 50 is the key provision. It was headed "Management Prior to Completion". Clause 50.1, headed "Dealings Pending Completion", provided that:-

"Subject to clause 50.2, from the date of this contract until Completion, the Vendor must carry on the Business in the usual and ordinary course as regards its nature, scope and manner and repair and maintain the Assets in the same manner as repaired and maintained as at the date of this Contract and use reasonable endeavours to ensure all items on the Inventory are in good repair and in proper working order having regard to their condition at the date of this Contract, fair wear and tear excepted." (emphasis added)

This appeal concerns the first limb of cl 50.1

The public health orders

Section 7 of the Public Health Act 2010 (NSW) enabled the Minister by order to give directions if the Minister considered that a situation had arisen that was, or was likely to be, a risk to public health. Under s 10 of that Act, a failure to comply with such a direction was a criminal offence with, relevantly, a maximum penalty of a fine, including a fine for each day the offence continued.

On 23 March 2020, the Minister made an order giving directions in response to the COVID‑19 pandemic. The order directed that pubs (meaning licensed premises under the Liquor Act) "must not be open to members of the public ... except for the purposes of ... selling food or beverages for persons to consume off the premises". The order applied to the Quarrymans Hotel.

In response, the Quarrymans Hotel was closed on 23 March 2020 to enable a shift to a takeaway‑only operation. By 26 March 2020, the Quarrymans Hotel had re‑opened, but only for the purpose of selling takeaway craft beer and food.

Two further public health orders were made on 14 and 29 May 2020 respectively. The order made on 14 May 2020 permitted pubs to sell food or drinks for not more than ten persons to consume on the premises, as well as food or drinks for persons to consume off the premises. The order made on 29 May 2020 permitted 50 persons in a separate seated food or drink area or the total number of persons calculated by allowing four square metres of space per each customer in a pub, whichever was the lesser. The Quarrymans Hotel continued to offer takeaway food and alcohol only until 1 June 2020 when it re‑opened in accordance with the customer number restrictions in the 29 May 2020 order (which commenced on 1 June 2020).

In the meantime, on 25 March 2020, the Purchaser informed the Vendor that it would not complete the contract as the Vendor was not ready, willing and able to complete the contract as the Vendor was in breach of cll 50.1, 58.1 and 58.2. On 27 March 2020, the Purchaser wrote to the Vendor asserting also that the contract had been frustrated or that the Purchaser could issue a notice to complete with which the Vendor could not comply, enabling the Purchaser to terminate the contract and sue for damages. The Vendor responded on the same day that it was ready, willing and able to perform its contractual obligations and called upon the Purchaser to complete the contract.

As noted, Completion was due to occur on 30 and 31 March 2020. On 31 March 2020, the Vendor confirmed that all Conditions Precedent to Completion prescribed by cl 35.1 had been satisfied and said it was "ready, willing and able to settle". The Vendor reiterated this position on 6 April 2020. On 22 April 2020, the Purchaser obtained an updated valuation of the hotel business of $10,250,000, being $1 million less than the contracted purchase price.

Ultimately, the Vendor served a notice to complete on the Purchaser on 28 April 2020 calling for completion of the sale of the Business Assets by 12 May 2020 and of the Property, Licence, and Gaming Machine Entitlements by 13 May 2020. In response, the Purchaser commenced proceedings seeking declaratory relief to the effect that the contract had been frustrated or alternatively that the Vendor was not entitled to issue the notice to complete. On 21 May 2020, the Vendor served a notice of termination on the Purchaser on the basis of the Purchaser's failure to complete in accordance with the notice to complete. On 23 May 2020, the Purchaser responded to the effect that the contract was frustrated but, if that were not so, the Vendor was not entitled to issue the notice to complete and its issue of the notice of termination constituted a repudiation of the contract which the Purchaser accepted.

The primary judge's decision

In the Supreme Court of New South Wales, the primary judge (Darke J) concluded that the contract had not been frustrated[9], and cl 50.1, properly construed, required the Vendor to "carry on the Business in the usual and ordinary course" as far as it remained possible to do so in accordance with law. It followed that the Vendor was not in breach of cl 50.1. Accordingly, the Vendor was entitled to serve the notice to complete, which was effective to make the time for completion essential. As the Purchaser failed to complete, the Vendor was entitled to terminate the contract and was able to seek damages for loss of the bargain.

The Court of Appeal's decision

The Purchaser appealed. The Purchaser alleged that the primary judge misconstrued cl 50.1 and ought to have held that from the coming into force of the first public health order the Vendor was unable to comply with cl 50.1 and, thereby, was not entitled to issue the notice to complete or to terminate when the Purchaser failed to comply with that notice. Accordingly, the Vendor's purported termination constituted a repudiation of the contract, which was accepted by the Purchaser. There was no appeal against the primary judge's conclusion that the contract was not frustrated. The Purchaser also did not allege that the Vendor was in breach of cl 50.1. Rather, the Purchaser's case on appeal was that as the Vendor could not comply with cl 50.1, the Vendor was not ready, willing and able to complete the contract, and therefore could not serve the notice to complete or terminate for the Purchaser's failure to complete.

A majority of the Court of Appeal of the Supreme Court of New South Wales (Bathurst CJ and Brereton JA) allowed the appeal and set aside the orders of the primary judge.

Bathurst CJ concluded that cl 50.1 was not to be construed as if the Vendor's obligation to "carry on the Business in the usual and ordinary course as regards its nature, scope and manner" was limited to the extent permitted by law. Bathurst CJ therefore considered that the public health order made on 23 March 2020 was a supervening event rendering the Vendor's compliance with cl 50.1 illegal. By analogy to cases involving the enforceability of contracts during wartime restrictions and covenants in leases, his Honour considered that the supervening illegality suspended the relevant contractual obligation in cl 50.1. His Honour concluded that cl 50.1 was an essential term with which the Vendor could not comply at the time it served the notice to complete. The purported termination relying on the Purchaser's failure to comply with the notice to complete, accordingly, involved a repudiation of the contract by the Vendor. Brereton JA agreed with Bathurst CJ, but also considered that the Vendor was in breach of cl 50.1 at the time it purported to serve the notice to complete.

Basten JA (in dissent) considered that the obligation in cl 50.1 meant that the Vendor had to "carry on the Business in the usual and ordinary course as regards its nature, scope and manner" as permitted by law. Accordingly, in complying with the public health order, the Vendor was not in breach of cl 50.1. It followed that the appeal had to fail.

Issue:-

Was Appelant in default of Clause 50.1 at the time when notice to complete was served on Respondents?

Consideration:-

The proper construction of the contract

This case is to be resolved on the proper construction of the contract, specifically cl 50.1.

"It is well established that the terms of a commercial contract are to be understood objectively, by what a reasonable businessperson would have understood them to mean, rather than by reference to the subjectively stated intentions of the parties to the contract. In a practical sense, this requires that the reasonable businessperson be placed in the position of the parties. It is from that perspective that the court considers the circumstances surrounding the contract and the commercial purpose and objects to be achieved by it."

It is not necessary to do more than construe cl 50.1 in its context to conclude that the obligation on the Vendor to "carry on the Business in the usual and ordinary course as regards its nature, scope and manner" incorporated an inherent requirement to do so in accordance with law. That is, the obligation imposed on the Vendor was to carry on the Business in the manner it was being conducted at the time of contract to the extent that doing so was lawful. There was no obligation (and could not have been an obligation) imposed on the Vendor to carry on the Business unlawfully. It is not necessary to have recourse to either the doctrine of implied contractual terms to impose on the Vendor an obligation to carry on the business to the extent that it was lawful, or the possible consequences of supervening illegality resulting in suspension rather than frustration of the contractual obligation imposed by cl 50.1.

The obligation in cl 50.1 was for the Vendor to carry on the Business. The "Business" was defined to be "the hotel business trading as the 'Quarrymans Hotel' which operates pursuant to the Licence". The Licence, annexed as Sch 5 to the contract, stated that "Licence conditions imposed by the Liquor Act and Regulation apply". That legislative scheme contains an extensive regime of conditions applying to hotel licences[24], for the regulation and control of licenced premises generally (including powers of the Independent Liquor and Gaming Authority to cancel, suspend, and impose new conditions on Licences), and offences for non‑compliance.

Accordingly, a reasonable businessperson in the position of the parties would have understood cl 50.1 to mean that from the date of the contract until Completion, the Vendor was required to carry on the Business "in the usual and ordinary course as regards its nature, scope and manner" in accordance with law. The past, current, and anticipated future lawfulness of the operation of the Business was objectively essential and a commercial necessity to the parties. Without the Licence and associated Gaming Machine Entitlements, there would be no "Business". The Vendor's obligation to "carry on the Business in the usual and ordinary course as regards its nature, scope and manner", on the proper construction of that provision, could never extend to an obligation on the Vendor to act illegally. The Vendor's obligation was necessarily moulded by, and subject to, the operation of the law from time to time.

Clause 50.1 is not a provision which, on its proper construction, has a "double intendment", in the sense of contemplating an operation both within and against the law so that the provision should be construed as meaning only the former intendment. Clause 50.1 has a single intendment – that the Vendor's obligation to "carry on the Business in the usual and ordinary course as regards its nature, scope and manner" is moulded by, and subject to, the law as in force from time to time. The contrary construction would require "intractable language" giving effect to an inferred objective intention of the parties that the Vendor be obliged to "carry on the Business in the usual and ordinary course as regards its nature, scope and manner" contrary to the law as in force from time to time and thereby place at risk the continuation of the Licence.

This conclusion is reinforced by the description that the Business is to be carried on in its "usual and ordinary course". The Business, which operates pursuant to the Licence, in its usual and ordinary course must operate in accordance with law. The further description "as regards its nature, scope and manner" reflects the objectively assumed common position of the parties at the date of the contract that the nature, scope and manner of the Business at that time was in the usual and ordinary course – that is, in "the undistinguished common flow" of the business – which inherently encompasses that the business was lawful.

The relevant obligation of the Vendor in cl 50.1 cannot be construed as if one part (the "usual and ordinary course") incorporates a requirement of lawfulness, but the other part (the "nature, scope and manner") does not. Otherwise, in the event of supervening illegality, the clause would contain potentially irreconcilable obligations in that the Vendor would be obliged to carry on the Business both in the "nature, scope and manner" that it operated at the contract date (on this hypothesis, unlawfully) and "in the usual and ordinary course" that it operated at the contract date (that is, lawfully). Construed as it must be, as a single obligation subject to an overriding qualification of lawfulness, the relevant part of cl 50.1 reflects the commercial reality that ongoing legal compliance was essential to the Business.

For these reasons, the Vendor was complying with cl 50.1 (and cll 58.1 and 58.2) of the contract at the time of Completion. The fact that the then extant public health order prevented the Vendor from carrying on the Business in the same way as it had been carried on at the contract date did not mean that the Vendor was not complying or could not comply with cl 50.1. The Vendor was "ready, willing and able to complete and ... not in default" in accordance with cl 51.7(b). Accordingly, the Vendor was able to serve the notice to complete making time of the essence for Completion as provided for in cl 51.7(b)(ii). By not completing as required, the Purchaser was in breach of the contract in an essential respect, entitling the Vendor to terminate the contract by notice under cl 63.1 (and to keep the Deposit and sue for damages).

On this basis, the status of cl 50.1 as an essential or intermediate term is immaterial. So too are the difficulties and uncertainties associated with any supposed doctrine of the suspension of a contractual promise temporarily incapable of being satisfied by reason of supervening illegality.

Conclusion:-

The appeal be allowed with costs.

 

 

Added a post 

Unions NSW v New South Wales [2023] HCA 4 (15 February 2023)

Intro:-

Facts:-

There has been a series of cases in the original jurisdiction of this Court concerned with the constitutional validity of provisions of legislation in New South Wales regulating political donations and expenditure on campaigns for elections in that State. This is the second case dealing with the validity of caps on the electoral expenditure of third-party campaigners ("TPCs") – broadly, a person or entity, other than a political party, associated entity, elected member or candidate, who seeks to participate in a New South Wales election campaign and incurs over $2,000.

The Electoral Funding Act, 2018 (NSW) , among other things, makes "provision for the disclosure, capping and prohibition of certain political donations and electoral expenditure for parliamentary and local government election campaigns". It creates a comprehensive scheme regulating the extent and sources of funding for elections, and requiring regular disclosure to the New South Wales Electoral Commission of political donations and of electoral expenditure by parties, elected members, candidates, groups of candidates and associated entities, as well as by major political donors and TPCs in certain circumstances.

Each of the plaintiffs was registered under the EFED Act and EF Act as a TPC for past New South Wales State elections and by‑elections. Each plaintiff asserted an intention to be registered as a TPC under the EF Act for future NSW State elections and by‑elections, and asserted an intention to incur "electoral expenditure" during the "capped State expenditure period" in those elections and by‑elections, to the extent permitted by law. The plaintiffs contended that two provisions of the EF Act governing TPCs impermissibly burdened the implied freedom of communication on governmental and political matters protected by the Constitution, and sought declarations of invalidity.

The first provision, s 29(11) of the EF Act, caps TPCs' "electoral expenditure" in the "capped State expenditure period" before a State by‑election for the Legislative Assembly to $20,000 indexed for inflation. Section 29(11), read with s 33(1), prohibits TPCs from incurring electoral expenditure exceeding that $20,000 indexed cap.

After the Court reserved its decision on the validity of s 29(11) of the EF Act, the State informed the Court that, on 23 November 2022, the New South Wales Joint Standing Committee on Electoral Matters ("JSCEM") had delivered Report 2/57 entitled "Caps on third-party campaigners' electoral expenditure in s 29(11) and s 35 of the Electoral Funding Act 2018" ("the 2022 JSCEM Report"). Among other things, the 2022 JSCEM Report recommended that the expenditure cap for TPCs in by‑elections be increased to $198,750. In light of that Report, the State said that it now conceded that s 29(11) was invalid. Orders were made by the Court to supplement the Further Amended Special Case with the 2022 JSCEM Report. The invalidity of s 29(11) is addressed later in these reasons.

The second provision, s 35 of the EF Act, created an offence, applicable only to TPCs, for acting in concert with another person or persons to incur electoral expenditure in relation to an election campaign during the capped expenditure period that exceeded the cap applicable to the TPC for the election. Section 35(2) provided that a person "acts in concert" with another person if the person acts under an agreement, whether formal or informal, with the other person to campaign with the object, or principal object, of having a particular party, elected member or candidate elected, or opposing the election of a particular party, elected member or candidate.

Two weeks before the hearing of this proceeding, the New South Wales Parliament repealed s 35 of the EF Act[22]. Following the repeal, the plaintiffs amended their statement of claim to seek a declaration that s 35, as it stood from 1 July 2018 until 2 November 2022, was invalid. The plaintiffs submitted that the Court retains jurisdiction in the matter so far as it concerns the validity of s 35. The State, and the Commonwealth intervening, contended that, in the circumstances, the Court no longer has jurisdiction to determine the validity of s 35.

Issue:-

After the repeal of s 35 of the EF Act, the following questions were stated for the opinion of the Full Court:

"1. Is section 29(11) of the [EF Act] invalid because it impermissibly burdens the implied freedom of communication on governmental and political matters, contrary to the Commonwealth Constitution?

1A. As to proposed question 2 below:

Does the Court have jurisdiction to hear and determine the question?

Should the Court in its discretion hear and determine the question?

If the answer[s] to questions 1A(a) and (b) are 'yes': was section 35 of the [EF Act], as it stood from 1 July 2018 until 2 November 2022, invalid because it impermissibly burdened the implied freedom of communication on governmental and political matters, contrary to the Commonwealth Constitution?

Who should pay the costs of the special case?"

Consideration:-

Question 1A – Jurisdiction to determine validity of s 35

The Court first heard argument on question 1A: whether, given the repeal of s 35, the Court had jurisdiction to hear and determine the constitutional validity of that section, and if it had jurisdiction, whether it should determine the question. At the end of argument, the Court informed the parties and the intervener that at least a majority of the Court would answer either question 1A(a) or 1A(b), "No". The Court therefore did not hear argument on question 2, as that question does not arise. What follows are the reasons for answering question 1A(a) "No", and question 1A(b) "Unnecessary to answer".

The requirement of a "matter"

The judicial power of the Commonwealth is vested by s 71 of the Constitution in the High Court, and such other federal courts as the Parliament creates or vests with federal jurisdiction. "Jurisdiction" is the "generic term" for the authority to adjudicate. Federal jurisdiction is the authority to adjudicate – the authority to exercise the judicial power of the Commonwealth – derived from the Constitution and laws passed by the Commonwealth Parliament under the Constitution. The extent of this Court's authority to exercise that power (and the authority of other courts invested with federal jurisdiction) is limited by the Constitution, reflecting notions of the separation of powers, and of responsible and representative government, found in the text and structure of the Constitution.

Consistent with those fundamental principles, the function of the Court is not the giving of legal answers or the declaration of legal principle – it is the resolution of a controversy about a legal right, duty or liability. The giving of answers or the making of declarations is an exercise of judicial power only where the seeking and giving of those answers or declarations arise in or out of the judicial determination of the rights and liabilities in issue in the dispute]. That understanding of the Court's function is reflected in the constitutional requirement that a dispute involves a "matter" for the purposes of Ch III of the Constitution.

Exceptional categories aside, there can be no "matter" within the meaning of Ch III of the Constitution unless "there is some immediate right, duty or liability to be established by the determination of the Court" in the administration of a lawand unless the determination can result in the Court granting relief which both quells a controversy between parties and is available at the suit of the party seeking that relief. Standing to seek relief is in that way "subsumed within the constitutional requirement of a 'matter'". While the concepts of standing and matter are not entirely co‑extensive, both are concerned to "mark out the boundaries of judicial power"; their attempted severance is "conceptually awkward, if not impossible".

What is required to establish standing varies with the nature of the relief that is sought and will apply differently to different sorts of controversies. Where, as here, the relief sought is declaratory of the invalidity of legislation, standing has traditionally been explained in terms of a requirement for the party seeking the relief to have a "real" or "sufficient" interest in obtaining the relief. That requirement is closely aligned with the requirement that, for the making of a declaration to constitute an exercise of judicial power, the declaration must be seen at the time of its making to produce foreseeable consequences for the parties.

Croome v Tasmania confirmed the "long-standing doctrine" that a "matter" can "consist of a controversy between a person who has a sufficient interest in the subject and who asserts that a purported law is invalid and the polity whose law it purports to be". The sufficiency of the interest in such a case provides standing to seek a declaration that the law is invalid. The immediate right, duty or liability to be established by the determination of the Court is the right to the declaration sought. The law that is administered by the Court is the Constitution.

As the standing of a party to seek declaratory relief depends on the sufficiency of the interest of that party in obtaining that relief, a sufficient interest must continue to subsist up until the time at which relief is granted or refused. If, after the commencement of a proceeding, a party ceases to have a sufficient interest in obtaining the relief sought, that party no longer has standing to obtain that relief, the "matter" ceases to exist and, in consequence, the jurisdiction of the Court comes to an end. But that is not to say that the interest must remain the same throughout the proceeding; the nature of a party's interest may change but still remain sufficient.

The need for standing, as a component of a "matter", to continue to exist throughout a proceeding for the Court to have jurisdiction to continue to entertain the proceeding is consistent with the settled understanding that a "matter" is a justiciable controversy identifiable independently of a proceeding. If a "matter" can exist whether or not a proceeding has been commenced, a "matter" can cease to exist after a proceeding has been commenced.

In this case, the question whether there is a justiciable controversy in relation to the constitutional validity of a law – s 35 of the EF Act – that was repealed after the proceeding had been filed is able to be addressed by asking if the applicable principles permit the plaintiffs to seek a declaration of invalidity.

Standing to seek a declaration

A plaintiff will have and maintain a real or sufficient interest in obtaining relief if and for so long as they seek a declaration of their own rights, legal interests or liabilities, or if and for so long as the declaration sought will directly affect their rights, legal interests or liabilities. Generally, such a declaration will have foreseeable consequences for the plaintiff because they will be able to legally enforce those rights, interests or liabilities. So, for example, a declaration of invalidity of a law (even where the law has been repealed or amended) may have foreseeable consequences for that plaintiff where such a declaration assists to negative a statutory defence to a common law cause of action such as an intentional tort, or where the plaintiff is being prosecuted for breach of that law. The past infringement of certain personal rights or interests of a plaintiff, such as reputation and liberty, may also be sufficient for seeking declaratory relief even where there are no other asserted legal consequences.

But when a plaintiff seeks a declaration not of personal rights or liabilities – for example, a declaration of invalidity of a law for breach of the implied freedom of political communication, which is not a personal right – a plaintiff must establish an interest other than that which any other ordinary member of the public has in upholding the law generally. A person is not sufficiently interested "unless [they are] likely to gain some advantage, other than the satisfaction of righting a wrong, upholding a principle or winning a contest, if [their] action succeeds or to suffer some disadvantage, other than a sense of grievance or a debt for costs, if [their] action fails". The test for a sufficient interest is broad and flexible, varying according to the nature and subject matter of the litigation. However, whether a plaintiff's interest is sufficient is a question of degree, not a question of discretion. The plaintiff must show that "success in the action would confer on [them] ... a benefit or advantage greater than [that] conferred upon the ordinary member of the community; or ... relieve [them] of a detriment or disadvantage to which [they] would otherwise have been subject ... to an extent greater than the ordinary member of the community". They must have more than a mere intellectual or emotional concern, and more than a belief, however strongly held, that the law or the Constitution should be upheld. As Croome demonstrates, a plaintiff may have a sufficient interest where their freedom of action is particularly affected by the impugned law. Other cases, such as Onus v Alcoa of Australia Ltd, demonstrate that the breadth of the categories of interest include economic, cultural and environmental interests.

No continuing matter in relation to s 35 of the EF Act

When the plaintiffs commenced these proceedings in June 2022, the matter within the Court's jurisdiction included the plaintiffs' challenge to the validity of s 35 of the EF Act. As TPCs registered under the EF Act whose expenditure and campaigning had been affected by that provision and could be expected to be affected in future elections, the plaintiffs had standing and a sufficient interest to seek a declaration as to the invalidity of s 35.

However, that state of affairs changed on 2 November 2022 when the Parliament of New South Wales repealed s 35. As mentioned, following that repeal, the plaintiffs amended the prayer for relief in their statement of claim to seek a declaration that s 35 of the EF Act, as it stood from 1 July 2018 until 2 November 2022, was invalid. The plaintiffs submitted that the Court retains jurisdiction in the matter so far as it concerns the validity of s 35, because the plaintiffs suffered for four years under the burden of the statutory norm in s 35 and modified their behaviour to avoid its criminal sanction. The plaintiffs also submitted that they have a reasonable apprehension that the State may reintroduce a provision in "materially similar" terms to s 35, and may do so before the State general election in May 2023. Neither of those matters assists the plaintiffs.

The plaintiffs have not demonstrated that they continue to have standing, or a real or sufficient interest, to seek a declaration as to the invalidity of s 35. The plaintiffs cannot and do not assert that any of their rights, duties or legal interests have been infringed by the past application of s 35. The plaintiffs are not the subject of enforcement action for any past breach of s 35, nor do they claim to have contravened s 35 in the past[58]. Nor do they assert that a declaration of invalidity would assist them in vindicating any right, duty or legal interest. The plaintiffs assert invalidity solely by reference to the implied freedom of political communication. That implied freedom is not a personal right[59]; it is a freedom from unjustified legislative interference.

For those reasons, the Court no longer has jurisdiction to hear and determine the plaintiffs' claim with respect to the purported invalidity of s 35 of the EF Act, now repealed. The scope of the constitutional "matter" before the Court has contracted and the only remaining issue is the validity of s 29(11). Question 1A(a) should be answered "No".
Given that conclusion, it is unnecessary to answer question 1A(b) – whether the Court, in its discretion, should hear and determine the question of the purported invalidity of s 35 of the EF Act, now repealed. That question was premised on the Court having jurisdiction to hear and determine that claim.

Question 1 – Validity of s 29(11) of the EF Act

It was accepted by the parties and the Commonwealth that caps on electoral expenditure (including s 29(11) of the EF Act) impose an effective and direct burden on political communication by restricting the capacity of the persons to whom they apply to participate in political debate during an election campaign[69], and that where a law imposes a burden on the freedom, it must be justified[70].

It was also accepted that the polity imposing the burden on political communication bears the persuasive onus of establishing that justification[71]. That is, at least in a practical sense, it is for the State defending the validity of the impugned provisions to justify the burden[72]. The Court must be satisfied of the existence of facts on which the State's justification for the burden depends[73]. That requirement was determinative in Unions [No 2].

As explained earlier in these reasons, shortly after oral argument had completed, the State told the Court that it now accepted that s 29(11) of the EF Act was invalid. The State said that it had changed its position because the JSCEM had just reported to the Parliament of New South Wales, among other things, that the cap on TPCs for a by-election should be increased to $198,750 indexed. The plaintiffs and the State consented to the 2022 JSCEM Report being brought to the attention of the Court. The parties submitted that the Court should answer question 1 stated for the opinion of the Full Court (which asked whether s 29(11) is invalid) "Yes".

Questions of the validity of a law cannot be decided by agreement of the parties. It is for the Court to be satisfied that a law is invalid before answering in that way a question reserved for the opinion of the Full Court and before granting any final declaratory or other relief. In the present case, the point of determinative significance is that the State no longer seeks to justify the burden which s 29(11) imposes on communication on governmental and political matters. And, as explained above, the polity imposing the burden bears the persuasive onus of establishing that the burden is justified.

The plaintiffs maintain their complaint that s 29(11) is invalid. This provision (unlike s 35) has not been repealed. It remains a purported law of New South Wales. The State has said nothing about any proposal to repeal or amend s 29(11). The plaintiffs have a sufficient interest to seek relief and the State accepts (by its agreement that question 1 should be answered "Yes") that there is continued utility in answering question 1 in that way. Only if that is done (and the law is declared invalid) are the plaintiffs relieved of the purported effect of s 29(11) on their electoral expenditure in future by-elections and the risk of the attempted enforcement of that provision. Section 29(11) imposes a burden on the implied freedom and that burden has not been justified. For those reasons, question 1 should be answered "Yes".

Conclusion:-

For those reasons, the questions stated by the parties for the opinion of the Full Court should be answered as follows:
Question 1: Yes.

Question 1A: (a) No.

(b) Unnecessary to answer.

Question 2: Does not arise.

Question 3: In relation to question 1, the defendant. Otherwise, there should be no order as to costs.

Added a post 

Stanley v Director of Public Prosecutions (NSW) [2023] HCA 3 (15 February 2023)

Intro:

This is an appeal from the Supreme Court of New South Wales.

Facts:

In summary, the District Court Judge dismissed the appeal and imposed upon the appellant a sentence of imprisonment to be served by full-time detention without undertaking the assessment mandated by s 66(2) of the relative merits of full-time detention as against intensive correction in the community, for the purposes of considering the "paramount consideration" of community safety identified in s 66(1). In failing to undertake that assessment, the District Court Judge misconstrued s 66 and thereby both misconceived the nature of her function under s 7 of that Act and disregarded a matter that the Sentencing Procedure Act required to be taken into account as a condition or limit of jurisdiction.

Where the power to make an ICO is enlivened, a sentencing court does not have jurisdiction to decide that a sentence of imprisonment is to be served by full-time detention without assessing the comparative merits of full-time detention and intensive correction for reducing the offender's particular risk of reoffending. The District Court Judge's error of law can be understood as an instance of both the second and third examples of jurisdictional error on the part of an inferior court identified in Kirk v Industrial Court (NSW)[53]. It was properly conceded by counsel for the first respondent, in her clear and comprehensive written and oral submissions, that, s 66 aside, every other provision in Div 2 of Pt 5 of the Sentencing Procedure Act, headed "Restrictions on power to make intensive correction orders", contains one or more jurisdictional conditions. On a proper construction of s 66, that provision is no exception.

Issue:-

Did the District judge fail to consider Section 66 before declining to make an intensive correction order and was that failure jurisdictional error?

Consideration:-

Jurisdictional error by an inferior court

The Supreme Court's jurisdiction to determine proceedings for judicial review of a sentence has been held to be limited to review for jurisdictional error of law, due to the constraint of the privative clause in s 176 of the District Court Act 1973 (NSW), which provides that "[n]o adjudication on appeal of the District Court is to be removed by any order into the Supreme Court". The District Court is a court of limited jurisdiction, and an inferior court. Whether an error of law by an inferior court, such as the District Court, is jurisdictional will depend on the proper construction of the relevant statute.

In Craig v South Australia, the Court described the scope of an inferior court's ordinary jurisdiction in the following passage:-

"[T]he ordinary jurisdiction of a court of law encompasses authority to decide questions of law, as well as questions of fact, involved in matters which it has jurisdiction to determine. The identification of relevant issues, the formulation of relevant questions and the determination of what is and what is not relevant evidence are all routine steps in the discharge of that ordinary jurisdiction. Demonstrable mistake in the identification of such issues or the formulation of such questions will commonly involve error of law which may, if an appeal is available and is pursued, be corrected by an appellate court and, depending on the circumstances, found an order setting aside the order or decision of the inferior court. Such a mistake on the part of an inferior court entrusted with authority to identify, formulate and determine such issues and questions will not, however, ordinarily constitute jurisdictional error. Similarly, a failure by an inferior court to take into account some matter which it was, as a matter of law, required to take into account in determining a question within jurisdiction or reliance by such a court upon some irrelevant matter upon which it was, as a matter of law, not entitled to rely in determining such a question will not ordinarily involve jurisdictional error."

The circumstances in which an inferior court may fall into jurisdictional error are not closed. In Craig, the Court gave examples of the circumstances in which an inferior court will fall into jurisdictional error, including, as is presently relevant, "if it misconstrues [the statute conferring its jurisdiction] ... and thereby misconceives the nature of the function which it is performing or the extent of its powers in the circumstances of the particular case", or "if it misapprehends or disregards the nature or limits of its functions or powers in a case where it correctly recognises that jurisdiction does exist", or if it "disregards ... some matter in circumstances where the statute ... conferring its jurisdiction requires that that particular matter be taken into account ... as a pre-condition of the existence of any authority to make an order". For instance, in Samad v District Court of New South Wales, certiorari was granted to quash a decision based on the District Court Judge's misapprehension of the scope of his discretion to cancel a licence. In this case, it is not necessary to go beyond the instances of jurisdictional error by an inferior court that were identified in Craig and reinforced in Kirk.

Legislative framework

Three steps to the sentencing process

There are three steps to be undertaken by a sentencing court prior to the final order by which a sentence of imprisonment is imposed under the Sentencing Procedure Act, or confirmed or varied on a sentencing appeal: first, a determination that the threshold in s 5(1), described below, is met; second, determination of the appropriate term of the sentence of imprisonment; and third, where the issue arises, consideration of whether or not to make an ICO. The identification of these steps does not conflict with the principle, stated in Markarian v The Queen, that sentencing does not involve a mathematical approach of increments to and decrements from a predetermined range of sentences. The sentencing court must engage in a process of instinctive synthesis of multiple factors at each stage of the sentencing process.

The first step requires the court to be satisfied, having considered all possible alternatives, that no penalty other than imprisonment is appropriate[69]. The possible alternative penalties include a community correction order[, a conditional release order, conviction with no other penalty and a fine. An ICO is not an alternative penalty.

ICOs are of a different kind – an ICO is a sentence of imprisonment (for the purposes of s 5) that is directed, under s 7, to be served by way of intensive correction in the community rather than full‑time detention. Section 7, headed "Intensive correction orders", provides:

"(1) A court that has sentenced an offender to imprisonment in respect of 1 or more offences may make an intensive correction order directing that the sentence or sentences be served by way of intensive correction in the community.

(2) If the court makes an intensive correction order directing that a sentence of imprisonment be served by way of intensive correction in the community, the court is not to set a non-parole period for the sentence.

(3) This section does not apply to an offender who is under the age of 18 years.

(4) This section is subject to the provisions of Part 5."

Power arises after sentence of imprisonment imposed

There was no dispute that the power to order or decline to order an ICO under s 7(1) is a discrete function that arises after the sentencing court has imposed a sentence of imprisonment. That is clear from the words of s 7(1). The possibility of an ICO does not arise unless and until the sentencing court has first determined that no penalty other than imprisonment is appropriate and has sentenced an offender to imprisonment[75].

Power to make or refuse to make an ICO

Discretionary power, corresponding duty

The power to make, or refuse to make, an ICO is discretionary. However, as the parties accepted, that conferral of power comes with a corresponding duty. The court will come under a duty to consider whether to make an ICO where that matter is properly raised in the circumstances of the case, and where the disentitling provisions identified below are not engaged[76]. This is consistent with the general principle that, where a jurisdiction is conferred and "created for the public benefit or for the purpose of conferring rights or benefits upon persons the court upon an application properly made is under a duty to exercise its jurisdiction and is not at liberty to refuse to deal with the matter"[77].

Provisions defining the jurisdiction to make an ICO

Once the power to make an ICO is enlivened, the sentencing court must address the requirements in the Sentencing Procedure Act relevant to the imposition of such an order.

Section 66 – Community safety and other considerations

Section 66, headed "Community safety and other considerations", provides (emphasis added):
"(1) Community safety must be the paramount consideration when the sentencing court is deciding whether to make an intensive correction order in relation to an offender.

(2) When considering community safety, the sentencing court is to assess whether making the order or serving the sentence by way of full-time detention is more likely to address the offender's risk of reoffending.

(3) When deciding whether to make an intensive correction order, the sentencing court must also consider the provisions of section 3A (Purposes of sentencing) and any relevant common law sentencing principles, and may consider any other matters that the court thinks relevant."

There was no dispute before this Court that s 66 imposes specific mandatory considerations upon the decision maker to make, or refuse to make, an ICO. Section 66(1) requires the court to treat community safety as the "paramount consideration". In the context of s 66(2), community safety principally concerns the possible harms to the community that might occur in the future from the risk of reoffending by the offender. The issue is not merely the offender's risk of reoffending, but the narrower risk of reoffending in a manner that may adversely affect community safety.

The identification of community safety in s 66(1) as the "paramount" consideration also indicates that s 66 is concerned with an aspect of the sentencing task that requires the sentencing court to have a particular and different focus at the third stage of the three-step process described earlier. When the court is deciding the discrete question whether or not to make an ICO, community safety is the consideration to which other considerations are to be subordinated, although other considerations must or may be taken into account as prescribed by s 66(3)[89].

Section 66(2) explains how the sentencing court must engage with the paramount consideration of community safety. For the purpose of addressing community safety, s 66(2) requires the sentencing court to undertake a task of assessing the possible impacts of an ICO or full-time detention on the offender's risk of reoffending. Section 66(2) gives effect to Parliament's recognition that, in some cases, community safety will be better promoted by a term of imprisonment served in the community than by full-time detention. Section 66(2) is premised upon the view that an offender's risk of reoffending may be different depending upon how their sentence of imprisonment is served, and implicitly rejects any assumption that full-time detention of the offender will most effectively promote community safety. Thus, s 66(2) requires the sentencing court to look forward to the future possible impacts of the sentence of imprisonment, depending upon whether the sentence is served by way of full-time detention or by way of intensive correction in the community.

The assessment required by s 66(2) is not determinative of whether an ICO may or should be made. To the contrary, as is plain from s 66(3), the assessment is required for the purpose of addressing community safety as the paramount, but not the sole, consideration in deciding whether or not to make an ICO. Thus, the power to make an ICO requires an evaluative exercise that treats community safety as the paramount consideration, with the benefit of the assessment mandated by s 66(2). In that respect, the nature and content of the conditions that might be imposed by an ICO will be important in measuring the risk of reoffending.

Failure to undertake assessment in s 66(2) is jurisdictional error

Whether s 66(2) imposed a condition or limit upon the power of the District Court Judge or affected the nature of the function to be performed by her Honour in deciding whether or not to make an ICO is a matter of statutory construction. The appellant did not seek to contend that the s 66(2) assessment was required to establish any jurisdictional fact[91]. Nor did the appellant treat the s 66(1) "paramount consideration" as merely a relevant consideration. As appears from Craig, a failure by a sentencing court to take into account a relevant consideration in the course of arriving at a sentencing decision will not ordinarily be jurisdictional error without more. Rather, the following matters combine to illustrate the jurisdictional nature of the paramount consideration in s 66(1) as directed by the assessment in s 66(2).

Assessment required by s 66

The inclusion of s 66 in Div 2 of Pt 5, which, as has been observed, is headed "Restrictions on power to make intensive correction orders", is an indication that the legislature intended s 66 to operate as an enforceable limit upon power. The Division heading is taken as part of the Act[92]. As identified above, Div 2 contains several restrictions on the power to make ICOs. As a general proposition, Div 2 reveals a clear legislative intention that sentencing courts are not "islands of power immune from supervision and restraint"[93] in respect of compliance with Div 2. The requirement for the assessment under s 66 is a limit that operates at the third step in the sentencing process, that is, the limit affects the power to decide whether or not to make an ICO under s 7; it does not operate at the first and second steps of deciding whether to impose a sentence of imprisonment and, if so, the term of the sentence.

A failure to undertake the assessment required by s 66(2) does not merely involve a mistake in the identification of relevant issues, the formulation of relevant questions or the determination of what was or was not relevant evidence[94]. Rather, it is a failure to undertake a task that is mandated for the purpose of deciding whether to make an ICO by reference to community safety as the paramount consideration. Such an error tends to defeat the evident statutory aim of improving community safety through provision of an alternative way to serve sentences of imprisonment by way of intensive correction in the community. The legislative importance of that aim is reinforced both by the characterisation of community safety as a "paramount" consideration and by the stipulation of the assessment task in s 66(2) to inform the consideration of community safety.

The jurisdiction conferred by s 7 is thus to decide whether community safety as a paramount consideration together with the subordinate considerations in s 66(3) warrant full-time detention or intensive correction in the community. The s 66(2) assessment is integral to the function of choosing between full-time detention and intensive correction in the community in compliance with the requirement in s 66(1) to treat community safety as the paramount consideration.

The question raised by this appeal is whether an error in undertaking this discrete task at the third step of the sentencing process can be characterised as one going to the jurisdiction of the sentencing court. There is no basis to assume that an error at that step is "necessarily" an error within the sentencing court's jurisdiction simply because it follows the imposition of a sentence of imprisonment. As explained, the jurisdiction to grant an ICO calls for a subsequent and separate decision to be made after a sentence of imprisonment is imposed. The fact that the sentencing court may have acted within jurisdiction at the first and second steps in imposing the sentence of imprisonment does not mean that the sentencing court will necessarily remain within jurisdiction when making the separate decision whether to order an ICO. Section 7 is not an inconsequential subsequent power after the sentencing process is complete. Section 66 is "more than one evaluative step amongst many" that the Act requires to be carried out after a sentence of imprisonment is imposed. Section 7 is itself a sentencing function that is to be exercised by reference to the paramount consideration in s 66(1). It is a discretionary power – which, when enlivened, comes with a corresponding duty – that fundamentally changes the nature of the sentence of imprisonment imposed from full-time detention to one of intensive correction in the community. The sentencing court may bring itself outside of jurisdiction if it misconceives the nature of that function or fails to comply with a condition on the jurisdiction when exercising the power. And, as will be seen, that is what the District Court did in this case.

Jurisdictional error

In addressing the appellant's application for an ICO, the District Court Judge did not refer to s 66 of the Sentencing Procedure Act specifically or in substance, although her reference to community safety as the paramount consideration indicated an awareness of the provision. Her Honour did not record any findings about whether an ICO or full-time detention was more likely to address the appellant's risk of reoffending. Nor did her Honour refer in any way to the conditions that might be suitably imposed in an ICO on the facts in this case. Without contemplating conditions of this kind, the risk of reoffending cannot have been measured.

The District Court Judge's reasons reveal no assessment of community safety based on whether the risk of reoffending by the specific offender – the appellant – would be better reduced by full-time imprisonment or by an ICO, giving consideration to the appellant's personal circumstances. It cannot be inferred from the reasons that she undertook any such assessment. Her Honour's statement that "[t]here are a substantial number of firearms. The firearms in my view pose a significant risk to the people of Dubbo" does not reveal a consideration of community safety in a forward-looking manner having regard to the appellant's risk of reoffending. In fact, the firearms posed no ongoing risk to community safety whether by future offending conduct on the part of the appellant or anyone else, as they had been seized. As is apparent, the District Court Judge purported to address community safety at the third step in the same manner that it might have been considered in step one or two by observing the safety risk posed by the offending conduct.

The inescapable conclusion is that the District Court Judge failed to undertake the assessment in s 66(2). A further conclusion is that the District Court Judge failed to apprehend that her function at the third stage of the sentencing process required her to assess the risks that the appellant would reoffend, in a manner that might affect community safety, depending upon whether she served her sentence of imprisonment by full-time detention or intensive correction in the community.

A further matter that supports the conclusion that the District Court Judge failed to undertake the assessment in s 66(2), identified by Beech-Jones JA, is the lack of any reference to the circumstances of the offending as a matter bearing upon the appellant's risk of future reoffending. As his Honour put it, the District Court Judge failed to address "whether the [appellant] was a dedicated gun runner or someone caring for five children who just wanted the guns out of her house". As earlier noted, the s 66(2) assessment required consideration, not merely of the appellant's risk of reoffending, but of her risk of reoffending in a manner that might affect community safety. That was a matter that almost certainly required consideration of the likelihood that the appellant would repeat offences of the kind for which she had been convicted. That assessment was not done.

Given the invalidity, there has been no decision on the issue of an ICO at all. As there is a duty to consider whether to grant an ICO in cases where the power is engaged (as it clearly was in this case), this duty remains unperformed. Therefore, the District Court failed to perform its duty and did not determine the appellant's appeal according to law. It was therefore appropriate to set aside the order of the District Court dismissing the appellant's appeal, and order the Court to determine her appeal according to law.

Conclusion:-

Appeal allowed.

Added a post 

Metal Manufactures Pty Limited v Morton [2023] HCA 1 (8 February 2023)

Intro:-

Facts:-

Metal Manufactures Pty Limited ("the appellant") was paid $50,000 and $140,000 by MJ Woodman Electrical Contractors Pty Ltd, a company now in liquidation ("MJ Woodman"). Both payments were made within the six‑month period prior to the winding up of MJ Woodman ("the relation-back period"). The liquidator of MJ Woodman ("the first respondent") sought to recover both payments from the appellant under s 588FF(1)(a) of the Corporations Act 2001 (Cth) ("the Act") on the basis that each was an unfair preference under s 588FA of the Act. The appellant alleges, and the respondents concede, that MJ Woodman owes the appellant $194,727.23. This is a separate and distinct debt from the liability which is said to arise under s 588FF(1)(a). The appellant contends that it has, pursuant to s 553C of the Act, a right to set off its potential liability to repay the alleged unfair preferences against the separate debt owed to it.

Given that the separate debt exceeds the amount of the alleged unfair preferences, if the appellant could set off that debt under s 553C(1), the first respondent would not obtain an order for payment under s 588FF(1)(a). Accordingly, by an Amended Special Case, Derrington J reserved for consideration by the Full Court of the Federal Court the following question:

"Is statutory set-off, under s 553C(1) of the Corporations Act 2001 (Cth) ("Act"), available to the [appellant] in this proceeding against the [first respondent's] claim as liquidator for the recovery of an unfair preference under s 588FA of the Act?"

In a comprehensive set of reasons, the Full Court said that the question posed should be answered "No". In separate reasons, the Full Court also ordered that the issue of costs in the special case be remitted for determination by the docket judge. For the reasons which follow, the answer given by the Full Court was correct, the costs order should not be disturbed, and the costs of this appeal should be costs in the cause.

The statutory scheme

Whether a right of set-off is available requires consideration of the applicable statutory scheme.

Proof and ranking of claims

Division 6 of Pt 5.6 of Ch 5 of the Act deals with the proof and ranking of claims against the company. Section 553(1) is a key provision and is as follows:

"Subject to this Division and Division 8, in every winding up, all debts payable by, and all claims against, the company (present or future, certain or contingent, ascertained or sounding only in damages), being debts or claims the circumstances giving rise to which occurred before the relevant date, are admissible to proof against the company."

Section 553 creates an important cut-off date to determine what debts and claims are provable in the winding up. As Allsop CJ observed below, a critical feature of this provision is that it addresses only debts and claims against the company arising from "circumstances" which had occurred "before the relevant date". Here, the "relevant date" is the date when the winding up of a company is taken because of Div 1A of Pt 5.6 of the Act to have begun. The breadth of the language of s 553 is noteworthy. It extends to all debts payable by and all claims against the company, whether "present or future, certain or contingent, ascertained or sounding only in damages", which arise from "circumstances" before the commencement of the winding up. In contrast, s 82 of the Bankruptcy Act 1966 (Cth), which addresses debts provable in bankruptcy, is limited to debts to which the bankrupt was subject as at the date of bankruptcy, or to which he or she may become subject "by reason of an obligation incurred before the date of the bankruptcy".

The purpose of s 553 is important. As Campbell JA observed in BE Aust WD Pty Ltd v Sutton, s 553 ensures that all legal obligations to which a company is subject are ascertained and then valued "at a common date", so that they can be taken into account in the winding up. Critically, and subject to one possible exception, no debt or claim arising from circumstances arising after the commencement of the winding up of the company is admissible to proof against the company in the liquidation.

Section 555 of the Act provides for the distribution of the assets of the company in accordance with the pari passu principle. It provides that, except as otherwise provided, all debts and claims proved in a winding up rank equally, and that if the property of the company is insufficient to meet such debts and claims, they must be paid proportionately. Many of the exceptions to this principle are set out in s 556 of the Act, which lists a series of payments to be made in priority to all unsecured debts and claims. These include: certain of the liquidator's expenses; the costs of the application for the winding up order; certain amounts owing to employees of the company before the date of winding up; and certain retrenchment payments payable to employees of the company.

Set-off

Section 553C of the Act confers the right of set-off relied upon by the appellant. It should be set out in full:

"Insolvent companies – mutual credit and set-off

(1) Subject to subsection (2), where there have been mutual credits, mutual debts or other mutual dealings between an insolvent company that is being wound up and a person who wants to have a debt or claim admitted against the company:
(a) an account is to be taken of what is due from the one party to the other in respect of those mutual dealings; and

(b) the sum due from the one party is to be set off against any sum due from the other party; and

(c) only the balance of the account is admissible to proof against the company, or is payable to the company, as the case may be.

(2) A person is not entitled under this section to claim the benefit of a set-off if, at the time of giving credit to the company, or at the time of receiving credit from the company, the person had notice of the fact that the company was insolvent."

Voidable transactions

Division 2 of Pt 5.7B of Ch 5 of the Act deals with voidable transactions. The relevant operative provision is s 588FF when read with a series of definitional provisions. It relevantly provides as follows:

"Courts may make orders about voidable transactions

(1) Where, on the application of a company's liquidator, a court is satisfied that a transaction of the company is voidable because of section 588FE, the court may make one or more of the following orders:

(a) an order directing a person to pay to the company an amount equal to some or all of the money that the company has paid under the transaction;
...

(3) An application under subsection (1) may only be made:

(a) during the period beginning on the relation‑back day and ending:

(i) 3 years after the relation‑back day; or

(ii) 12 months after the first appointment of a liquidator in relation to the winding up of the company;
whichever is the later; or

(b) within such longer period as the Court orders on an application under this paragraph made by the liquidator during the paragraph (a) period.

..."

The liquidator's right to seek recovery pursuant to s 588FF of an unfair preference is part of the statutory scheme of liquidation concerned with maximising the distributable pool of assets. It is thus "in the nature of a corollary" to that scheme, including the provisions which prescribe the order of priorities to be observed by the liquidator in the application of the company's property. It is directed at ensuring that the administration of the company is not distorted by putting a debt "ahead of the place appropriate to it in the prescribed order". In this way, the provision achieves its primary objective of "securing equality of distribution amongst creditors of the same class".

Key features of the statutory scheme

Five features of the foregoing statutory scheme of liquidation should be emphasised.

First, the liquidator is given power and responsibility to identify and gather in the assets of the company for distribution to creditors and contributories. Secondly, the liquidator is also obliged to distribute those assets by the making of priority payments and then on a pari passu basis by paying creditors and contributories. Thirdly, a bright line is drawn to enable the liquidator to determine what debts are payable by the company and what claims must be met against it; here it is those arising from "circumstances" which existed "before" the date of winding up. Fourthly, in aid of the duty to gather in the assets of the company, the liquidator may recover preference payments as a debt owed to the company. Finally, in determining what debts are payable and what claims must be met, a set-off must take place between what is due as between the company and another person arising from "mutual credits, mutual debts or other mutual

The appellant's case

The appellant submitted that it was entitled to set off its potential liability said to arise under s 588FF(1)(a) against amounts owing to it by MJ Woodman because there had been a mutual dealing between it and that company. That mutual dealing was said to include the trading transactions which had taken place during the relation-back period for which the appellant had been paid by MJ Woodman – the alleged unfair preferences. The liability under s 588FF(1)(a) will arise because of those voidable transactions; when this liability crystallises, it was said, an account will therefore need to be taken of it as against the remaining amount owed to the appellant for the purposes of s 553C(1). The appellant, in that respect, emphasised that its future liability under s 588FF(1)(a) was no different to any other claim owed to the company precisely because, for the reasons given in Linter Textiles, when it is paid, the company will receive it beneficially.

It was said that it did not matter that, as at the date of the commencement of the winding up, that liability had not yet sprung into existence. It was sufficient, the appellant submitted, that it existed as a contingent liability which might in the future mature into an actual present liability. Relevantly, the essential contingencies were said to be the bringing of an action by the liquidator under s 588FF(1) and the court's satisfaction that there had been a voidable transaction for the purposes of s 588FE of the Act which justified an order obliging the appellant to pay MJ Woodman. All of the facts necessary to make good those contingencies, including satisfaction of ss 588FE, 588FC and 588FA, existed as at the date the company was wound up.

Issue:-

Is statutory set-off, under Section 553C(1) of the Corporations Act 2001 (Cth) ("Act"), available to the [appellant] in this proceeding against the [first respondent's] claim as liquidator for the recovery of an unfair preference under s 588FA of the Act?

Consideration:-

Mutual credits, mutual debts and mutual dealings

The appellant's case turned upon the presence as at the date of the commencement of the winding up of an inchoate or contingent right to sue under s 588FF(1) which was capable of growing or maturing into a money claim that could then be set off against the amount owed by MJ Woodman to it. That proposition suffers from a fatal flaw. Construed in the context of the statutory scheme of liquidation, s 553C(1) requires that the mutual credits, mutual debts or other mutual dealings be credits, debts or dealings arising from circumstances that subsisted in some way or form before the commencement of the winding up. That is because under that statutory scheme, s 553C exists in aid of s 553, which is concerned with debts and claims, whether "present or future, certain or contingent, ascertained or sounding only in damages", arising from "circumstances" that had occurred before the commencement of the winding up. That is why s 553C(1) refers to a "person who wants to have a debt or claim admitted against the company" and then provides that only the balance of any set-off is "admissible to proof against the company, or is payable to the company, as the case may be". As such, the function and purpose of s 553C is to permit a reckoning of amounts owing to and by the company during the relation-back period prior to the appointment of the liquidator.

Here, immediately before the commencement of the winding up there was nothing to set off as between the appellant and MJ Woodman; the company owed money to the appellant, but the appellant owed nothing to the company. Moreover, the inchoate or contingent capacity held by the liquidator to sue under s 588FF could not and did not exist before then. It could only be made following the commencement of the winding up. It was wholly "new" in the sense described by Dixon J in Hiley. It sprang into existence as a specific statutory right held by the liquidator for the purposes of recovering preference payments to secure the equitable distribution of assets amongst creditors. As such, it was not eligible to be set off against the pre-existing amount owed to the appellant.

It follows that the appellant could not identify a relevant mutual dealing. Contrary to its contentions, neither the trade transactions which were undischarged by MJ Woodman during the relation-back period nor, for the reasons already expressed, the discharged trade transactions (giving rise to the liabilities of $50,000 and $140,000), together with the liability which may arise under s 588FF(1)(a), were mutual dealings. Section 553C(1), correctly construed, does not address dealings which straddle the period before and after the commencement of the winding up.

Conclusion

For these reasons, the appeal should be dismissed, with costs in the cause.

Added a post 

Realestate.com.au Pty Ltd v Hardingham; RP Data Pty Limited v Hardingham [2022] HCA 39 (14 December 2022)

Intro:

This is an appeal from the Full Federal Court of Australia.

Facts:-

Mr Hardingham is a professional photographer and the sole director of Real Estate Marketing Australia Pty Ltd ("REMA").
REMA's business has been the supply of photographs taken and floor plans made of residential properties by Mr Hardingham in an editable digital form to real estate agencies for use in the marketing of those properties for sale or lease.

Upon receipt of the photographs and floor plans the commissioning agencies used the images in their marketing in various ways such as in brochures. One of those ways was by uploading them to the realestate.com.au platform operated by Realestate.com.au Pty Ltd (”REA"). The platform of REA is used by a majority of real estate agencies in Australia. REA then provided the images to RP Data Pty Limited, which operates a website and provides a service, RP Data Professional, to which real estate agencies subscribe.

The images provided by REMA to the agencies would appear on RP Data Professional within a few days of upload. The images would remain on REA's platform and RP Data Professional after the completion of the sale or lease of the property the subject of the images. They remained as part of the historical information about completed transactions presumably for purposes such as the assessment of price for future sales.

In proceedings brought in the Federal Court, Mr Hardingham and REMA claimed that RP Data had infringed and continued to infringe the copyright in a large number of the photographs and floor plans. The claims involved a substantial number of such works. With the consent of the parties the Court ordered that questions of liability for infringement of the copyright in the works relating to twenty properties, chosen by Mr Hardingham and REMA, be heard and determined separately from and before the determination of questions ofinfringement of the copyright in other works and questions of relief.

It does not appear to have been in issue in the proceedings that the photographs and floor plans were original artistic works within the meaning of s 32(1) of the Copyright Act 1968 (Cth), or that Mr Hardingham was the author of those works and that he was the owner of the copyright in the works. It appears that Mr Hardingham granted REMA a licence to use and to sub-license the use of the works, at first informally, and then, in April 2018, by way of a formal deed of licence. It was also not in issue in the proceedings that in using the works as it did,RP Data would infringe the copyright in them if it did so without a licence referable to Mr Hardingham or REMA. Attention was necessarily directed to the terms of the licence, which it was not disputed that REMA gave to the agencies, which permitted the agencies in turn to sub-license the use of the works.

The licence issue

The agreements between REMA and the real estate agencies were not in writing. There was no express oral agreement for the grant of a licence by REMA to the agencies in terms which would enable them to license to REA. Nevertheless, Mr Hardingham and REMA knew that the agencies uploaded the images to the REA platform and that it was necessary that they do so. The primary judge found that it was central to the objective of marketing sought to be achieved by all parties.

Consistently with that mutual understanding, Mr Hardingham and REMA's case was conducted on the basis that a licence was granted by REMA to theagencies and that that licence permitted the agencies to grant a sub-licence. But they contended that the licence which the agencies had was subject to a limitation.The limitation was that the images were to be used only for the purpose of marketing the property the subject of the images for a sale or lease. Once a sale or lease of the property was completed, the licence came to an end.

A licence subject to these limitations would not have permitted the agencies to accept the terms of the licence required by REA. The written subscription agreement which REA required the agencies to enter into included a term that, in consideration of REA granting the agency the right to upload listings to its platform, the agency "grant[s] ... an irrevocable, perpetual, world-wide, royalty free licence" to do many things including to license other persons. Clearly enough a term as broad as this would have authorised REA to sub-license to RP Data on terms which included permitting RP Data to maintain the images on its RP Data Professional service after the completion of the sale or lease of the property the
subject of the images.

Decision of Primary Judge

The primary judge found that, objectively viewed, Mr Hardingham, REMA, and the agencies conducted themselves on the basis that the agencies had the right to upload the works to REA's platform in accordance with the terms and conditions required by REA. Mr Hardingham and REMA either knew or assumed that REA was permitted to make the works available after marketing campaigns had ended and the relevant sale and lease transactions had been completed.

His Honour further found that Mr Hardingham and REMA knew that the agencies had to grant REA a licence on REA's terms. They knew that there was an agreement between RP Data and REA by which RP Data was provided with the
content which had been uploaded to the REA platform and that RP Data made those works available to its subscribers.

His Honour held that the objective circumstances relating to the twenty transactions were such that it is either:-

(1) to be inferred from the conduct of the parties including their course of dealings; or

(2) to be implied into the agreements between them, in order to give business efficacy to those agreements, that
Mr Hardingham and REMA agreed that the agencies were authorised, by way of a licence, to upload the images to REA's platform and to grant to REA a licence in the form required by REA.

As mentioned earlier, his Honour considered uploading the works to REA's platform to be central to the objective sought to be achieved by the parties. That objective could not have been achieved unless the agencies could grant a licence to REA on the terms and conditions it usually required.

It followed, his Honour concluded, that Mr Hardingham and REMA authorised, consented to, or permitted (which is to say licensed) the agencies to sub-license the works to REA on REA's usual terms and conditions, which would include authorising REA to grant a sub-licence to RP Data. The sub-licence to RP Data did not go beyond that which was permitted by the sub-licence granted to REA by the agencies. Copyright was not infringed.

Decision of the Full Federal Court

The majority in the Full Court (Greenwood and Rares JJ, Jackson J dissenting) allowed Mr Hardingham and REMA's appeal. Greenwood J (Rares J agreeing) considered that any inference as to the terms upon which the agencies
could grant a sub-licence to REA required actual knowledge of the precise scope of the term. This was necessary because of the gravity of the effect of REA's usual terms and conditions. Their Honours, for reasons which differed, did not consider
that the requirements for the implication of a term were satisfied.

Issue:-

Could it be inferred from the conduct of the parties, or to be implied into the agreements to give business
efficacy to them, that Mr Hardingham and REMA licenced the agencies to upload the works to REA's platform, and in so doing to grant a licence to REA in accordance with REA's standard terms and conditions?

Consideration:-

Ascertainment of terms

In a case such as this where the terms of an agreement between the parties have not been articulated, those terms must be ascertained by reference to the parties' words and conduct. The words and conduct of each party must be understood by reference to what the words and conduct would have led a reasonable person in the position of the other party to believe.The ultimate question is what reasonable people with knowledge of the background circumstances then known to both parties would be taken by their words and conduct to have agreed. In Hawkins v Clayton, in reasoning adopted by Brennan CJ, Dawson and Toohey JJ in Byrne v Australian Airlines Ltd and by Dawson and Toohey JJ in Breen v Williams, Deane J said that the first step in ascertaining what was included in the agreement is one of inference of the actual intention of the parties, taking account of the circumstances disclosed by the evidence. It is only when that first enquiry is complete that consideration might be given, in an appropriate case, to whether a term may be implied as a matter of imputed intention
.
Although Deane J in Hawkins v Clayton used the word "intention", indeed "actual intention", it must be understood as it is used in a contractual context. In Ermogenous v Greek Orthodox Community of SA Inc, it was said that the word "intention" describes what it is that would objectively be conveyed by what was said or done, having regard to the circumstances in which those statements and actions happened. It is not a search for the uncommunicated subjective motives or intentions of the parties. In Pacific Carriers Ltd v BNP Paribas, this Court confirmed the principle of objectivity by which the rights and liabilities of the parties to a contract are determined.

The conditions necessary to ground the implication of a term are well known. Apart from being reasonable and equitable, capable of clear expression and non-contradictory of the express terms of the contract, to be implied a term must be necessary to give business efficacy to the contract (which will not be satisfied if the contract is effective without it), and it must be so obvious that "it goes without saying".
.
In Hospital Products Ltd v United States Surgical Corporation, Deane J cautioned against an over-rigid application of the criteria for the implication of a term. In particular, his Honour said, there should not be such an approach to "business efficacy" where a term otherwise satisfies the requirement that it be "so obvious that it goes without saying". In Hawkins v Clayton, his Honour said that
a term may be implied if it is "necessary for the reasonable or effective operation of a contract of that nature in the circumstances of the case". This general statement was approved in Byrne v Australian Airlines Ltd.

At this point it might be thought that there had been something of a departure from the criterion that a term be obvious. But in Hospital Products, Deane J had clearly stated obviousness to be a criterion and so too had McHugh and Gummow JJ in Byrne v Australian Airlines Ltd. If a resolution of their approach is necessary, Hely J of the Federal Court may be thought to have provided one in Yau's Entertainment Pty Ltd v Asia Television Ltd24, as Jackson J in the Full Court in the present case observed25. Hely J pointed out that it is unlikely that a term which "fails to meet the obviousness criterion would be one which is necessary for the reasonable or effective operation of the contract" (original emphasis).

Inferences and implications

At first instance the issue in this case – what the licence from REMA to the agencies authorised the agencies to agree to when sub-licensing to REA – was approached by reference to two enquiries: what may be said to be inferred from all the circumstances and what may be implied. This may be seen to reflect the approach taken by Deane J in Hawkins v Clayton. There his Honour observed
that there are limits to what may be inferred from all the circumstances, and inferences may overlap with implications. In Breen v Williams, Dawson and Toohey JJ observed that the line between an inference and an implication will not always be easy to draw.

The approach taken by Deane J should not distract attention from the full enquiry as to the rights and liabilities of the parties, which in the first place has regard to their words and conduct – here their conduct in particular – taking account of all the circumstances in which they took place. In focusing attention on the distinction between an inference and an implication his Honour should not be
understood to be limiting that enquiry. His Honour himself said that it was necessary to have regard to the "circumstances disclosed by the evidence". His Honour's discussion of the enquiries there undertaken should be understood as referable to the facts of that case.

The principal question for the Court in Hawkins v Clayton was whether and to what extent a firm of solicitors was obliged to bring to the attention of an executor of a will, and those who may be taken to have had an interest under it, the existence of the will and its contents following the death of the testatrix, who had left the executed will in the custody of the firm.

As to the first enquiry, Deane J observed that it was "obviously" in the contemplation of both the testatrix and the firm that the will might remain in the firm's custody at the time of her death. That being so, his Honour found that one might infer a term by which the firm assumed continuing responsibility for its safe custody. His Honour described as "[c]loser to the borderline between inference and
imputation" a further term that the firm was authorised to communicate the contents of the will to the executor and others having an interest under it when the testatrix died, but appears to have considered it to have been open to draw such an inference.

On the other hand, whether it was a term of the agreement between the firm and the testatrix that when she died the firm was under an obligation to take any positive steps to locate some or all of the persons named in the will, in his Honour's view, was an enquiry beyond the stage of inclusion of terms by inference. His Honour said "[i]t simply cannot be inferred or assumed as a matter of actual fact that the testatrix ever directed her mind to that question or that, if she did, there was any actual joint intention of herself and [the solicitor] which can be expressed as a contractual term". His Honour concluded that a term such as this would have to be implied. But it is to be noted that there was little else in the surrounding circumstances and the dealings of the parties in that case which could be said to have informed their mutual understanding.

The present case

Mr Hardingham and REMA bear the onus of establishing infringement of copyright. Their case for infringement depended upon the scope of the licence given to the agencies, and the sub-licence which the agencies could grant to REA being limited in the way contended for.

Mr Hardingham, REMA, and the agencies dealt with each other in the context of an industry where residential properties were marketed for sale or lease in a particular way. How things were done to achieve this objective forms part of the circumstances in which their conduct is to be considered. This is not to equate what was understood to occur, and the reasons for it, with an industry practice akin to a custom or usage, from which a term might be implied. Rather it is relevant to the question of what might be considered to be the mutual understanding on which they dealt.

Most agencies in Australia conducted their marketing using the REA platform. This was well known. Mr Hardingham, REMA, and the agencies knew that REA uploaded photographs and floor plans of a property to be marketed to its platform and that it then maintained them there after the completion of the sale or lease as available to its subscribers as historical transactions. REA had done so since the platform came into existence in 2003 and over the course of the dealings between the parties. This is hardly consistent with the licence to be given by the agencies to REA being limited in the way contended for.

Within a few days of REA uploading the images they appeared on RP Data's service and remained there post sale or lease. This too must have been apparent to Mr Hardingham and REMA. And prior to their entry into the relevant transactions, Mr Hardingham and REMA understood that RP Data had a contractual relationship with REA by which REA licensed it not only to use the data but in
terms which allowed RP Data to keep the data in its service.

Although Mr Hardingham and REMA may be taken to have understoodwhat had transpired between the agencies, REA, and RP Data in relation to the transactions in question, they said nothing. They made no objection. No question \ of estoppel on the part of Mr Hardingham and REMA has been raised in the proceedings but that is not to say that their silence has no relevance to what may
be taken as conveyed to the agencies. An agreement and its terms may be inferred from the acts and conduct of the parties, including the absence of their words. In light of surrounding circumstances that absence may evidence a tacit understanding

Here, that tacit understanding may be understood to have been evident to the agencies in light of what was taken to be part of the mutual understanding on which the parties conducted their contractual relationships.

The silence of Mr Hardingham and REMA when they well knew what REA and RP Data did with the images, and for how long they continued to use them, is consistent with an acceptance of what was necessary to achieve the intended marketing. Both Mr Hardingham and REMA and the agencies appreciated that it could only be achieved if the agencies submitted to REA's terms to upload the images to its platform. The agencies were no doubt led to believe by the conduct of Mr Hardingham and REMA that they knew and accepted that as a commercial reality. In these circumstances it is not possible to conclude that it was intended that the agencies could only license REA on the basis of the limitation contended for.

In these circumstances no question as to whether a term needs to be implied in the sub-licence to REA arises.

Conclusion and orders

The appeals should be allowed. 

Added a post 

RGCZ v Minister for Immigration, Citizenship, Migrant Services and Multicultural Affairs [2022] FCAFC 201 (14 December 2022)

Intro:-

This is appeal from an order of a primary judge of this Court in which the primary judge upheld a decision of the Administrative Appeals Tribunal to to affirm a decision made by a delegate of the Minister to refuse to grant a partner visa.

Facts:-

The appellant and the three children currently reside in Australia. The eldest child has developmental delays and particular needs. The appellant’s parents, relatives and friends predominantly live in Australia.

The visa applicant arrived in Australia on 6 May 2013 as an unauthorised maritime arrival. On 8 March 2017, the visa applicant applied for a Safe Haven Enterprise Visa, and was granted a bridging visa while that application was considered.

In 2017 the visa applicant was convicted of the offences of recklessly causing injury and contravening a family violence order (the victim being the appellant). He was sentenced to aggregate imprisonment of 42 days, a 12 month community corrections order, and 150 hours of unpaid community work. The visa applicant’s bridging visa was cancelled by reason of his offending, and he was taken into immigration detention.

In June 2018 the visa applicant voluntarily left Australia and returned to Lebanon. The appellant and the three children spent time with the visa applicant in Lebanon before returning to Australia in February 2019. On 11 June 2019, while still offshore, the visa applicant applied for the Partner (Provisional) (Class UF) visa.

On 12 April 2021 the Minister’s delegate refused to grant the visa on the basis that the visa applicant did not pass the character test described in s 501(6) of the Act. The basis on which the visa applicant failed the character test was his 2017 convictions.

On 27 April 2021 the appellant sought merits review of the delegate’s decision in the Tribunal. The appellant was the review applicant in connection with that application because the visa applicant was offshore. Following a hearing, the application was refused on 26 August 2021. The appellant then sought judicial review in the Federal Court on 22 September 2021, and the primary judge dismissed this application on 6 April 2022.

Ministerial Direction No. 90

Before turning to the Tribunal’s reasons and the reasons of the primary judge, it is necessary to have regard to the relevant terms of the Direction, which was given under s 499 of the Act.

Paragraph 5.1(4) of the Direction provides as follows:

The purpose of this Direction is to guide decision-makers in performing functions or exercising powers under section 501 and 501CA of the Act. Under section 499(2A) of the Act, such decision-makers must comply with a direction made under section 499.

Paragraph 5.2 of the Direction sets out a series of principles which provide the framework within which decision‑makers should approach their task of deciding whether to refuse or cancel a non-citizen’s visa under s 501 of the Act, or whether to revoke a mandatory cancellation under s 501CA of the Act. The principles emphasise Australia’s sovereign right to determine whether non-citizens who are of character concern are allowed to enter and/or to remain in Australia. Criminal conduct is expressed to be something which may, in many circumstances, deny a non-citizen the privilege of coming to or remaining in Australia.

Pursuant to Part 2, section 6 of the Direction, which is entitled “Exercising discretion”, a decision-maker, informed by the principles in paragraph 5.2, must take into account the considerations that are identified in sections 8 and 9 of the Direction, where relevant to the decision.

Section 8 of the Direction is entitled “Primary considerations”, the introduction to which provides as follows:

In making a decision under section 501(1), 501(2) or 501CA(4), the following are primary considerations:

(1) protection of the Australian community from criminal or other serious conduct;

(2) whether the conduct engaged in constituted family violence;

(3) the best interests of minor children in Australia;

(4) expectations of the Australian community

(Emphasis added)

Paragraph 8.3 of the Direction is entitled “Best interests of minor children in Australia affected by the decision”, and
provides as follows:

(1) Decision-makers must make a determination about whether cancellation or refusal under section 501, or non-revocation under section 501CA is, or is not, in the best interests of a child affected by the decision.

(2) This consideration applies only if the child is, or would be, under 18 years old at the time when the decision to refuse or cancel the visa, or to not revoke the mandatory cancellation of the visa, is expected to be made.

(3) If there are two or more relevant children, the best interests of each child should be given individual consideration to the extent that their interests may differ.

(4) In considering the best interests of the child, the following factors must be considered where relevant:

a) the nature and duration of the relationship between the child and the non-citizen. Less weight should generally be given where the relationship is non-parental, and/or there is no existing relationship and/or there have been long periods of absence, or limited meaningful contact (including whether an existing Court order restricts contact);

b) the extent to which the non-citizen is likely to play a positive parental role in the future, taking into account the length of time until the child turns 18, and including any Court orders relating to parental access and care arrangements;

c) the impact of the non-citizen's prior conduct, and any likely future conduct, and whether that conduct has, or will have a negative impact on the child;

d) the likely effect that any separation from the non-citizen would have on the child, taking into account the child's or non-citizen's ability to maintain contact in other ways;

e) whether there are other persons who already fulfil a parental role in relation to the child;

f) any known views of the child (with those views being given due weight in accordance with the age and maturity of the child);

g) evidence that the child has been, or is at risk of being, subject to, or exposed to, family violence perpetrated by the non-citizen, or has otherwise been abused or neglected by the non-citizen in any way, whether physically, sexually or mentally;

h) evidence that the child has suffered or experienced any physical or emotional trauma arising from the non-citizen's conduct.

(Emphasis added)

Section 9 of the Direction is concerned with “other considerations” and is not presently relevant.

The Tribunal's Reasons

The appellant sought review in the Tribunal of the delegate’s decision to refuse the visa applicant the visa. It was an important part of the appellant’s case in the Tribunal that the grant of the visa would be in the best interests of the three children.

The Tribunal identified in its reasons the principles that were required to be considered pursuant to paragraph 8.3 of the Direction at [112], and said as follows:

[Paragraph] 8.3 of the Direction requires decision-makers to determine whether revocation is in the best interests of the child. This provision applies only if the child is, or would be, under 18 years old at the time when the application is decided. If there are two or more relevant children, the best interests of each child affected by the decision whether to revoke cancellation of a visa should be given individual consideration, to the extent that their interests may differ.

The Tribunal then set out (at [113]) each of the factors in paragraph 8.3(4) of the Direction which were to be considered, where relevant.

The Tribunal then proceeded (at [114]-[115]) to note the following aspects of the visa applicant’s evidence in light of its obligations pursuant to paragraph 8.3, including:

(a) the interests of the children in the visa applicant resuming a paternal role in their lives, which was said to be most prominent for the eldest child (at [114]);

(b) that the appellant had forgiven the visa applicant for his violence against her, allowed him to return home after release from prison, visited him with the children in custodial settings, and allowed the children to talk on the phone with him multiple times each day, thereby enabling him to remain part of the children’s lives (at [115(a)]);

(c) that the children have a right to a father figure in their lives and his absence deprives them of his physical love and attention which is detrimental to their emotional and psychological wellbeing (at [115(b)]);

(d) the difficulty for the appellant in raising the children in Australia herself without the visa applicant, and the impact of a visa refusal on the appellant’s mental health (at [115(c)]‑[115(d)]);

(e) the hardship on the appellant and the children if the visa was refused, regardless of whether they remained in Australia or if they relocated to Lebanon (at [115(e)]-[115(f)]);

(f) the inadequate, unavailable or affordable childcare services in Lebanon and that the eldest child would not receive state funding for his various problems (at [115(f)]); and

(g) that the appellant would only likely be able to bring the children to Lebanon to see the visa applicant once or twice a year, which would become more difficult once they started school (at [115(g)]).

The Tribunal also noted (at [116]) that it had considered expert medical evidence. This included a psychologist report, a clinical psychologist report, a consultant psychologist report, speech pathologist letters and a list of forthcoming NDIS-funding therapy sessions, substantially in relation to the eldest child. The Tribunal also considered the substance of that evidence.

The Tribunal then assessed the best interests of the children at [119]-[121] of its reasons. At [119] the Tribunal:

(a) made findings favourable to the visa applicant regarding his relationship with his children;

(b) observed that the visa applicant was “concerned” about the appellant’s inability to return to Lebanon, given COVID-19 restrictions and the difficulties confronting her as she raises their three children in Australia without his direct support or any support from her own family, noting that this was particularly so given their eldest child’s development delays and behavioural issues, as well as the general challenges of concurrently raising three children under the age of five; and

(c) accepted evidence from the appellant that she and the children would be assisted by the visa applicant’s direct contribution to the children’s care.

At [120] the Tribunal identified that what the appellant would do if the visa was refused was uncertain, but accepted that, in that instance, the couple faced a “very difficult choice”. The Tribunal observed that if the appellant travelled to Lebanon with the children then some of the concerns about shared parenting and support may be alleviated. However if the appellant remained in Australia without the visa applicant, the Tribunal accepted that the appellant’s mental health may deteriorate and this may adversely affect her parenting and coping ability.

In light of this consideration the Tribunal concluded at [121] that:

This primary consideration weighs substantially in favour of granting the visa if the Review Applicant decided to remain in Australia with her children, and less so if she decided to reunite with the Visa Applicant in Lebanon as she previously intended.

The Tribunal concluded (at [156]):

Having weighed all relevant considerations individually and cumulatively, the Tribunal finds the discretion granted by s 501(1) of the Act should be exercised to refuse the visa. That is because the primary considerations ‘Protection of the Australian community’, Family violence committed by the non-citizen, and ‘Expectations of the Australian community,’ substantially outweigh the combined weight to be given to the primary consideration Best interests of minor children in Australia and the other countervailing considerations.

It followed that the Tribunal affirmed the decision under review.

Primary Judge's decision

The primary judge concluded that the Tribunal’s reasons did not evidence any jurisdictional error in the way in which it approached its task and determined to refuse to grant the visa applicant a visa under s 501(1) of the Migration Act 1958 (Cth). His Honour considered that, contrary to the appellant’s argument, the Tribunal did consider the best interests of each child and the extent to which their interests differed, thereby following the procedure required by paragraph 8.3(3) of Ministerial Direction No. 90 - Visa refusal and cancellation under s 501 and revocation of a mandatory cancellation of a visa under s 501CA.

Issues:-

Did the primary judge err by failing to find that the Tribunal had failed to comply with paragraph 8.3(3) of the Direction, and thus failed to comply with s 499(2) of the Act by:

(a) considering the best interests of the minor children in Australia affected by the decision through the prism of the hypothetical future decisions that might have been made in the event the visa was refused, rather than by reference to whether the best interests of the children would be served by the grant or refusal of the visa; and

(b) failing to consider the differential interests of the children, which existed whether the visa was granted or refused, and which (the appellant contends) were particularly acute if the visa was refused.

Consideration:-

Was there a failure to consider the best interests of the children by reference to the grant or refusal of the visa?

In relation to this aspect of the matter it is the appellant’s case that both paragraphs [119] and [120] of the Tribunal’s reasons refer to, but are not directed to, the interests of the children. It is submitted that paragraph [119] is directed to the interests of the appellant in having support to raise the children, including by reason of the eldest child’s particular needs, and that the Tribunal makes a limited finding as to the interests of the children; namely, that the children would be “assisted by the visa applicant’s direct contribution” to their care. It is further contended by the appellant that paragraph [120] of the Tribunal’s reasons is directed to the choice that the appellant and the visa applicant would be faced with in the event that the decision to refuse the visa was affirmed. It is submitted that paragraph [120] is not directed to whether the best interests of the children would be served by the grant or refusal of the visa.

Referring to the finding in paragraph [121] that this primary consideration weighs substantially in favour of granting the visa if the appellant decided to remain in Australia with the children and less so if they all went to Lebanon, the appellant submits that the question of the best interests of the children should not have been viewed by the Tribunal through the prism of what the appellant would do if the visa was refused. The question, rather, was whether the best interests of the children were served by the grant or refusal of the visa. Those considerations, the appellant submits, were to be weighed by reference to the grant or otherwise of the visa, not by hypothetical considerations of how the appellant might act if the visa was refused.

The appellant contends that the reasons of the Tribunal demonstrated no intellectual engagement with the question of whether the best interests of the children were served by the grant or refusal of the visa outside of the hypothetical circumstances that might arise if the application was refused, and as the conclusion in paragraph [121] of the Tribunal’s reasons makes clear, the question was assessed by reference to what the appellant might do in the event that the visa was refused. This, it is said, was not the question the Tribunal was required to consider.

In support of these submissions the appellant relied on Wan v Minister for Immigration and Multicultural Affairs [2001] FCA 568; (2001) 107 FCR 133 (Branson, North and Stone JJ). The appellant in Wan was married to an Australian citizen and they had two minor children together who were also Australian citizens. The appellant had applied for permanent residence on a general spouse visa which was refused by the Minister’s delegate and by the Tribunal because he was not of good character. An appeal to this Court was dismissed at first instance. The Full Court of this Court allowed the appeal and held that the Tribunal had failed to identify what the best interests of the children required and did not treat their best interests as a primary consideration in its determination (at 141-142 [30]-[31]). Furthermore, the Full Court concluded that the Tribunal had analysed the children’s best interests not by reference to the question of whether the grant or refusal of the visa was in their best interests, but by reference to the hypothetical question of what would the parents have done if there were to have been a grant or a refusal of the visa (at 141 [29]). In Wan this was whether the children would accompany their father to China, or remain in Australia with their mother after their father returned to China.

The appellant drew the Court’s attention to certain paragraphs of the Full Court’s reasoning in Wan explaining the Tribunal’s failure to consider the best interests of the children, as it should have done. Citing Vaitaiki v Minister for Immigration and Ethnic Affairs [1998] FCA 5; (1998) 150 ALR 608, the Full Court in Wan observed:

[23] In Vaitaiki, Burchett J pointed out (at 236; 618) that the question which the Tribunal was required to answer was what the best interest of the children required it to decide with respect to the proposed deportation of their father, not what the children should do given that their father would be deported. ...

...

[25] In Vaitaiki, Branson J accepted (at 249; 630) that the reasons for decision of the Tribunal were to be understood on the basis that the Tribunal did purport to act in conformity with the Convention. However, her Honour observed (at 250; 631):


"If the reasons for decision of the AAT are considered on the above basis, it is noteworthy that nowhere do they seek to identify what would, in the circumstances before the AA T, be the result that would overall be conducive to the best interests of the children. At best they give consideration to the children's best interests in a limited way within a restricted framework. For example, the AAT concluded that the younger children's interests in a 'continuing close and daily relationship with their father' would be served by his being deported in circumstances in which they would accompany him to Tonga. It reached this conclusion without, apparently, having regard to the fact that by so accompanying him they would have to leave the community in which they had lived all of their respective lives, start a new life in a new land, and lose the many benefits available to them as citizens of Australia. As to the older children, the AA T concluded that their best interests would be served by remaining in Australia with their mother, should their father be deported. It reached this conclusion without, apparently, having regard to the break-up of their family unit which would flow from their father's deportation, with consequential restrictions on their subsequent contact with their father and half-siblings and a likely diminution in their father's capacity to influence and guide them.

I conclude, in broad agreement with Burchett J, that although the AAT purported to act on the basis that the best interests of the appellant's children were a primary consideration before it, it did not in fact give proper, genuine and realistic consideration to the children's best interests: Teoh v Minister for Immigration and Ethnic Affairs [1994] FCA 1017; (1994) 49 FCR 409 at 414; [1994] FCA 1017; 121 ALR 436 at 443 per Black CJ; Turner v Minister for Immigration and Ethnic Affairs [1981] FCA 65; (1981) 35 ALR 388 at 392 per Toohey J; Flentjar v Repatriation Commission [1997] FCA 1200; (1997) 26 AAR 93 at 96."

[26] [As to the written reasons of the Tribunal]...[o]f particular significance is the failure of the Tribunal to identify anywhere in its written reasons for decision what the best interests of the children indicated that it should decide with respect to Mr Wan's application for a visa. That this is the starting point for the Tribunal's consideration follows from Teoh and from Vaitaiki. The Tribunal did not refer to either of these cases.

...

[28 ] In giving consideration, as it had done in Vaitaiki, to what the children might do if their father were required to cease living in Australia, the Tribunal was not undertaking an inherently inappropriate task. Such consideration was capable of assisting the Tribunal in determining whether the strength of any other consideration or considerations outweighed the best interests of the children. However, it was not a useful thing to do without the Tribunal having first identified what the best interests of the children indicated should be decided with respect to Mr Wan's visa application.

(Emphasis added)

In substance the appellant submits that, as was the case in Wan (albeit in Wan the obligation to consider the children’s best interests stemmed from the United Nations Convention on the Rights of the Child 1990 and the decision of the High Court in Minister of State for Immigration and Ethnic Affairs v Teoh [1995] HCA 20; (1995) 183 CLR 273), the Tribunal did not make a proper determination for the purposes of paragraph 8.3(1) of the Direction of how the best interests of the children would be affected by the decision whether to grant or refuse the visa. Indeed, the appellant’s position is that the Tribunal’s analysis did not come close to engaging with the best interests of the children. No finding is made as to whether the best interests of the children would support the grant of the visa.

The Minister submits, by way of response, that due consideration was given by the Tribunal to whether the best interests of the children were served by the grant of the visa, having regard to the evidence about the visa applicant’s role in the children’s lives. It is submitted that this was an acknowledgment that the children’s best interests favoured the grant of the visa, which was a factor that the Tribunal considered weighed in favour of granting the visa.

Insofar as the Tribunal considered the weight to be given to this construction on a contingent basis (whether the appellant remained in Australia or returned to Lebanon), the Minister’s submission was, in effect, that nothing turned on this in circumstances where there was some uncertainty about where the appellant and the children might live if the visa was refused. The Minister submits that the Tribunal was only required to give consideration to this claim. It was not required to “resolve” (as to which see EXT20 v Minister for Home Affairs [2022] FCAFC 72 at [120]- [121] (Wigney J) and at [168] (Snaden J)) any uncertainty presented by the relevant evidence and “make actual findings of fact as an adjudication of all material claims made by the applicant”: Minister for Immigration, Citizenship, Migrant Services and Multicultural Affairs v Viane [2021] HCA 41; (2021) 395 ALR 403 at 407 [14]- [15] (Keane, Gordon, Edelman, Steward and Gleeson JJ). Nor was it inappropriate, the Minister submits, for the Tribunal to assess the weight to be given to the children’s best interests by reference to what the appellant and her children might have decided to do: Wan at 140-141 [28]; Tohi v Minister for Immigration, Citizenship, Migrant Services and Multicultural Affairs [2021] FCAFC 125; (2021) 285 FCR 187 at 211-212 [66] (Derrington J) and 241-242 [182]-[191] (O’Bryan J, with whom Katzmann J agreed at 191 [1]).

We accept the appellant’s submission that the best interests of the children were not to be viewed through the prism of what the appellant would do if the visa was refused.

Paragraph 8.3(1) of the Direction required the Tribunal to engage with the question of whether the children’s best interests were served by the grant or refusal of the visa, not by hypothetical considerations of how the appellant might act if the visa was refused. The Tribunal needed to make a determination in relation to this question. Paragraph 8.3(1) of the Direction constituted a mandatory relevant consideration. The use of the word “must” indicates that the Tribunal was positively required to make a determination about whether refusal under s 501 of the Act was, or was not, in the best interests of children affected by the decision.

It is no answer for the Minister to say that the Tribunal has made such a determination by accepting (as it apparently does in the final sentence of paragraph [119]) the appellant’s evidence that she and the children would be assisted by the visa applicant’s direct contribution to the children’s care. We accept that, at most, this finding could be said to have dealt with the best interests of the children in a way that is neutral to the question of whether or not the visa should have been granted.

Notwithstanding the Tribunal’s references to various matters concerning the children, it failed to make a determination about whether refusal of the visa under s 501 of the Act was, or was not, in the best interests of the children. The final sentence in paragraph [119], even when read with paragraph [121] as the Minister submits it should be, cannot fairly be taken to have constituted such a determination. For these reasons the first aspect of the appellant’s ground of appeal succeeds.

Was there a failure to give individual consideration to the differing interests of the children?

By this second aspect of the appellant’s ground of appeal the appellant contends that the Tribunal and the primary judge erred by failing to find that the Tribunal failed to comply with paragraph 8.3(3) of the Direction (and thus with s 499(2) of the Act) by not considering the differential interests of the children. It is submitted that the reasons of the Tribunal do not reveal any consideration of the differential interests of the children and that there was not a separate assessment or determination in relation to the particular interest of the eldest child.

Having determined that the first aspect of ground one succeeds, it is, of course, strictly unnecessary to determine this issue also: Boensch v Pascoe [2019] HCA 49; (2019) 268 CLR 593 at 600-601 [7]- [8] (Kiefel CJ, Gageler and Keane JJ) and 629-630 [101] (Bell, Nettle, Gordon and Edelman JJ). Nonetheless, as we accept that the Tribunal did fail to give individual consideration to the extent that the interests of the children may differ in the decision to grant or refuse the visa, it is appropriate that we say something briefly about this.

The appellant contends that the difference between the interests of the children is stark. The following evidence, as set out in the Tribunal’s reasons, makes it plain that this is so:

(a) the eldest child suffered from “language delays, reduced attention, and behavioural difficulties” for which he receives government funded support and assistance in Australia (at [115(f)]);

(b) the family could not afford to pay for medical care for the eldest child in Lebanon or Australia (at [115(f)]);

(c) a speech pathologist treating the eldest child opined that the child would benefit from being reunited with the visa applicant in Australia, particularly given that he was about to commence school (at [116(e)]);

(d) as the two younger children had spent considerable time in Lebanon as part of a cohesive family unit it may be inferred that they could reasonably adjust to living in Lebanon permanently, and although there was no evidence that the younger children would not thrive in Australia, they would do so without the presence of their father with whom they have a loving and close relationship (at [115]);

(e) the eldest child could not obtain in Lebanon the care provided to him in Australia, and if the family reunited in Lebanon the eldest child would have the considerable benefit of having his father, however, he would lose access to the medical care he needs, with a probable negative impact on his future development and capacity to achieve his maximum potential in life (at [115(f)] and [120]); and

(f) if the family remains in Australia without the presence of the visa applicant, the eldest child would continue to have access to the government funded medical care to which he needs and is entitled but lose the ability to have close contact with his father (at [155(f)] and [120]).

The appellant accepts that the Tribunal understood its obligation to have regard to the best interests of the children, including where those interests differed. However, it is contended that the Tribunal’s reasoning did not weigh the best interests of the children by reference to the individual interests of each child to the extent that they diverged. It is submitted that although all the children’s best interests would be served by the grant of a visa, the eldest child’s best interests were significantly more affected by this outcome. And, while the children’s best interests were all harmed by the refusal of the visa, the eldest child’s best interests were far more significantly harmed by the refusal.

Whilst the appellant accepts that the Tribunal was “aware” of the circumstances of the eldest child, it is submitted that there was no assessment in relation to the eldest child’s interest in particular, nor any determination of any of the children’s interests, and this is indicative of error on the part of the Tribunal which was material.

The Minister contends that there was no relevant failure by the Tribunal for the following reasons:

(a) the Tribunal was not required to consider whether the children’s best interests might diverge because no such argument was put – rather, the appellant’s claim before the Tribunal was that the refusal of the visa would operate more harshly on the eldest child, whatever the family decided to do;

(b) in any event, the primary judge was correct to hold that the Tribunal did consider the best interests of the each child and considered the extent to which their interests may differ; and

(c) the Tribunal was aware of the adverse impact on the eldest child in comparison to the other two children remaining in Australia without the visa applicant, noting that the Tribunal referred to:

(i) the eldest child’s developmental delays and behavioural issues;
(ii) evidence from medical professionals to the effect that the eldest child was being adversely impacted by separation from his father; and

(iii) the developmental support the eldest child needs and receives in Australia.

It is the Minister’s position that the Tribunal recognised that the impact of refusing the visa would operate more harshly on the eldest child, and that there had been due compliance with paragraph 8.3(3) of the Direction.

We accept, for the reasons submitted by the appellant, that although the Tribunal appears to have recognised that the best interests of each child needed to be given individual consideration, there was no explicit consideration of the children’s differential interests. Insofar as the Tribunal noted the particular challenges facing the eldest child, this was insufficient to constitute compliance with the Direction in the absence of a determination about whether the grant or refusal of the visa was, or was not, in the best interests of the children having regard to their differential interests.

For these reasons, the second aspect of the appellant’s ground of appeal also succeeds.

Conclusion:-

The appeal will thus be allowed with costs. The orders made by the primary judge on 6 April 2022 will be set aside and in lieu thereof it will be ordered that:

(a) a writ of certiorari be issued quashing the decision of the second respondent dated 26 August 2021;

(b) a writ of mandamus directed to the second respondent be issued requiring it to review the decision of the delegate of the first respondent made on 12 April 2021 according to law; and

(c) the first respondent pay the appellant’s costs of the proceeding.



 

Added a post 

FSKY v Minister for Immigration, Citizenship and Multicultural Affairs [2023] FCAFC 2 (20 January 2023)

Intro:-

This is appeal from orders of the Federal Court of Australia affirming a decision of the Administrative Appeals Tribunal (Tribunal) not to grant a protection visa by reason of the appellant being a danger to the Australian community.

This appeal concerns the process of reasoning required of a decision-maker when determining whether an applicant for a protection visa is “a danger to the Australian community” under s 36(1C)(b) of the Act. Here, the gravamen of the appellant’s appeal is that it was not rational or logical for the Tribunal to find that he posed the “danger” to the Australian community required under s 36(1C)(b) where the Tribunal accepted that the appellant’s risk of recidivism was “low to moderate”.
.
Facts:-

The appellant was born in 1984 in Cambodia and is a Cambodian citizen. The appellant was orphaned by the age of 12. The appellant and his sisters lived with relatives in Cambodia after his parents had died. In 1997, the appellant’s oldest sister applied for a Class UF Subclass 309 (Provisional Spouse) visa which included the appellant and his younger sister as dependents; this visa was granted on 7 December 1998. In October 1999, the appellant was granted a Spouse (Class BC Subclass 100) visa as a dependent applicant in Australia (dependant spouse visa).

Criminal history

In the period between 2001 and 2015 the appellant was convicted of 131 separate offences and crimes (including drug trafficking, drug use, possessing money and property being the proceeds of crime, possession of heroin/methamphetamine, theft, theft from a motor vehicle, drive under influence of drug, go equipped to steal/cheat, fail to answer bail, theft, obtain property by deception, possession of dangerous article in public place, cultivate narcotic plant, armed robbery) and served 10 terms of imprisonment. In November 2015, the appellant was convicted of Recklessly Causing Serious Injury, Fail Stop Accident, Unlicensed Driving and was sentenced to two years and six months’ imprisonment (non-parole period of 18 months).

Spousal visa cancellation

On 3 February 2017, the appellant’s dependant spouse visa was cancelled pursuant to s 501(3A) of the Act because the appellant had failed to pass the “character test”. This was on the basis that he had a “substantial criminal record” as a result of being sentenced to a term of imprisonment of 12 months or more, which was being served on a full-time basis: ss 501(6)(a) and 501(7)(c) of the Act. Upon completion of his prison sentence, the appellant was transferred to immigration detention.

Protection visa

The appellant first lodged an application for a protection visa on 3 April 2018 and that application was refused on 4 June 2018.
On 15 August 2018, a delegate of the Minister decided not to revoke the appellant’s visa cancellation. On 8 November 2018, the Tribunal affirmed the decision of that delegate to not revoke the cancellation: FSKY and Minister for Home Affairs (Migration) [2018] AATA 4183. The Tribunal found that the appellant was a person in respect of whom Australia has protection obligations as a refugee under s 36(2)(a) of the Act. The Tribunal remitted the protection visa refusal decision to the Department of Immigration and Border Protection (as it then was) on the grounds that the applicant met the criterion in s 36(2)(a) of the Act.

On 15 October 2020, the delegate refused to grant the appellant a protection visa on the basis that he did not meet the criterion in s 36(1C) and was therefore ineligible under s 36(2)(b). On 30 June 2021, the Tribunal affirmed the delegate’s decision to refuse to grant the appellant a protection visa: FSKY and Minister for Immigration, Citizenship, Migrant Services and Multicultural Affairs (Migration) [2021] AATA 2037. It is this decision that was the subject of the relevant review before the primary judge and is the subject of this appeal.

Legislative provision

In order for a person to be eligible for a protection visa, he or she must satisfy the criteria under s 36 of the Act, which provides (only the portions relevant to this appeal are extracted):

(1A) An applicant for a protection visa must satisfy:

(a) both of the criteria in subsections (1B) and (1C); and

(b) at least one of the criteria in subsection (2).

...

(1C) A criterion for a protection visa is that the applicant is not a person whom the Minister considers, on reasonable grounds:

(a) is a danger to Australia’s security; or

(b) having been convicted by a final judgment of a particularly serious crime, is a danger to the Australian community.


Note: For paragraph (b), see section 5M.

(Emphasis added.)

For the purposes of construing the requirements of s 36(1C), a “particularly serious crime” is defined in s 5M of the Act as follows:

For the purposes of the application of this Act and the regulations to a particular person, paragraph 36(1C)(b) has effect as if a reference in that paragraph to a particularly serious crime included a reference to a crime that consists of the commission of:

(a) a serious Australian offence; or

(b) a serious foreign offence.

“[S]erious Australian offence” is defined in s 5 of the Act as follows:

serious Australian offence means an offence against a law in force in Australia, where:

(a) the offence:

(i) involves violence against a person; or

(ii) is a serious drug offence; or

(iii) involves serious damage to property; or

(iv) is an offence against section 197A or 197B (offences relating to immigration detention); and

(b) the offence is punishable by:

(i) imprisonment for life; or

(ii) imprisonment for a fixed term of not less than 3 years; or

(iii) imprisonment for a maximum term of not less than 3 years.
(Emphasis in original.)

The Tribunal’s decision

On 30 June 2021, in its reasons for decision (T), the Tribunal affirmed the delegate’s decision to refuse to grant the appellant a protection visa.

The Tribunal found that the purpose of s 36(1C) of the Act is to give effect in domestic law to Art 33(2) of the Convention relating to the Status of Refugees, opened for signature 28 July 1951, 189 UNTS 137 (entered into force 22 April 1954). Article 33(2) operates as a “compromise” that enables a state to maintain the integrity of its community despite a person being otherwise owed international protection obligations: T[108]–[109].

As to the question of whether a person is a “danger to the community” within the meaning in s 36(1C), the Tribunal found that the determination does not involve the Tribunal balancing considerations, including whether the person has been found to be a refugee. Rather, the Tribunal’s task is to address this question based on what the facts disclose and does not involve the exercise of a discretion: T[110]–[111].

At T[112]–[113], the Tribunal adopted, as a “useful” but not exhaustive list of relevant factors to be considered in determining whether a person constitutes a danger to the Australian community, the factors identified by Tamberlin DP in WKCG and Minister for Immigration and Citizenship [2009] AATA 512 at [26]–[29].

The Tribunal recognised that the presence of violence in past offending constitutes an ingredient in assessing danger, but is not a mandatory one when construing the definition of “serious Australian offence” in s 5 of the Act: T[117].

As to the first element of s 36(1C)(b), the Tribunal found that the appellant had been convicted of a “particularly serious crime”: T[121]–[125].

In respect of the second element of s 36(1C)(b), that is, whether the appellant is a “danger to the Australian community”, the Tribunal said that, although the notion of danger has a logical origin in the person’s previous criminal convictions, such an assessment is a “contemporary exercise”. Citing WGKS v Minister for Immigration, Citizenship, Migrant Services and Multicultural Affairs [2020] FCA 1060 (Davies J), the Tribunal found that it is a factual inquiry, specific to each case, and where the assessment of danger may not necessarily “directly” link to the offending which is just a starting point: T[126].

As part of this factual inquiry, the Tribunal considered the appellant’s risk of recidivism. The Tribunal referred to psychological evidence given to a differently constituted earlier Tribunal in the 2018 proceedings (FSKY and Minister for Home Affairs (Migration) [2018] AATA 4183). That evidence related to the appellant’s “risk of relapse [to heroin use]” and risk of “re-offending” based on the appellant’s (then) submission that his “offending” was “entirely drug-related”. At T[138], the Tribunal said:

When FSKY was before the Tribunal in 2018 in relation to the cancellation of his visa, the learned Member, Mr Maryniak, QC, heard evidence from Ms Elizabeth Warren, psychologist, and stated (T, p 749) at [38] of his decision:

In oral evidence, Ms Warren stated that the Applicant’s risk of relapse was on the ‘cusp’ of low to medium or just at the high end of low close to medium. Therefore, his risk of relapse to heroin use is not insignificant. To the extent that the Applicant maintains that his offending is entirely drug-related, it was submitted by the Respondent that there is at least a low to moderate risk of the Applicant re-offending and exposing the Australian community to more criminal behaviour. The Tribunal accepts this submission and finds accordingly.

(Emphasis in original.)

The Tribunal then considered the evidence given by Mr Simmons, psychologist, in the 2021 Tribunal hearing. In his most recent report, Mr Simmons had written that the appellant was “aware” that “should he relapse” into drug addiction there would be a significant probability of him returning to offending and that he would face issues with respect to his immigration status. However, Mr Simmons noted that should the family support (along with employment) continue and should the appellant remain substance-free, there would be “little likelihood [that] he will commit further offences”: T[139].

The Tribunal noted that, based on his discussions with the appellant, Mr Simmons recorded that the appellant had undertaken “five or six residential drug related programs” as well as other courses, sometimes as a result of a Court order. However, the Tribunal concluded that despite the abatement of the appellant’s offending in 2014 and 2015, none of the other courses appeared to have had much effect on preventing him from returning to re-offending: T[140].

After referring to the conclusion of the earlier Tribunal (extracted at [23] above), the Tribunal made the following conclusion at T[141]:

Because Mr Simmons’ most recent conclusions were qualified by the conditions of FSKY’s knowledge of the potential effect on his migration status and the existence of a supportive family, both of which have not had a material effect in the past, I prefer the conclusion of the previous Tribunal that there is at least a low to moderate risk of FSKY re-offending.

The Tribunal came to the conclusion that the appellant is a danger to the Australian community based on findings that:

(a) The appellant had had a number of opportunities to rehabilitate but has nonetheless re-offended: T[142];

(b) The appellant had demonstrated an unwillingness to take responsibility for his actions when he had hurt people, demonstrated by particular offences, including “the most recent serious offending” in which he had “recklessly” injured his former partner. The Tribunal noted that the appellant’s heroin addiction was not a central factor in that offending as he had not been affected by heroin at the time: T[142];

(c) The prospect of employment with the appellant’s sister was not a protective factor as the appellant did not appear invested in the plan to work at his sister’s business: (T[143];

(d) The provision of stable accommodation with family members was not a protective factor, based on the fact that the appellant had lived with these family members in the past but he nonetheless committed offences: T[144];

(e) The appellant had undertaken several targeted courses to reform his behaviour while he had been in prison but had nonetheless gone on to re-offend: T[145];

(f) Multiple previous warnings given to the appellant about the need to modify his behaviour in order to avoid migration status consequences did not affect his conduct: T[145]; and

(g) Despite the appellant’s “credible” abstinence from drug-taking, this was first “in the context of a custodial and therefore protective environment and while on a methadone treatment regimen” and secondly (as stated above at [27(b)]) his heroin addiction was not a factor in the most recent serious offending: T[145]–[146].

The primary judge’s reasons

Before the primary judge, the appellant advanced the following grounds of review:-

1. The Tribunal’s decision is affected by jurisdictional error in that, in purporting to assess the level of risk that the Applicant posed to the community pursuant to s 36(1C)(b), the Tribunal erred in finding that a “low to moderate risk of reoffending” equated to a “high risk of reoffending” as required by law. In doing so the Tribunal was irrational and illogical in its reasoning, which no rational or logical decision maker could arrive at on the same evidence.

3. The Tribunal’s decision is affected by jurisdictional error in that it erred in not applying the correct legal principle in relation to whether the Applicant is a danger to the Australian community pursuant to s 36(1C)(b). The Tribunal has applied the incorrect legal principles, and pursuant to s 5(1)(f) and (j) of the ADJR Act the decision is ultra vires.

5. The Tribunal’s decision was affected by jurisdictional error when it failed to consider that there has been a particularly long lapse in time since the Applicant last offended and relapsed into heroin use, and that the Applicant was now in a different stage of his life.

6. The Tribunal’s decision was affected by jurisdictional error when it failed to take into consideration the causal link between the Applicant’s risk of reoffending and the Applicant’s heroin use.

The primary judge dealt with grounds 1 and 3 together as they both concerned the Tribunal’s assessment of “danger” in the context of s 36(1C)(b) (see J[57]). With respect to ground 1, the primary judge did not accept the appellant’s submission that the Tribunal’s reasons equated a “low to moderate risk” with the “high level of risk” test. Her Honour held that the “low to moderate risk” finding was made about the appellant’s risk of re-offending in the context of the Tribunal considering Mr Simmons’ evidence, and that this was only one of several factors that the Tribunal considered as part of the overall assessment of danger: at J[71].

With respect to ground 3, the primary judge did not accept the appellant’s submission that the Tribunal incorrectly adopted a standard that “the element of risk must be completely removed”, which was based on the conclusions reached by the Tribunal at T[146]: see J[73]. Her Honour held that the Tribunal was “merely remarking that the bare fact of abstinence from drugs did not remove the risk of re-offending”, and the Tribunal highlighted that the appellant was not affected by heroin at the time of his most recent serious offending: at J[74].

The primary judge also rejected ground 5. Her Honour noted that the Tribunal made references to the appellant’s stage of life, the period of abstinence from drug-use and the period of non-offending: see J[82]–[83]. However, the Tribunal noted (at T[145]) that it was concerned that the appellant had either been in immigration detention or imprisoned during this period of abstinence and non-offending, “thus restricting access to heroin and providing a “protective environment””: at J[83]. As such, it is clear that the Tribunal grappled with the appellant’s submissions and it was open to the Tribunal to reach the conclusions that it did.

Finally, the primary judge held that ground 6 was not made out. Her Honour held that the Tribunal’s reasons clearly demonstrated that it considered Mr Simmons’ evidence as well as the links between the appellant’s offending and his heroin use disorder: see J[88]. The Tribunal ultimately did not accept the submission that most of the appellant’s offending was related to heroin use, and this conclusion was open to the Tribunal: J[89]–[90].

Given none of the appellant’s grounds were made out, her Honour dismissed the application.

Issue:-

Was the Tribunal’s reasons illogical or irrational in finding both that there was a low to moderate risk of recidivism and that the appellant still posed a danger to the Australian community as required under Section 36(1C) Migration Act, 1958 (Cth)?

Consideration:-

The appellant contended that, for the appellant to present as being a “danger”, he must pose a “high level of risk” – adopting the phrase used by Bromberg J in KDSP v Minister for Immigration, Citizenship, Migrant Services and Multicultural Affairs [2020] FCAFC 108; 279 FCR 1 at [54].

On the appellant’s reading of s 36(1C), in the context of its codification of Australia’s obligations under the Convention to protect persons from refoulement and the prospect of indefinite detention if the exception arises, the section constitutes a mandatory power for which a “strictness of logic” must be applied in order for a person to constitute the requisite kind of danger. On the appellant’s submission, the demands of this “strictness of logic” required the decision-maker (here the Tribunal) to attribute “risk” to each of the criteria [or each criterion] that the Tribunal identified and the fact that it did not do so constituted jurisdictional error.

For the reasons which follow, we do not accept that the Tribunal erred in its determination that the appellant posed a danger to the Australian community under s 36(1C) of the Act.

First, the appellant correctly conceded at the hearing that the strictures of s 36(1C) involve a multifactorial assessment of considerations which include those identified by Tamberlin DP in WKCG at [25]–[31].

In WKCG, the question of whether a person was a danger to the Australian community was considered in the context of the exception to a country’s obligation to afford protection to refugees under Art 33(2) of the Convention. Deputy President Tamberlin found that:

(a) The question of whether a person constitutes a “danger to the Australian community” is one of fact and degree. Regard must be had to all of the circumstances of each individual case: WKCG at [25];

(b) The person’s criminal record must be considered as a whole and their prospects of rehabilitation must be assessed: WKCG at [26];

(c) Relevant considerations include the seriousness and nature of the crimes committed, the length of the sentence(s) imposed, and any mitigating and aggravating circumstances. The extent of the criminal history (both nature of prior crimes and the period over which they took place) is also relevant: WKCG at [26];

(d) The risk of re-offending and recidivism and the likelihood of relapsing into crime is a primary consideration, which involves a consideration of the person’s previous general conduct and total criminal history: WKCG at [26] and [27];

(e) The assessment, which includes future conduct, involves a consideration of character and the possibility or probability of any threat which could be posed to a member or members of the Australian community: WGCG at [26];

(f) Once it is found that the person has been convicted of a particularly serious offence, it is then necessary to consider separately whether the person constitutes or is a danger to the Australian community: WKCG at [29];

(g) Whilst the nature and circumstances of the conviction(s) will be highly relevant to the question, it is not conclusive and it will be necessary to look at the person’s conduct in light of all the circumstances that have occurred up to the time of making the tribunal decision: WKCG at [29]; and

(h) It is not necessary to establish that there is a probability of a real and immediate danger of present harm – the provision seeks to protect the community from both immediate harm and harm in the reasonably foreseeable future: WKCG at [31].

Once it was conceded that the required assessment is “multifactorial” we do not understand how the appellant can successfully maintain the position that the Tribunal’s decision was “illogical and unreasonable”.

This is readily apparent from the Tribunal’s reasoning: The Tribunal (at T[112]) extracted the relevant portions of WKCG (and thereafter adopted) its useful non-exhaustive ‘list’ of considerations when assessing whether a person is a ‘danger to the community’, namely – the seriousness and nature of the crimes and the sentences imposed (at T[129]–[134]), mitigating or aggravating circumstances (at T[135]–[136]), the period of offending (at T[137]), and the risk of recidivism (at T[138]–[141]).

Secondly, the appellant’s contention as articulated in his submissions arises from an incorrect reading of Bromberg J’s reasoning in KDSP. In KDSP at [54], his Honour considered what “danger” in the context of s 36(1C) was said to comprise:-

"Section 36(1C) will not be engaged by any risk to Australia whatsoever. It requires a “danger” to Australia – a term suggestive of a high level of risk. In the view reached by Logan J in DOB18 v Minister for Home Affairs [2019] FCAFC 63; (2019) 269 FCR 636, the word “danger” in s 36(1C) means “present and serious risk” (see at [83]). Nor is any past criminality sufficient to engage s 36(1C)(b). To engage that subsection a person must have been convicted by a final judgment of a “particularly serious crime” (as defined by s 5M) and therefore be a “danger to the Australian community”.

The appellant’s contention appeared to assume that his Honour’s reasoning meant that the level of risk to the Australian community which must be found to exist is a “high level of risk [of recidivism]”. The appellant’s counsel rightly conceded at hearing that this is not how “high level of risk” should be read."

Thirdly, it is clear from the Tribunal’s reasons that it did, in fact, undertake the multifactorial assessment it was required to do. Contrary to the appellant’s contention, the Tribunal’s reasons are rational and logical.

The focus of the appellant’s appeal was on one phrase in T[141] – namely that “there is at least a low to moderate risk of [the appellant] re-offending”.

In isolation, this phrase may lead one to pause to consider whether, by reason of such a finding, a person could pose “a danger to the Australian community”. However, care needs to be taken to understand the phrase within the context of the Tribunal’s reasons as a whole.

Relevantly, in the context of adopting the test the appellant contends the Tribunal was required to adopt under KDSP (see T[127]), the Tribunal first considered the seriousness and nature of the crimes and sentences imposed (at T[129]–[134]).

In this context, and departing from the earlier Tribunal’s finding, the Tribunal identified that “some” but (importantly) not all of the appellant’s offending was “driven by his drug-taking”: at T[129]. The Tribunal then found (at T[130]) that despite the fact that the courts had applied a wide range of sentencing and behaviour management tools, the appellant “returned to offending”.

Risk of recidivism

When FSKY was before the Tribunal in 2018 in relation to the cancellation of his visa, the learned Member, Mr Maryniak, QC, heard evidence from Ms Elizabeth Warren, psychologist, and stated (T, p 749) at [38] of his decision:

In oral evidence, Ms Warren stated that the Applicant’s risk of relapse was on the ‘cusp’ of low to medium or just at the high end of low close to medium. Therefore, his risk of relapse to heroin use is not insignificant. To the extent that the Applicant maintains that his offending is entirely drug-related, it was submitted by the Respondent that there is at least a low to moderate risk of the Applicant re-offending and exposing the Australian community to more criminal behaviour. The Tribunal accepts this submission and finds accordingly.

139. Mr Simmons in his most recent report (A2) wrote that FSKY:

"...is aware that should he relapse, then the probability of returning to offending and the problems that would ensue with regard to his immigration status would be significant. He has the support of family and should that continue, along with any employment will go a way towards his remaining substance free....Should FSKY remain substance free, there is little likelihood he will commit further offences.

I note that Mr Simmons also records, based on his discussions with FSKY, that the Applicant has undertaken ‘five or six residential drug related programs’. He has also undertaken a large number of courses, evidence of which was before the Tribunal, both while incarcerated and in the community, sometimes as the result of a Court order. While there was an abating of his offending in 2014 and 2015 up until the incident involving P, none of the other courses appear to have had much of an effect on preventing FSKY returning to re-offending.

Because Mr Simmons’ most recent conclusions were qualified by the conditions of FSKY’s knowledge of the potential effect on his migration status and the existence of a supportive family, both of which have not had a material effect in the past, I prefer the conclusion of the previous Tribunal that there is at least a low to moderate risk of FSKY re-offending.
(Emphasis in original.)

These reasons reveal a number of things: First, the Tribunal accepted the earlier Tribunal’s reasoning as identified at T[138]. Secondly, the earlier Tribunal’s reasoning dealt with both the “risk of relapse” into heroin use which was “not insignificant” and the risk of “re-offending”. Thirdly, the Tribunal then considered the most recent report of Mr Simmons (at T[139] and [140]) and expressed concern that despite the appellant having undertaken courses whilst incarcerated and in the community, “none of the other courses appear to have had much of an effect on preventing [the appellant] from returning to re-offending”. Fourthly, the Tribunal referred to Mr Simmons’ most recent conclusions being qualified by the knowledge of the potential effect on his migration status and the existence of a supportive family – “both of which have not had a material effect in the past” (at T[141]), such that the Tribunal found that “there is at least a low to moderate risk of [the appellant] re-offending” (emphasis added).

Under the ‘heading’ “Conclusion”, the Tribunal went on to draw together the multifarious factors and to undertake the required evaluative exercise.

The Tribunal said the following at T[142]:

The Tribunal is satisfied that FSKY has been convicted by final judgment of a particularly serious crime, within the terms of section 36(1C)(b) of the Act. The Tribunal has carefully considered whether the Applicant is (presently) a danger to the community. Given the number of times FSKY has had the opportunity presented to him to rehabilitate but has nonetheless re-offended, the Tribunal concludes that he is. Of particular concern to me is FSKY’s evidence that, in terms of circumstances of the most recent serious offending where he recklessly injured his former partner, he was not affected by heroin. In his evidence at the hearing, the Applicant said that he had taken ‘some sleeping pills’ during the day. I therefore conclude that his significant heroin addiction problem was not a central factor in that serious offending. It is also concerning to me that the 2015 offence was not the first time FSKY had been convicted of offences of that nature. As set out above, in 2009 he was also convicted of the offence of Fail to Render Assistance after an Accident, and a related offence of Fail to give name and address to injured. Together with the Recklessly causing injury conviction in 2000, I consider there is some element of unwillingness of the Applicant to take responsibility for his actions when he has been directly involved in hurting people, and that is significantly concerning.

(Emphasis in original in italics; emphasis in bold added.)

We note that the Tribunal identified that the appellant is “presently” a danger to the Australian community. Picking up from T[140], the Tribunal noted that, despite numerous opportunities to “rehabilitate”, the appellant has nonetheless re-offended. Consistent with its finding at T[129], the Tribunal considered that the appellant was not affected by heroin when he “recklessly” injured his former partner and thus concluded that “his significant heroin addiction problem was not a central factor in that serious offending”.

The Tribunal then went on to consider the protective factors available upon release (at T[143]–[145]) and to then conclude (at T[146]) as follows:

I am not convinced that FSKY’s creditable abstinence from drug-taking in detention completely removes the element of risk of re-offending, especially as it was not his major vice of heroin that was apparently a factor in his most recent serious offending in 2015 involving the injury to P. Consequently, I find there are reasonable grounds to conclude that FSKY is a danger to the community.

We do not consider that the primary judge erred in holding that the Tribunal’s finding at T[146] was “merely remarking that the bare fact of abstinence from drugs did not remove the risk of re-offending” when the Tribunal highlighted that the appellant was not affected by heroin at the time of his most recent serious offending.

On this basis, the Court does not accept that the Tribunal’s finding with respect to the risk of recidivism at T[141] constrained the Tribunal’s ability to reach the conclusion that it did. Its reasoning was not “irrational” or “illogical”. The finding of a “low to moderate risk” of recidivism will not impede a finding of “danger” – the test of “danger” is multifactorial, and as identified by WKCG and the Tribunal, it involves a complex assessment matrix (see WKCG at [26], approved by Logan J in DOB18 v Minister for Home Affairs [2019] FCAFC 63; 269 FCR 636).

The nature of the past criminal behaviour is relevant, as submitted by the respondent. To isolate the bare probabilities of recidivism as constituting the relevant inquiry demanded by s 36(1C) would constitute error. By way of an illustration provided by the Minister, it would be misleading to describe one turn of a gun barrel in a game of Russian roulette as only exposing the participant to a 16.66% chance of harm (which may be assessed as a low to moderate risk in the abstract). One would, however, describe that exposure to being shot in the head as a “danger” to the person in the firing line. That would be so even if the odds were smaller because while the probability of a bullet emerging from the gun may be low, the consequence of the gun firing a shot to the participant’s head is catastrophic.

Conclusion

For these reasons, the appeal is dismissed with costs.

Added a post 

Allianz Australia Insurance Limited v Delor Vue Apartments CTS 39788 [2022] HCA 38 (14 December 2022)

Intro:

This case involves a dispute between Delor Vue Apartments and Allianz Australia Insurance Limited over an insurance contract. The main issues revolve around whether Allianz can rely on section 28(3) of the Insurance Contracts Act 1984 (Cth), whether Allianz's waiver of its reliance on s 28(3) was irrevocable, and whether Allianz breached its duty of utmost good faith under the Insurance Contracts Act. The court found that Allianz could rely on section 28(3) and that its waiver of reliance on this section was revocable. Additionally, Allianz did not breach its duty of utmost good faith. The appeal was allowed with costs.

Issue:

The case concerns an insurance contract dispute between Delor Vue Apartments and Allianz Australia Insurance Limited. The main issues are whether Allianz can rely on section 28(3) of the Insurance Contracts Act 1984 (Cth), whether Allianz's waiver of its reliance on s 28(3) was irrevocable, and whether Allianz breached its duty of utmost good faith under the Insurance Contracts Act.

Rule:

Section 28(3) of the Insurance Contracts Act 1984 (Cth) operates as a defence to reduce the amount of the insurer's liability by reference to a counterfactual assumption. It does not operate as a condition precedent, extinguishing a corresponding contractual obligation. Therefore, the doctrine of election by affirmation does not apply to make Allianz's waiver of s 28(3) irrevocable.

The duty of utmost good faith in s 13(1) of the Insurance Contracts Act 1984 (Cth) is not a free-standing or independent general duty to act in good faith. Instead, it has two aspects: (i) it is a principle upon which a contract of insurance is based, and thus assists in the recognition of particular implied duties; and (ii) it is an implied condition on existing rights, powers, and duties, governing the manner in which each contracting party must act towards the other party "in respect of any matter arising under or in relation to" the contract of insurance.

Application:

The Court found that Allianz can rely on section 28(3) of the Insurance Contracts Act 1984 (Cth), and that its waiver of reliance on this section was revocable. The doctrine of election by affirmation does not apply to make Allianz's waiver of s 28(3) irrevocable.

The Court also found that Allianz did not breach its duty of utmost good faith. The duty of utmost good faith is not a free-standing or independent general duty to act in good faith. Any assessment of whether the conduct of a party to an insurance contract has breached the duty of utmost good faith, in either of its aspects, requires consideration of the whole of the context of that party's conduct. When the relevant communications between Allianz and Delor Vue are read in context, it is clear that Allianz was not accepting liability for the whole of Delor Vue's claim.

Conclusion:

Allianz's waiver of the defence under s 28(3) of the Insurance Contracts Act was revocable, and was revoked. Delor Vue did not establish that Allianz was precluded from revoking its waiver by reason of "election", "waiver", estoppel, or the duty of utmost good faith. The Court found that Allianz did not breach its duty of utmost good faith. As a result, the appeal was allowed with costs.



Detailed Analysis


Intro:-

This is an appeal from the Full Federal Court of Australia.

Facts:-

A body corporate brings a claim for indemnity under an insurance policy following damage to apartment buildings by a cyclone. The cyclone damage exposes the existence of pre‑existing defects in the apartment buildings which the body corporate had not disclosed to the insurer. Some of those defects need to be repaired concurrently with the cyclone damage. The insurer sends the body corporate an email containing a gratuitous representation that the insurer will grant indemnity despite its power to reduce its liability arising from the body corporate's non‑disclosure. But the email is ambiguous as to the extent of indemnity offered. In particular, the insurer denies liability for defective materials and construction, and requires the body corporate to pay for rectification repairs to the roof, with the scope and costs of those works yet to be determined. The insurer also states that the roof repairs will need to be carried out before internal damage repairs can proceed.

Over the course of the next year, investigations by the insurer reveal further pre‑existing defects. A dispute eventually arises between the insurer and the body corporate. After having incurred nearly $200,000 of costs, the insurer informs the body corporate with greater precision about the extent of its offer to grant indemnity for repairs and replacements, to an estimated cost of around $918,709.90, with other repairs to be undertaken by the body corporate. The insurer informs the body corporate that, unless the body corporate agrees to the proposed terms within 21 days (later extended to more than three months), the insurer will rely on its power not to pay anything due to the non‑disclosure. The body corporate refuses the offer. The insurer denies indemnity.

At trial, the body corporate argued that the insurer was bound by its gratuitous representation that it would grant indemnity because the insurer: (i) had irrevocably elected not to exercise its power to rely on the defence arising from non‑disclosure; (ii) had waived its right to rely on the defence arising from non‑disclosure; (iii) was estopped from resiling from its representation that it would grant indemnity; and (iv) had failed to act with the utmost good faith. In the Federal Court of Australia, the primary judge (Allsop CJ) upheld the body corporate's claims on (ii), (iii), and (iv). A majority of the Full Court of the Federal Court of Australia (McKerracher and Colvin JJ, Derrington J dissenting) dismissed an appeal, finding that all four claims by the body corporate were established.

The appeal to this Court should be allowed. In the law of contract there are limited circumstances in which a gratuitous waiver of rights becomes irrevocable. In this case, where the body corporate did not establish that it had suffered any detriment in reliance upon the insurer's representation, none of those limited circumstances is present. And the insurer did not breach its duty of utmost good faith when, acting lawfully and honestly, it clarified the extent of its offer of indemnity, but required that offer to be accepted for it to waive the defence based on non-disclosure.

The decisions of the primary judge and the Full Court

In conclusions that were not challenged on appeal, the primary judge held that: (i) the failure by Delor Vue to disclose the known defects in the buildings to Allianz prior to entry into the Policy amounted to a breach of Delor Vue's duty of disclosure under s 21(1)(b) of the Insurance Contracts Act; (ii) Allianz (through its agent, SCI) would not have accepted the risk had the disclosure been made by Delor Vue; and (iii) subject to any waiver, estoppel, or failure to act with the utmost good faith, Allianz was entitled, under s 28(3) of the Insurance Contracts Act, to reduce its liability to nil for the claim made by Delor Vue for property damage consequent upon the cyclone.

The primary judge rejected the submission by Delor Vue that Allianz was bound by an election not to rely upon the defence under s 28(3) of the Insurance Contracts Act, but found that Allianz was unable to rely upon s 28(3) for reasons of waiver, estoppel, and the duty of utmost good faith. The primary judge made declarations to that effect. An injunction to "hold the insurer to its stated position" arising from the breach of the duty of utmost good faith would have been ordered but for the making of the declarations.

A majority of the Full Court dismissed an appeal by Allianz. Although their Honours refused to make an additional declaration, McKerracher and Colvin JJ accepted Delor Vue's submissions on its notice of contention to the effect that Allianz was bound by an election not to rely on the defence under s 28(3) of the Insurance Contracts Act. The appeal was otherwise dismissed.

In dissent in the Full Court, Derrington J would have allowed the appeal. His Honour considered that Allianz was not precluded from revoking its promise by any doctrine of election, waiver, or estoppel and that Allianz had not failed to act with the utmost good faith in revoking the waiver of its right to rely on the defence under s 28(3) of the Insurance Contracts Act. For the reasons below, Derrington J was correct.

Issues:-

i) Was Allianz's waiver of the defence under s 28(3) of the Insurance Contracts Act, 1984 (Cth) revocable?

ii) Was Allianz precluded from revoking its waiver by reason of "election", "waiver", estoppel, or the duty of utmost good faith?

Consideration:-

The Insurance Contracts Act: ss 13, 14, 28

Sections 13, 14 and 28 of the Insurance Contracts Act relevantly provide as follows:

"13 The duty of the utmost good faith

(1) A contract of insurance is a contract based on the utmost good faith and there is implied in such a contract a provision requiring each party to it to act towards the other party, in respect of any matter arising under or in relation to it, with the utmost good faith.

(2) A failure by a party to a contract of insurance to comply with the provision implied in the contract by subsection (1) is a breach of the requirements of this Act.

...

14 Parties not to rely on provisions except in the utmost good faith

(1) If reliance by a party to a contract of insurance on a provision of the contract would be to fail to act with the utmost good faith, the party may not rely on the provision.

(2) Subsection (1) does not limit the operation of section 13.

...

28 General insurance

(1) This section applies where the person who became the insured under a contract of general insurance upon the contract being entered into:
(a) failed to comply with the duty of disclosure; or
(b) made a misrepresentation to the insurer before the contract was entered into;

 

but does not apply where the insurer would have entered into the contract, for the same premium and on the same terms and conditions, even if the insured had not failed to comply with the duty of disclosure or had not made the misrepresentation before the contract was entered into.

(2) If the failure was fraudulent or the misrepresentation was made fraudulently, the insurer may avoid the contract.

(3) If the insurer is not entitled to avoid the contract or, being entitled to avoid the contract (whether under subsection (2) or otherwise) has not done so, the liability of the insurer in respect of a claim is reduced to the amount that would place the insurer in a position in which the insurer would have been if the failure had not occurred or the misrepresentation had not been made."
Allianz's waiver of the s 28(3) defence

The nature of an irrevocable waiver

It has been repeatedly said that "waiver" is a term that is used in many different senses. Perhaps the most common usage of waiver is to describe an unequivocal decision by a party, communicated to the other party, not to insist upon a right or not to exercise a power.

By itself, a waiver of a right is rarely irrevocable. For that reason, it has sometimes been said that the general rule concerning a waiver of a right, "in the sense of an intimation of an intention not to enforce it", is that the mere act of representing that a right has been waived is "of itself inoperative". Similarly, it has been said that "the mere statement of an intention not to insist on a right is not effectual unless made for consideration ... A mere waiver signifies nothing more than an expression of intention not to insist upon the right". Perhaps more accurately, the legal position is that although a waiver does have legal effect in that "the waiver is binding on the waiving party, unless the waiver is effectively retracted", the waiver can generally be revoked at any time with reasonable notice.

Nevertheless, exceptions or "special cases" exist where a unilateral waiver cannot be revoked. One exception is where the strength of the interest of finality in litigation can sometimes mean that a waiver of particular rights related to litigation is irrevocable. For instance, the waiver of legal professional privilege will be irrevocable "where the actions of a party are plainly inconsistent with the maintenance of the confidentiality which the privilege is intended to protect". A similar approach, which also "depended upon considerations founded in the nature of the adversarial litigious process" that are "not relevant to the identification of the rights and obligations of parties to contracts", was taken in The Commonwealth v Verwayen by Toohey J and Gaudron J. Their Honours concluded that an undertaking not to plead a limitations defence had become irrevocable. But, even then, that view did not command the support of a majority of the Court. Brennan J, by contrast, considered that the "ordinary principles of estoppel" applied to the waiver in that case so that it could be revoked at any time before it had been relied upon to the detriment of the other party, or otherwise until judgment was entered so that no amendment to the pleading was possible.

Outside the context of litigation, and in the law of contract, the circumstances in which a waiver cannot be revoked have always been exceptional. If such circumstances were not both exceptional and justified they would undermine other contractual rules, including those generally requiring that variation of a contract be in the form of a deed or supported by consideration. Hence, aside from circumstances where a legal right can no longer be enforced due to entry into a deed, a fresh agreement for consideration, or expiry of a limitation period, the general rule is that, despite a "mere naked promise ... not founded upon any consideration" not to enforce a legal right, the legal right may continue to be enforced until it is fully satisfied.

For the same reasons, the development of loose legal rules for an irrevocable waiver would undermine formalities where they are required for written contracts. Indeed, writing after the decision of Denning J in Central London Property Trust Ltd v High Trees House Ltd, Cheshire and Fifoot observed that "in their efforts to circumvent this objection ... the courts have excelled themselves in ingenuity, if not in wisdom".

Consistently with the stance of parties in previous litigation concerning waiver in this Court, Delor Vue properly did not, at any stage in this litigation, submit that there was any independent doctrine precluding revocation of a waiver based on concepts such as "unfairness" or based on any assertion of notions of waiver peculiar to insurance contracts. Such submissions would require revision of our understanding of basic principles of contract, even if confined to insurance contracts. At the very least, such an approach should not be taken by a court without argument. Delor Vue's submissions were more modest but, if accepted, would nevertheless undermine the integrity of established contractual rules by expanding the principles of election by affirmation, or extinguishment of rights, in such a way as to make irrevocable a unilateral waiver of a defence to liability by a party to a contract, outside the context of litigation.

The waiver and its revocation by Allianz

As described above, the extent to which Allianz undertook to grant indemnity in the 9 May 2017 email was ambiguous. Allianz undertook in the email to cover the costs associated with "[r]esultant damage including but not limited to internal water damage, fascia, guttering and roof sheeting (for those buildings which lost roof sheeting only)" but not for the overlapping category of "[d]efective materials and construction of the roof". Nevertheless, Allianz made no submission in this Court that the ambiguities in its 9 May 2017 email, and the lack of any reference to s 28(3) of the Insurance Contracts Act, precluded an interpretation of that email as containing an unequivocal waiver of Allianz's defence under s 28(3).

Putting aside its ambiguities, and on the assumption that the 9 May 2017 email contained a waiver of Allianz's defence under s 28(3), Allianz did not express the extent of the defence that would otherwise have applied. It did not express in the 9 May 2017 email, and could not have been certain of, the extent to which it would have been entitled under s 28(3) to reduce its liability in respect of Delor Vue's claim.

Although the primary judge made a carefully worded declaration that Allianz was entitled to a remedy that would reduce its liability to nil "for the claim made consequent on damage caused to [Delor Vue's] property", this did not mean that Allianz was free from all liability to Delor Vue. Assuming that Allianz was entitled to reduce its liability under s 28(3) on the basis that it would not have issued any policy if the disclosure had been made, it may be that the "amount which would place the insurer in the position it would have been in" but for the non‑disclosure would have required Allianz to have refunded to Delor Vue all premiums paid by Delor Vue.

It is not entirely accurate to describe the waiver by Allianz as having been revoked by Allianz's letter to Delor Vue's solicitors on 28 May 2018. In that letter, Allianz undertook to grant indemnity, subject to conditions, for estimated costs of $918,709.90. The only sense in which Allianz could be said to have "revoked" its waiver on 28 May 2018 was that the continued operation of the waiver was made conditional upon acceptance of terms, in order to resolve the dispute between the parties, within a reasonable time (21 days, later extended to more than three months). It is only in that sense that the waiver can be described as having been revoked.

Election by affirmation cannot be applied to Allianz's waiver

By its proposed notice of contention in this Court, Delor Vue sought leave to support the conclusion of the majority of the Full Court on the basis that Allianz's representation that it would not rely on s 28(3) of the Insurance Contracts Act was a choice between alternative and inconsistent sets of rights. Delor Vue should be given leave to file its notice of contention but its submission that the doctrine of election by affirmation applied to make Allianz's waiver irrevocable is, nevertheless, unsustainable.

As to the historical application of election by affirmation, s 28(3) does not operate to make disclosure by the insured a condition precedent to any obligation of the insurer. Different views about the operation of s 28(3) have been expressed[60]. On one view, s 28(3) operates on the basis of the existence of an insurance policy by reference to the additional premium that would have been charged if there had been full disclosure. On another view, s 28(3) can operate on the basis that the insurer would not have accepted the policy at all, so that liability may be reduced as low as the amount of the premium paid. On either view, however, s 28(3) operates only as a defence to reduce the amount of the insurer's liability by reference to a counterfactual assumption. It does not operate as a condition precedent, extinguishing a corresponding contractual obligation. It is therefore unnecessary to consider whether, in the modern law, a waiver of a condition precedent is irrevocable without detrimental reliance.

As to the modern approach to election by affirmation, s 28(3) does not give the insurer any power to elect to affirm the contract rather than to avoid or terminate its contractual obligations. There is no sense in which a decision by an insurer to waive the defence under s 28(3) involves an election between alternative and inconsistent sets of rights (or even an immediate inconsistency between continuing legal positions). With or without waiver, the insurance contract remains on foot and reliance on the defence under s 28(3) is not immediately inconsistent with any of the contractual rights. In its operation in relation to rights, s 28(3) stands in stark contrast with s 28(2), which is a statutory recognition of the power of an insurer to avoid a contract from its inception for a fraudulent non‑disclosure or a fraudulent misrepresentation. An insurer that elects to waive the power under s 28(2) elects to affirm the set of continuing rights under the relevant contract of insurance rather than to exercise the immediately inconsistent power to avoid the contract from inception.

Indeed, the submissions of Delor Vue and the decision of the majority of the Full Court to the contrary are directly inconsistent with the reasoning and unanimous result in this Court in Gardiner. Although numerous facts were disputed in that case, Gummow, Hayne and Kiefel JJ proceeded on the assumption that a lender and an indemnifier had represented to an indemnified party that the indemnity "remained effective and enforceable, despite past defaults". The defaults concerned failures to make punctual performance under separate agreements with the lender. Despite that representation, their Honours said that "there was no election between inconsistent rights" and that to hold the lender and indemnifier to the representation would "supplant accepted principles governing whether an estoppel is established and whether a contract has been varied".

The submission of Delor Vue that Allianz had irrevocably elected not to rely on the defence under s 28(3) can be aptly expressed in the words of Rix LJ in Kosmar Villa Holidays plc v Trustees of Syndicate 1243[68] as a submission that "goes far wider than the doctrine of election has ever been previously explained or applied":-

"While a contract is in operation, it is important to know, in circumstances where it lies in the choice of a party, whether the contract lives or dies (or at least whether purported performance under it, such as a delivery of goods, is accepted or not); and, whether the option is for life or death, acceptance or rejection, the choice is unilateral and irrevocable. But when it is merely a defence to a claim that is in question, there would not seem to be the same necessity to choose timeously and irrevocably between reliance or not on the defence in question."

Allianz's duty of utmost good faith

The nature of the duty of utmost good faith

Section 13(1) of the Insurance Contracts Act, set out earlier in these reasons, is an instantiation of the centuries‑old common law "duty of utmost good faith" in commercial contracts. Like the common law duty, the duty in s 13(1) is not a free‑standing or "independent general duty to act in good faith". Rather, as s 13(1) provides, the duty has two aspects: (i) it is a principle upon which a contract of insurance is "based" and thus assists in the recognition of particular implied duties; and (ii) it is an implied condition on existing rights, powers, and duties, governing the manner in which each contracting party must act towards the other party "in respect of any matter arising under or in relation to" the contract of insurance.

Each of these two aspects of the duty of utmost good faith applies equally to the insurer and to the insured. Indeed, it has long been recognised that the duty of utmost good faith applies symmetrically to both parties to an insurance contract. This symmetrical operation was generally incorporated into s 13(1) of the Insurance Contracts Act.

The first aspect of the duty of utmost good faith, as the principle on which the contract of insurance is based, requires various implied duties to be recognised. The most widely recognised of these is the duty of full disclosure. As early as 1766, Lord Mansfield said in the insurance context in Carter v Boehm, albeit in remarks intended to apply to all contracts, that "[g]ood faith forbids either party by concealing what [they] privately know[], to draw the other into a bargain" where the other is ignorant of the concealed fact. The duty of disclosure by an insured is now the subject of a detailed statutory regime in Divs 1 and 3 of Pt IV of the Insurance Contracts Act.

The second aspect of the duty of utmost good faith, as an implied condition, requires each party "to have regard to more than its own interests when exercising its rights and powers under the contract of insurance". This condition upon the exercise of rights and powers and the performance of obligations is not fiduciary. It does not require a party to an insurance contract to exercise rights or powers or to perform obligations only in the interests of the other party. But nor is the condition limited to honest performance. The duty to act honestly, or not deceitfully, has been said to be "a duty of universal obligation". Section 13(1) would add nothing to the conditions on the exercise of those contractual rights and powers, and the performance of obligations, if it merely required the exercise or performance to be honest.

It has therefore been said that rights and powers must be exercised, and duties must be performed, "consistently with commercial standards of decency and fairness" as distinct from standards of decency and fairness more generally. Several examples can be given of how the duty of utmost good faith conditions the exercise of contractual rights and powers and the performance of obligations. The refusal to cooperate with another contractual party in the exercise of a power can involve a lack of utmost good faith. The failure, "within a reasonable time of the receipt of the claim", to perform the obligation to accept or refuse a claim can involve a lack of utmost good faith. And, as s 14 of the Insurance Contracts Act provides, reliance upon a power specifically provided to one party in the contract will be precluded if the reliance would involve a lack of utmost good faith.

Allianz did not breach its duty of utmost good faith

By whichever approach this novel duty of a party to an insurance contract is sought to be derived, it cannot be accepted. It is not fatal to the existence of this novel duty that Delor Vue was unable to point to a single case identifying a remotely similar duty over the period of more than 250 years since a duty of utmost good faith in insurance contracts was recognised. Nor is it fatal that the Australian Law Reform Commission did not contemplate anything like it in the report which formed the basis of the Insurance Contracts Act[132]. But, in a context in which insurers have been operating for nearly 40 years on the basis of a particular understanding of the operation of the Insurance Contracts Act, these matters are not a promising start.

What is fatal to the recognition of this novel duty is that it would not be coherent either with the operation of existing legal doctrines, whose existence was well established at the time of the Insurance Contracts Act, or with the Insurance Contracts Act itself. In relation to insurers, it would have the effect of subsuming much of the operation of the doctrines of election, waiver, and estoppel into a broader positive duty not to unreasonably depart from significant representations. No reliance or detriment would be required.

The recognition of such a duty would also have radical consequences for an insured that would not be coherent with the generally symmetrical operation of the Insurance Contracts Act. Div 2 of Pt IV of the Insurance Contracts Act is concerned with misrepresentations by an insured. Suppose that, following the occurrence of an insured event, an insured party, carelessly assuming that the damage was minimal, made a representation to their insurer that no claim would be brought under the policy. The factual aspect of that representation – the "state of affairs" – being the present state of mind of the insured party[133] would not be a misrepresentation by the insured within s 24 of the Insurance Contracts Act. But if it were a breach of the duty of utmost good faith for the insured to depart unreasonably from their representation concerning a claim, then the insurer could cancel the contract under s 60(1)(a) if a claim was subsequently brought by the insured.

These matters are sufficient to conclude that there is no basis to find that Allianz breached its duty of utmost good faith by imposing conditions upon its representation that it would not rely on s 28(3) of the Insurance Contracts Act. In any event, however, even if there were a duty of the kind suggested by the primary judge or by Delor Vue, that duty would not have been breached by Allianz.

Delor Vue's submission that Allianz's conduct amounted to a breach of a duty not to resile, without a reasonable basis, from its representation is based on the premise that it is possible to fillet the representation by Allianz that it would not rely on s 28(3) of the Insurance Contracts Act from the remainder of the 9 May 2017 email. That premise is incorrect. Any assessment of whether the conduct of a party to an insurance contract has breached the duty of utmost good faith, in either of its aspects, requires consideration of the whole of the context of that party's conduct.

When the representation in the 9 May 2017 email is read in its full context, it is clear that Allianz was not accepting liability for the whole of Delor Vue's claim. Allianz's representation that it would not rely on s 28(3) was inseparable from Allianz's limited offer of indemnity that excluded "[d]efective materials and construction of the roof, including but not limited to tie downs, rafters and timbers and soffit" and required Delor Vue to pay for roof repairs of a scope yet to be defined, but to be undertaken prior to internal repairs.

And when the representation in the 28 May 2018 letter is read in its full context, which included almost a year of dispute about the terms of Allianz's limited offer of indemnity, it is clear that Allianz was endeavouring to give more detailed and precise content to the terms of its 9 May 2017 email (which were set out in full). That included greater precision about the repairs and replacements that Allianz would undertake, to an estimated cost of around $918,709.90, in addition to costs of nearly $200,000 that it had already incurred.

When the 9 May 2017 email and the 28 May 2018 letter are both read in context, the appropriate characterisation of the 28 May 2018 letter, in the words of Derrington J in dissent in the Full Court, is that Allianz was giving content to its offer "to pay a large gratuitous amount in respect of a liability which did not exist", albeit with a limited time for acceptance (ultimately, around three months). Even if the novel duty proposed by Delor Vue were accepted, the 28 May 2018 letter could not be a breach of the duty of utmost good faith.

For these reasons, Allianz did not breach its duty of utmost good faith. It is, therefore, unnecessary to consider any of Allianz's submissions concerning the utility or availability of a declaration if a breach had occurred, including submissions that Delor Vue: (i) did not seek any declaration of a breach of s 13; (ii) sought only damages for breach of s 13, in place of which a declaration was made; (iii) led no evidence of any consequential loss suffered from the alleged breach of the duty of utmost good faith and obtained no award of damages from the primary judge; and (iv) did not seek or obtain any injunction to enforce the duty alleged to have been breached.

Conclusion:-

Allianz's waiver of the defence under s 28(3) of the Insurance Contracts Act was revocable and was revoked. Delor Vue did not establish that Allianz was precluded from revoking its waiver by reason of "election", "waiver", estoppel, or the duty of utmost good faith. Orders should be made as follows:

1. The appeal be allowed with costs.

Added a post 

Electricity Networks Corporation v Herridge Parties [2022] HCA 37 (7 December 2022)

Intro:-

This is an appeal from the Supreme Court of Western Australia relating to claims made by a number of plaintiffs for loss and damage resulting from a bushfire in Parkerville, Western Australia. The fire was caused when a jarrah pole on the land of the fourth respondent, to which the electrical cable and other apparatus of Electricity Networks Corporation (trading as Western Power) were attached, fell to the ground and ignited dry vegetation. The main issue in the case was whether Western Power owed a common law duty of care to persons in the vicinity of its electricity distribution system to take reasonable care to avoid or minimise the risk of injury to those persons and damage to their property. The trial judge found that Western Power owed a duty to take reasonable care, but that it had discharged that duty. The Court of Appeal upheld the trial judge's finding that Western Power owed a duty of care, but held that Western Power had breached that duty by failing to have a system for the periodic inspection of wooden PA poles. The High Court of Australia will determine the applicable principles for determining the existence of a common law duty of care allegedly owed by a statutory authority and the content of the broader duty imposed on Western Power.

Issue: The issue in this case is whether Western Power, a statutory authority responsible for operating and maintaining the electricity distribution system, owed a duty of care to prevent the risk of harm from the ignition and spread of fire in connection with the delivery of electricity through its electricity distribution system to a property owner, Mrs. Campbell.

Rule: There is no freestanding common law rule which fixes whether and when a common law duty of care upon a statutory authority might, or might not, arise. The starting point for analysis of any common law duty of care that might be owed by any statutory authority must always be the particular statutory framework within which the statutory authority operates. When a statutory authority has entered into the exercise of its statutory powers, the question is whether the relationship between the statutory authority and a class of persons affected by the manner of exercise of the power was such as to give rise to a duty of care. The common law imposes a duty in tort which operates alongside the rights, duties and liabilities created by statute. A duty cannot arise where it would be inconsistent or incompatible with the statutory powers or duties imposed on the statutory authority or it would be incoherent with the statutory framework.

Application: Western Power had stepped into the arena and exercised specific statutory powers in performing its statutory functions of undertaking, operating, managing, and maintaining the electricity distribution system, which was essential for providing electricity distribution services. Western Power had connected Mrs. Campbell's premises to its distribution system and energized those premises. Western Power exercised those powers continuously, creating a relationship between it and all other persons within the vicinity of its electricity distribution system. The critical feature of that relationship was that Western Power exercised those powers in a manner which created or increased the risk of harm to those persons. The common law imposed a duty in tort on Western Power, which operated alongside the rights, duties, and liabilities created by statute.

Conclusion: Western Power had a duty to take reasonable care in the exercise of its powers, and the content of that duty required it to avoid or minimize the risk of injury to those persons, and loss or damage to their property, from the ignition and spread of fire in connection with the delivery of electricity through its electricity distribution system. The broader duty was not inconsistent or incompatible with the statutory functions and powers imposed on it. Western Power had ample power to discharge its duty of care. The application for special leave to cross-appeal filed by the Herridge Parties and the IAG/Allianz Parties was refused with costs, and the appeal was dismissed with costs.

Detailed Analysis

Facts:-

This appeal arises from claims made by a large number of plaintiffs for loss and damage resulting from a bushfire in Parkerville, Western Australia. The fire started on 12 January 2014 when a jarrah pole on the land of the fourth respondent ("Mrs Campbell"), to which the electrical cable and other apparatus of the appellant, Electricity Networks Corporation (which traded as Western Power) ("Western Power"), were attached, fell to the ground causing electrical arcing and igniting dry vegetation around the base of the pole. The pole is referred to as a "point of attachment pole" or "PA pole", being a pole at which an electricity distribution system is attached to the consumer mains.

The PA pole that fell was installed on Mrs Campbell's property by her late husband and was in place since at least 1983. It was made of jarrah and was 5.5m tall above the ground and embedded 1.1m into the ground. At the time of the fire, it was approximately 21cm in diameter at ground level tapering to 16cm in diameter at the tip. The PA pole failed below the ground line due to fungal decay and damage by termites.

Western Power operated the electricity distribution system called the South West interconnected system ("SWIS"), which was used to deliver electricity to Mrs Campbell's property. The link by which the distribution system delivers electricity to individual consumers is called a "service cable" when above ground. The service cable is owned by Western Power. In suburban areas, the service cable typically runs from the nearest distribution pole to the front eave of the consumer's house. In regional and subregional areas, the service cable often runs from the nearest network distribution pole (a "termination pole") across the property boundary to a privately owned PA pole. From there, electricity will run to the consumer's residence, either underground or above ground.

The fifth respondent, Ventia Utility Services Pty Ltd (formerly known as Thiess Services Ltd) ("Thiess"), carried on a business of, among other things, constructing and maintaining electricity distribution system installations and was contracted by Western Power to construct, maintain and manage aspects of Western Power's distribution system. The relationship between Western Power and Thiess was one of principal and independent contractor. Thiess was contracted by Western Power to undertake works in the vicinity of Mrs Campbell's PA pole in July 2013 ("the July 2013 works"). Those works, part of a broader series of works, included replacing Western Power's termination pole adjacent to Mrs Campbell's property from which its service cable ran across her property boundary to the PA pole. Replacement of the termination pole required removing and replacing the service cable between the termination pole and the PA pole.

Trial judge's decision

The trial judge found that industry practice required steps to be taken before performing works like the July 2013 works, including inspecting and sounding (striking with a hammer, axe or solid bar) the PA pole to identify signs of deterioration, as well as digging around the base of the pole to allow detection of one or both of surface decay and termite attack in the below ground critical zone. Thiess' leading hand did not perform a sounding test on the PA pole in accordance with industry standards: he did not perform the necessary hammer test adequately and he did not adequately dig around the base of the PA pole.

The main issue at trial was the defendants' liability, if any, for the failure and collapse of the PA pole and the subsequent fire and damage. The trial judge found Thiess and Mrs Campbell liable to the plaintiffs in negligence and nuisance, and apportioned liability between them as 70 per cent to Thiess and 30 per cent to Mrs Campbell.

All claims against Western Power were dismissed. The trial judge found Western Power owed the plaintiffs a duty of care – the "pre-work inspection duty of care": that before undertaking works on the PA pole and when undertaking those works, Western Power had a duty to take reasonable care to inspect the PA pole to ascertain whether it was in a safe and fit condition for use in the supply of electricity and if, when undertaking a pre-work inspection, or when undertaking works on the PA pole, Western Power identified that the PA pole was not in a safe and fit condition for use in the supply of electricity, a duty not to use the PA pole in or in connection with the supply of electricity. The trial judge, however, rejected the contention that Western Power had breached that duty of care by failing to supervise Thiess' line crew or otherwise ensure that the line crew inspected the PA pole in accordance with industry practice and by failing to implement systems for training or instructing the line crews to conduct pre‑work pole inspections in accordance with industry practice.

The trial judge concluded that Western Power had taken reasonable precautions to ensure that qualified and competent personnel carried out the work, including the pre-work inspections of wooden poles, by retaining a competent, reputable and experienced contractor, namely Thiess, to carry out the work under a contract which required Thiess to engage personnel to perform the work who were competent, had all the necessary skills, training and qualifications to carry out the work in accordance with the contract, and had been inducted by Thiess and were able to perform the work without the supervision of Western Power's personnel. In sum, the trial judge found Western Power had discharged its pre‑work inspection duty of care by engaging and instructing Thiess to carry out the relevant work, including the inspection of the PA pole. The trial judge was not satisfied that a reasonable person in the position of Western Power would have taken any additional steps to implement systems for training or instructing line crews to conduct pre-work pole inspections in accordance with industry practice. The trial judge rejected the plaintiffs' contention that the pre-work inspection duty of care was non‑delegable.

Court of Appeal's decision

All parties, other than Western Power, appealed or cross‑appealed to the Court of Appeal of the Supreme Court of Western Australia. Western Power did not appeal the trial judge's finding that it owed the plaintiffs the pre‑work inspection duty of care. The reasons for decision of the Court of Appeal were detailed and addressed many issues. It is necessary to deal only with those aspects of the Court of Appeal's reasons relevant to Western Power's appeal to this Court and to the applications for special leave to cross‑appeal filed in this Court by the Herridge Parties and the IAG/Allianz Parties in respect of whether the pre-work inspection duty of care was non-delegable.

First, the Court of Appeal held that Western Power owed to persons in the vicinity of its electricity distribution system a duty to take reasonable care to avoid or minimise the risk of injury to those persons, and loss or damage to their property, from the ignition and spread of fire in connection with the delivery of electricity through its electricity distribution system. That duty of care was broader than the duty found by the trial judge in two ways: it was broader in its temporal scope and it was not limited to occasions when work was to be, or was being, done. The Court of Appeal held that Western Power breached that broader duty of care by failing to have a system for the periodic inspection of wooden PA poles owned by consumers and used to support live electrical apparatus forming part of Western Power's electricity distribution system.

Second, the Court of Appeal upheld the finding of the trial judge that Western Power's pre-work inspection duty of care was not a non-delegable duty. If Western Power was successful in its appeal to this Court, the Herridge Parties and the IAG/Allianz Parties sought special leave to cross-appeal against that finding. Given that Western Power is unsuccessful in its appeal, and that the Herridge Parties and the IAG/Allianz Parties chose to make that application for special leave conditional, it is inappropriate to grant those parties special leave to cross-appeal in relation to non‑delegability and those applications are refused with costs. It is unnecessary to address that issue further.

Issues:-

1) What are the the applicable principles for determining the existence or otherwise of a common law duty of care allegedly owed by a statutory authority?

2) What is the existence and content of the broader duty imposed on Western Power, by reference to the terms, scope and purpose of the statutory framework and, in that context, the statutory functions and powers which Western Power in fact exercised.

Principles

There is no freestanding common law rule which fixes whether and when a common law duty of care upon a statutory authority might, or might not, arise. Statutory authorities take many forms and have different functions and powers. It is wrong to treat all statutory authorities alike.

The starting point for analysis of any common law duty of care that might be owed by any statutory authority must always be the particular statutory framework within which the statutory authority operates:-

"The existence or otherwise of a common law duty of care allegedly owed by a statutory authority turns on a close examination of the terms, scope and purpose of the relevant statutory regime. The question is whether that regime erects or facilitates a relationship between the authority and a class of persons that, in all the circumstances, displays sufficient characteristics answering the criteria for intervention by the tort of negligence."

And in formulating a common law duty, it is important to keep in mind the distinction between the existence and content of the duty (who owes the duty, whom do they owe the duty to, and what kind of risks of harm must they take reasonable care to minimise or avoid?) and questions of breach (what were the reasonable precautions required in the circumstances, and did the person discharge the duty?).

The two propositions – that there is no freestanding common law rule which fixes whether and when a common law duty of care upon a statutory authority might, or might not, arise, and that the starting point for the analysis of any such duty is the terms, scope and purpose of the applicable statutory framework – require first that the functions of the statutory authority are identified and, second, that the statutory powers that the statutory authority in fact did exercise in performance of those functions (as well as those which it could have exercised but did not) are identified.

Generally speaking, a statutory authority which is under no statutory obligation to exercise a power comes under no common law duty of care to do so: "[t]he common law does not superimpose such a duty on a mere statutory power".

This appeal is concerned with the existence and content of a duty of care that is owed in the exercise of statutory powers. But although this appeal does not concern a failure by a statutory authority to exercise particular statutory powers – powers that the statutory authority had, and which it could have exercised but did not – that does not mean that those powers that were not exercised are irrelevant. As explained below, in determining the existence and content of a duty of care arising from the statute, the whole statutory regime must be considered, including powers which have not been exercised but are interconnected with powers which have been exercised.

When a statutory authority has entered into the exercise of its statutory powers, the question is whether the relationship between the statutory authority and a class of persons affected by the manner of exercise of the power was such as to give rise to a duty of care. The focus of the analysis is upon the relevant legislation – the powers that have been exercised in the performance of the authority's statutory functions – and the positions occupied by the parties. If such a relationship is created, then "the common law imposes a duty in tort which operates alongside the rights, duties and liabilities created by statute". A duty cannot arise where it would be inconsistent or incompatible with the statutory powers or duties imposed on the statutory authority or it would be incoherent with the statutory framework.

Where a statutory authority which, consistent with its express functions, in fact "ent[e]r[s] into the field" of exercising specific powers in the discharge of its functions, the exercise of power is sometimes also described in terms of "control": the assumption of control; the taking "advantage of ... control"; or the "control exercised". However described, it is the identification of the statutory authority's powers that it in fact exercised that is critical because it is the manner of the exercise of those powers to which a common law duty of care may attach. Having identified the powers that were in fact exercised by the statutory authority in the performance of its functions, the question is: does the common law impose on the statutory authority a duty of care as to the manner of its exercise of those statutory powers (or performance of its statutory duties)? And in answering that question, it is often helpful to ask whether the statutory authority has exercised its powers to "intervene in a field of activity" in a manner which has increased the risk of harm to persons whom it had the power to protect.

Put in different terms, a statutory authority which enters upon the exercise of statutory powers with respect to a particular subject matter may place itself in a relationship to others where a common law duty of care attaches to the manner of the exercise of those powers.

As has been observed, a statutory authority which enters upon the exercise of its statutory powers with respect to one of its functions may be subject to a common law duty to exercise those powers with reasonable care.

These reasons speak of the common law imposing a duty of care on a statutory authority. That language recognises that the common law and statute interact and operate concurrently. As these reasons emphasise, the starting point of any inquiry about whether or when a statutory authority owes a common law duty to take reasonable care will be the statute and, where the authority has entered the field, what statutory powers it has exercised and in what circumstances. Holding that the statutory authority in this case owed a common law duty to exercise those powers in the discharge of its functions with reasonable care is consistent with and required by the principles of negligence that apply more generally, whether to natural persons, bodies corporate or other commercial enterprises. Here, we are concerned with a statutory authority which has exercised its statutory powers in the discharge of its statutory functions and in so doing has created relationships which give rise to a common law duty of care.

It is the functions and powers of Western Power, in the context of the terms, scope and purpose of the statutory framework, that are addressed next.

Statutory framework and Western Power

The Electricity Networks Corporation, trading as Western Power, was established in 2006 as a statutory corporation by s 4(1)(b) of the Electricity Corporations Act 2005 (WA). Pursuant to an interconnected statutory framework, Western Power undertook, operated, managed and maintained the SWIS electricity distribution system, which was used to deliver electricity to consumers' premises, including Mrs Campbell's property.

In sum, Western Power was a statutory corporation, a commercial body with a profit making purpose and with no policy making functions, which was required to act on prudent commercial principles endeavouring to make a profit from its prescribed statutory functions, including, relevantly, that of undertaking, operating, managing and maintaining the SWIS electricity distribution system as well as any works, system, facilities, apparatus or equipment required for those purposes.

Western Power's powers in the discharge of its statutory functions

The critical feature of this appeal is that Western Power stepped into the arena; it exercised specific statutory powers in performing its statutory functions of undertaking, operating, managing and maintaining the SWIS electricity distribution system as well as any works, system, facilities, apparatus or equipment required for those purposes, and had attached Mrs Campbell's premises to Western Power's distribution system and energised those premises.

So, what were the statutory powers that Western Power had in fact exercised in performing its statutory functions of undertaking, operating, managing and maintaining the SWIS electricity distribution system as well as any works, system, facilities, apparatus or equipment required for those purposes?

Under s 59(2) of the Electricity Corporations Act, Western Power was given all the powers it needed to perform its functions under the Electricity Corporations Act – including, relevant to this appeal, s 41 of the Electricity Corporations Act – or any other written law. And for the purpose of performing any function, Western Power was also given specific power, among other things, to acquire, hold, manage, improve, develop and dispose of any real or personal property; to enter into any contract or arrangement; to appoint agents or engage persons under contracts for services to provide professional, technical or other assistance to it; and to carry out any investigation, survey, exploration or boring. These specific powers did not limit the general powers of Western Power under s 59(2) or the other powers of Western Power under the Electricity Corporations Act or any other written law. Finally, it had specific power under s 49(d) of the Energy Operators (Powers) Act relevantly to "cause any distribution works or service apparatus or related things to be supported by affixing or annexing them to or against any part of a house, building or other structure".

In sum, Western Power connected Mrs Campbell's premises to its distribution system by affixing elements of its system (namely, its service cable, fuses and meter) to the PA pole, and energised her premises. In order to perform those acts, Western Power exercised statutory powers pursuant to its functions to undertake, operate, manage and maintain its distribution system, which was essential for the broader purpose of Western Power providing electricity distribution services. Western Power affixed its apparatus to the PA pole at various points, and continued to use those apparatus, pursuant to broad powers to perform its functions and specific powers to cause its distribution system to be supported by affixing it to or against any structure and its duty to connect and energise premises with power. Put in different terms, Western Power exercised its powers in the performance of its functions. It had to enter into the field or step into the arena – it had the responsibility to undertake, operate, manage and maintain the SWIS electricity distribution system, and it did.

But that is not all. The statutory framework expressly recognised that Western Power required access to land or premises to perform its functions of undertaking, operating, managing and maintaining the SWIS electricity distribution system. Western Power was therefore given the power to enter and re‑enter land or premises on which any works, apparatus or system (including any meter, fitting or connection) used by Western Power for the purpose of distributing energy to a consumer were lawfully situated.

In direct terms, Western Power exercised its powers in performing its statutory functions of undertaking, operating, managing and maintaining the SWIS electricity distribution system and any works, system, facilities, apparatus or equipment required for those purposes. In the exercise of those powers, Western Power's service cable, fuses and meter were on Mrs Campbell's land and, in particular, attached to her PA pole and those apparatus remained there as Western Power exercised its powers in performing its statutory functions of undertaking, operating, managing and maintaining the SWIS electricity distribution system. Western Power exercised those powers continuously.

Western Power's exercise of those powers therefore created a relationship between it and all other persons within the vicinity of its electricity distribution system. And a critical feature of that relationship was that Western Power exercised those powers in a manner which created or increased the risk of harm to those persons – persons it had the power to protect. The PA pole only posed the risk that it did because Western Power had attached its live electrical apparatus to it. Identification of the precise point at which Western Power's transportation of electricity using its distribution system was made to, or received by, the consumer, Mrs Campbell, was and is not determinative or necessary.

Western Power had a duty to take reasonable care in the exercise of its powers, and the content of that duty relevantly required it to avoid or minimise the risk of injury to those persons, and loss or damage to their property, from the ignition and spread of fire in connection with the delivery of electricity through its electricity distribution system – an electricity distribution system which it undertook, operated, managed and maintained in the discharge of its functions and powers by placing its apparatus on Mrs Campbell's land. The common law imposed that duty in tort on Western Power which operated alongside the rights, duties and liabilities created by statute.

Next, contrary to Western Power's contentions, the broader duty was not inconsistent or incompatible with the statutory functions and powers imposed on it. It is not necessary to that finding of duty to point to the numerous other powers which Western Power could have exercised, but did not, to take reasonable precautions to prevent the risk of harm from the ignition and spread of fire in connection with its electricity distribution system. Those powers went to questions of breach which were not in issue in this appeal. It is, however, important to consider the other powers in asking whether that duty of care was incoherent with the broader statutory framework. It was not.

On the question of coherence of the duty of care with the statutory framework, Western Power had ample power to discharge its duty of care. Some of those powers have been addressed. Examples of other powers included: the power to enter land or premises and improve works and maintain undertakings and facilities if requisite, advantageous or convenient to the exercise of Western Power's functions; the power to enter any land, premises or things not under Western Power's control or management without consent where notice has been given; the power to enter land, premises or things, relevantly, to maintain any supply system, undertaking or things; the power to do all things necessary, relevantly, for maintaining or repairing any supply system, undertaking or related works; the power to enter land without notice to clear or remove vegetation if Western Power was of the opinion that an occupier of land had not complied with their duty to do so; and the power, relevantly, to require a consumer to make adjustments to the manner of operating electrical equipment if in Western Power's opinion the consumer's operation of that equipment would interfere with supply to other consumers.

Conclusion and orders:-

For those reasons, the applications for special leave to cross-appeal filed by the Herridge Parties and the IAG/Allianz Parties, seeking to contend that the pre‑work inspection duty of care was a non‑delegable duty, are refused with costs. The appeal is dismissed with costs.

Latest Products
SSL Certificates